estadÍstica empresarial ii problemas de examen...

124
Estadística Empresarial II 21 ESTADÍSTICA EMPRESARIAL II PROBLEMAS DE EXAMEN RESUELTOS 1º) Supongamos que nos dicen que la v. a. X= “Renta anual de las personas en Valladolid” (en millones de ptas.), sigue una distribución U[0.5, 20]. Comentar esta afirmación. ¿Crees que puede ser cierta? SOLUCIÓN Existiría la misma probabilidad de superar 10.25 que de no hacerlo, de estar entre 0.5 y 1 que de estar entre 19.5 y 20 (ser muy rico o en absoluto). Luego esta afirmación no puede ser cierta. 2º) Sea X una v. a. con distribución binomial, siendo n = 1 000, y su media 500. Calcular la probabilidad de que X sea menor que 480, comentando los pasos. SOLUCIÓN Puesto que X B(1 000,p) su E(X) = np = 500 p = 0.5 luego X B(1 000,0.5), y la probabilidad pedida sería: p(X < 480) = 1 000 k 0.5 0.5 k 1 000 k k 0 479 = difícil de obtener. Entonces, como toda variable binomial se puede escribir como suma de variables aleatorias independientes idénticamente distribuidas de la siguiente forma: X = X i i 1 n = tal que X i = 1 exito p 0 fracaso q con X i v. a. i. i. d. podemos aplicar el Teorema Central del Límite y tendremos: p(X < 480 ) = p X np npq 480 np npq < = p X 500 250 480 - 500 250 < Φ(-1.26) = 0.1038 Aplicando el Teorema Central del Límite 0.5 20 10.25 X f X

Upload: others

Post on 01-Apr-2021

4 views

Category:

Documents


0 download

TRANSCRIPT

Page 1: ESTADÍSTICA EMPRESARIAL II PROBLEMAS DE EXAMEN …media0.webgarden.es/files/media0:4bd76ebb98146.pdf.upl... · 2010. 4. 27. · Estadística Empresarial II 23 La correlación negativa

Estadística Empresarial II 21

ESTADÍSTICA EMPRESARIAL II

PROBLEMAS DE EXAMEN RESUELTOS

1º) Supongamos que nos dicen que la v. a. X= “Renta anual de las personas en Valladolid” (enmillones de ptas.), sigue una distribución U[0.5, 20].Comentar esta afirmación. ¿Crees que puede ser cierta?

SOLUCIÓN

Existiría la misma probabilidad de superar 10.25que de no hacerlo, de estar entre 0.5 y 1 que deestar entre 19.5 y 20 (ser muy rico o en absoluto).Luego esta afirmación no puede ser cierta.

2º) Sea X una v. a. con distribución binomial, siendo n = 1 000, y su media 500. Calcular laprobabilidad de que X sea menor que 480, comentando los pasos.

SOLUCIÓN

Puesto que X → B(1 000,p) su E(X) = np = 500 ⇒ p = 0.5

luego X → B(1 000,0.5), y la probabilidad pedida sería:

p(X < 480) = 1 000

k 0.5 0.5 k 1 000 k

k 0

479

=∑

difícil de obtener.

Entonces, como toda variable binomial se puede escribir como suma de variables aleatoriasindependientes idénticamente distribuidas de la siguiente forma:

X = Xii 1

n

=∑ tal que Xi =

1 exito p 0 fracaso q

con Xi v. a. i. i. d.

podemos aplicar el Teorema Central del Límite y tendremos:

p(X < 480 ) = pX np

npq

480 npnpq

−<

= p

X 500250

480 - 500

250−

<

≈ Φ(-1.26) = 0.1038

Aplicando el Teorema Central del Límite

0.5 2010.25 X

fX

Page 2: ESTADÍSTICA EMPRESARIAL II PROBLEMAS DE EXAMEN …media0.webgarden.es/files/media0:4bd76ebb98146.pdf.upl... · 2010. 4. 27. · Estadística Empresarial II 23 La correlación negativa

22 Problemas de examen resueltos

3º) Se desea estudiar la relación entre el precio (X) y la cantidad vendida (Y) de undeterminado producto. Para de diferentes establecimientos que venden dicho producto, se haobtenido la siguiente tabla de la ley de probabilidad conjunta de ambas variables:

YX 1 2 3

20 0 0.1 0.430 0 0.2 040 0.3 0 0

a) Calcular el valor del coeficiente de correlación. Comentar el resultado obtenido en relacióncon las variables.

b) Sin realizar ningún cálculo, si nos dicen que en una determinada tienda, el precio delproducto es 38, ¿cuál sería aproximadamente el número de unidades vendida?

c) Calcular p(XY > 50).

SOLUCIÓN

YX 1 2 3 p(X = x)

20 0 0.1 0.4 0.530 0 0.2 0 0.240 0.3 0 0 0.3

p(Y = y) 0.3 0.3 0.4 1

a) Para calcular el coeficiente de correlación tenemos que calcular la covarianza entre X e Y y lasdesviaciones típicas de ambas variables, y para ello necesitamos obtener las esperanzas siguientes:

E(X) = (20x0.5) + (30x0.2) + (40x0.3) = 28

E(X2) = (202x0.5) + (302x0.2) + (402x0.3) = 860

E(Y) = (1x0.3) + (2x0.3) + (3x0.4) = 2.1

E(Y2) = (12x0.3) + (22x0.3) + (32x0.4) = 5.1

E(XY) = (20x1x0) + (30x1x0) + (40x1x0.3) + (20x2x0.1) + (30x2x0.2) + (40x2x0) ++ (20x3x0.4) + (30x3x0) + (40x3x0) = 52

Var(X) = E(X2) - E2(X) = 860 - 282 = 76

Var(Y) = E(Y2) - E2(Y) = 5.1 - 2.12 = 0.69

Cov (X,Y) = E(XY) - E(X)E(Y) = 52 - (28x2.1) = -6.8

ρX,Y Cov (X,Y)

Var(X) Var(Y)= =

− 6.876 x 0.69

= - 0.9390259

Page 3: ESTADÍSTICA EMPRESARIAL II PROBLEMAS DE EXAMEN …media0.webgarden.es/files/media0:4bd76ebb98146.pdf.upl... · 2010. 4. 27. · Estadística Empresarial II 23 La correlación negativa

Estadística Empresarial II 23

La correlación negativa nos indica que la relación entre ambas variables es inversa, es decir, cuandoaumenta una variable la otra disminuye y al revés, además como esta muy próxima a -1, nos dicetambién que el grado de relación o de dependencia entre ambas variables es muy alto.

b) Como hemos dicho en el apartado anterior, a medida que aumenta la X, la Y disminuye por queel coeficiente de correlación es negativo, luego si el precio es 38, un valor cercano a 40, la cantidadvendida lo será a 1 unidad.

c) Obtengamos la distribución de XY

p(XY = 40) = p(X = 20, Y = 2) + p(X = 40, Y = 1) = 0.1 + 0.3 = 0.4

p(XY = 60) = p(X = 20, Y = 3) + p(X = 30, Y = 2) = 0.4 + 0.2 = 0.6

luego:

XY p(XY = k)40 0.460 0.6

con lo que

p(XY > 50) = p(XY = 60) = 0.6.

4º) El número de unidades vendidas al día de tres productos A, B y C son v. a. que siguenrespectivamente distribuciones N(100, 10), N(200, 15) y N(150, 10). Si los precios de dichosproductos son 150, 200 y 100 ptas. se pide:

a) Calcular la distribución de la v. a. ingresos diarios (I).b) Calcular p( I > 61 580) y el valor de k tal que p( I > k) = 0.975.c) Calcular la moda de la v. a. ingresos diarios (No se necesita realizar ningún cálculo).d) Calcular la probabilidad de que los ingresos mensuales (20 días) estén entre 1 375 000 y1 383 000.

SOLUCIÓN

Definimos las siguientes variables:

A = “Número de unidades vendidas del producto A” → N(100,10)B = “Número de unidades vendidas del producto B” → N(200,15)C = “Número de unidades vendidas del producto C” → N(150,10)

a) Sea la v. a. I = “Ingresos diarios” = 150 A + 200 B + 100 C

E(I) = 150E(A) + 200E(B) + 100E(C) = (150x100) + (200x200) + (100x150) = 70 000

Var(I) = 1502Var(A) + 2002Var(B) + 1002Var(C) = (1502x102) + (2002x152) + (1002x102) == 12 250 000

σI = 3 500

Page 4: ESTADÍSTICA EMPRESARIAL II PROBLEMAS DE EXAMEN …media0.webgarden.es/files/media0:4bd76ebb98146.pdf.upl... · 2010. 4. 27. · Estadística Empresarial II 23 La correlación negativa

24 Problemas de examen resueltos

luego

I → N(70 000,3 500)

b) p( I > 61 580) = pI 70 000

3 500

61 580 70 0003 500

>−

= 1 - Φ(-2.40) = 1 - 0.0082 = 0.9918

0.975 = p( I > k) = pI 70 000

3 500

k 70 0003 500

1-k 70 000

3 500−

>−

=

Φ

⇒ Φk 70 000

3 500−

= 0.025

⇒k 70 000

3 500−

= -1.96 ⇒ k = 63 140

c) I sigue una distribución unimodal y simétrica como es la distribución normal, en la que la modacoincide con la media, luego

Moda de I = Media de I = 70 000

d) Sean I1, I2, I3, …, I20 v. a. i. i. d. con una distribución N(70 000, 3 500) y sea

IM = “Ingreso mensual” = I1 + I2 + I3 + … + I20

E(IM) = E(I1 + … + I20) = 20E(Ii) = 14 x 105

Var(IM) = Var(I1 + … + I20) = 20Var(Ii) = 245 x 106

Con lo que

IM → N(14x105,103x 245 )

p( 1 375 000 < IM < 1 383 000) =

= p1 375 000 1 400 000

10 245

IM 1 400 00010 245

1 383 000 1 400 000

10 245−

<−

<−

=

= p(-1.59 < Z < -1.08) = Φ(-1.08) - Φ(-1.59) = 0.1401 - 0.0559 = 0.0842

5º) Sea (X, Y) una v. a. bidimensional con función de densidad conjunta:

f (x, y) k 1 y x 10 en el restoX,Y =

− < < <

Page 5: ESTADÍSTICA EMPRESARIAL II PROBLEMAS DE EXAMEN …media0.webgarden.es/files/media0:4bd76ebb98146.pdf.upl... · 2010. 4. 27. · Estadística Empresarial II 23 La correlación negativa

Estadística Empresarial II 25

a) Hallar el valor de k.b) Comprobar si las v. a. X e Y son o no independientes.c) Calcular E(3X2 + Y).d) Calcular la función de distribución de la v. a. X.e) Sea Z = 5 - Y/2. Calcular la función de densidad de Z.f) Calcular la función de densidad de X condicionada a que Y = 1/4 y calcular ( ).1/4Y

1/2Xp =≥

SOLUCIÓN

a) k dy dxy 1

y x

x 1

x 1

=−

=

=−

=

∫∫

= ( ] k y dx y=-1

y=x

x 1

x 1

=−

=

∫ =

= k (x 1) dx x 1

x 1

+=−

=

∫ = k ( x 1)

2

2

x 1

x=1+

=

=−

2k = 1 ⇒

⇒ k = 1/2

b) f (x) 12

dyXy= 1

y=x

=−∫ =

y2 1

y=x

=−y

= x 1

2+

f (x)

x 12

1 x 1

0 en el restoX =

+− < <

f (y) 12

dxYx=y

x=1

= ∫ = x2 x=y

x=1

= 1 y

2−

f (y)

1 - y2

1 y 1

0 en el restoY =

− < <

Como el producto de las funciones de densidad marginales no es la función de densidad conjunta,las variables X e Y no son independientes.

c) E(X2) = xx 1

2 dx2

x

x=1 +

=−∫

1

= x x

2 dx

3 2

x

x=1 +

=−∫

1

=

x4

x

2

4 3

1

x=1

+

=−

3

x

= 1/3

1-1

Page 6: ESTADÍSTICA EMPRESARIAL II PROBLEMAS DE EXAMEN …media0.webgarden.es/files/media0:4bd76ebb98146.pdf.upl... · 2010. 4. 27. · Estadística Empresarial II 23 La correlación negativa

26 Problemas de examen resueltos

E(Y) = y 1 y

2 dy

y=

=

∫1

1y

= y y

2 dy

2

y=

y 1 −

=

∫1

=

x2

y3

2

2 3

1

x=1

=−x

= -1/3

E(3X2 +Y) = 3 E(X2) + E(Y) = (3 x 1/3) - 1/3 = 2/3

d) Al ser t 1

2 dt

t=

t x +

=

∫1

= ( t 1)

4

2

1

t=x+

=−t

= (x 1)

4

2+

la función de distribución de la v. a. X es:

F (x) X = p(X ≤ x) =

0 x 1

(x 1)4

1 x 1

1 x 1

2

≤ −

+− ≤ ≤

e) Sea la v. a. Z = 5 - Y2

, para calcular la función de densidad de Z empezaremos viendo que

relación hay entre las funciones de distribución de Z y de Y, y luego derivando tendremos larelación entre las respectivas funciones de densidad.

F (z) Z = p(Z ≤ z) = p(5 - Y2

≤ z) = p(- Y2

≤ z - 5) = p(Y ≥ 10 - 2z) = 1 - F (10 2z)Y −

f (z) f (10 2z) ( 2) 2 f (10 2z) 2 ( z 92

) 2z 9Z Y Y= − − − = − = − = −

Además, como -1 < y < 1 ⇒ -1 < 10 - 2z < 1 ⇒ -11 < -2z < -9 ⇒ 9/2 < z < 11/2

(También se puede hacer -1 < y < 1 ⇒ 1 > - y > -1 ⇒ 1/2 > -y/2 > -1/2 ⇒ 11/2 > z > 9/2)

tenemos finalmente que la función de densidad de Z es:

f (z)2z 9

92

z112

0 en el restoZ =

− < <

f) Nos piden calcular f (x),XY=1 4/

lo haremos en general y luego particularizaremos en Y = 1/4.

Page 7: ESTADÍSTICA EMPRESARIAL II PROBLEMAS DE EXAMEN …media0.webgarden.es/files/media0:4bd76ebb98146.pdf.upl... · 2010. 4. 27. · Estadística Empresarial II 23 La correlación negativa

Estadística Empresarial II 27

f (x) XY y=

=f (x, y)

f (y)X,Y

Y =

1 / 2(1 y) / 2−

= 1

1 y−

luego

f (x) XY y=

=

11 y

y x 1

0 en el resto

−< <

⇒ f (x) XY=

=1 4/

11 1

1x 1

0 en el resto−

< <

/ 4 4

con lo que

( )p X 1 / 2Y 1 / 4

>= =

43

dx x=1/2

x=

∫1

= 4x3 x=1/2

x=1

= 2/3.

6º) Sabemos que los ingresos de una empresa son una v. a. X cuya función de densidad es unade estas:

a) ¿Con cual de las dos es mayor el ingreso medio?b) ¿Cual de las dos tiene mayor dispersión?c) ¿En cual de ellas la p(350 < X < 500) será mayor? Razona la respuesta.

SOLUCIÓN

a) A la vista de los gráficos ambas funciones de densidad tienen la misma esperanza, 500.

b) La segunda tiene los valores más dispersos que la primera que les tiene más agrupados.La mas favorable para la empresa será la primera por tener mayor seguridad en los ingresos (menosriesgo).

c) La p(350 < X < 500) será mayor en la primera, por haber menor dispersión.

7º) Sea (X, Y) una v. a. bidimensional con función de densidad conjunta:

f (x, y)

x2

0 x 2, 0 y 1

0 en el restoX,Y =

< < < <

Función de densidad

250 350 450 550 650 7500

2

4

6

8(X 0,001) Función de densidad

250 350 450 550 650 7500

1

2

3

4(X 0,001)

Page 8: ESTADÍSTICA EMPRESARIAL II PROBLEMAS DE EXAMEN …media0.webgarden.es/files/media0:4bd76ebb98146.pdf.upl... · 2010. 4. 27. · Estadística Empresarial II 23 La correlación negativa

28 Problemas de examen resueltos

a) Calcular las marginales de X e Y.b) Calcular la p(X >1).c) Calcular Var(X+Y) y Var(X-Y).

SOLUCIÓN

a) Función de densidad marginal de X:

f (x) = 12

x dyXy=0

y=1

∫ = x2

y x2

y=1

==y 0

f (x)

x 2

x 2

0 en el restoX =

< <

0

Función de densidad marginal de Y:

f (y) 12

x dxYx=0

x=2

= ∫ = x4

2

x=0

x=2

= 1 f (y)

1 y 10 en el restoY =

< <

0

X e Y son independientes, pues la función de densidad conjunta coincide con el producto de lasfunciones de densidad marginales.

b) p(X >1) = f (x) dxXx=1

x=∞

∫ = 12

x dxx=1

x=2

∫ = x4

2

x=1

x=2

= 1-

14

= 3/4.

c) Por ser independientes,

Var(X+Y) = Var(X) + Var(Y)

Var(X-Y) = Var(X) + (-1)2Var(Y) = Var(X) + Var(Y)

E(X) = x 12

x dxx=0

x=2

∫ = x6

3

x=0

x=2

= 4 3/

E(X2) = x 12

x dx2

x=0

x=2

∫ = x8

4

x=0

x=2

= 2

Var(X) = E(X2) - E2(X) = 2 - (4/3)2 = 2/9

E(Y) = y 1 dyy=0

y=1

∫ =y2

2

y=0

y=1

= 1 2/

E(Y2) = y 1 dyy=0

y=12∫ =

y3

3

y=0

y=1

= 1 3/

Var(Y) = E(Y2) - E2(Y) = 1/3 - (1/2)2 = 1/12

Page 9: ESTADÍSTICA EMPRESARIAL II PROBLEMAS DE EXAMEN …media0.webgarden.es/files/media0:4bd76ebb98146.pdf.upl... · 2010. 4. 27. · Estadística Empresarial II 23 La correlación negativa

Estadística Empresarial II 29

luego

Var(X+Y) = Var(X-Y) = Var(X) + Var(Y) = 2/9 + 1/12 = 11/36

8º) Un estudio de Arquitectura tiene tres tipos de ingresos mensuales independientes, porTarifa de Edificación, de Urbanismo y por Peritaciones, que siguen distribuciones normalesN(14, 2), N(12, 4) y N(3, 5 ), respectivamente, medidas en millones de ptas. Se pide:a) Calcular la probabilidad de que los ingresos totales mensuales sean mayores de 30millones.b) Calcular la probabilidad de no cubrir gastos si estos se estiman en 14.5 millones mensuales.c) Si consideramos 12 meses, ¿cuál es la probabilidad de que al menos un mes no se cubrangastos?

SOLUCIÓN

Consideremos las variables siguientes:

X = “Ingresos mensuales Tarifa de Edificación” → N(14, 2)Y = “Ingresos mensuales Urbanismo” → N(12, 4)Z = “Ingresos mensuales Peritaciones” → N(3, 5 )

a) Sea I = “Ingresos totales” = X + Y + Z → N(29, 5) ya que:

E(I) = E(X + Y + Z) = E(X) + E(Y) + E(Z) = 14 + 12 + 3 = 29

Var(I) = Var (X + Y + Z) = Var(X) + Var(Y) + Var(Z) = 22 + 42 + ( 5 )2 = 25

con lo cual la probabilidad pedida es:

p(I > 30) = pI 29

5

30 295

>−

= p(Z > 0.2) = 1 - Φ(0.2) = 1 - 0.5793 = 0.4207.

b) p(I < 14.5) = pI 29

5

14.5 295

<−

= p(Z < -2.9) = Φ(-2.9) = 0.0019

c) Llamamos Ti = 1 si en ese mes no se cubren gastos p = 0.00190 si en ese mes si se cubren gastos q = 0.9981

Entonces S Tii 1

12

==∑ → B(12, 0.0019)

p(S ≥ 1) = 1 - p(S = 0) = 1 - (0.9981)12 = 0.0225632.

9º) Una industria conservera obtiene al día, por termino medio, 0.5 toneladas de tomate enconserva, con una desviación típica de 30 kilogramos.

Page 10: ESTADÍSTICA EMPRESARIAL II PROBLEMAS DE EXAMEN …media0.webgarden.es/files/media0:4bd76ebb98146.pdf.upl... · 2010. 4. 27. · Estadística Empresarial II 23 La correlación negativa

30 Problemas de examen resueltos

a) Si la industria conservera vende su producto al precio de 100 pesetas el kilogramo y loscostes de elaboración, etc., son de 20 pesetas por kilogramo, más unos gastos fijos de 10 000pesetas, calcular la probabilidad de que el beneficio en un trimestre (90 días) sea mayor que4 555 000 pesetas. Comentar el significado de dicha probabilidad.b) ¿Cuantos días tardarán en obtener al menos 100 toneladas con una probabilidad del97.5%?

SOLUCIÓN

a) Sea Xi = “Kilogramos de tomate en el día i-ésimo que obtiene la empresa”. De esta variableconocemos:

E(Xi) = 500 Kg. σXi = 30 Kg.

Definimos Bi = “Beneficio del día i-ésimo” = Ii - Ci = 100 Xi - (20Xi + 10 000) = 80Xi - 10 000

E(Bi) = E(80Xi - 10 000) = 80E(Xi) - 10 000 = 80x500 - 10 000 = 30 000

Var(Bi) = Var(80Xi - 10 000) = 802 Var(Xi) = 802 x 302 = 5760000

Consideremos la variable B = “Beneficio en un trimestre” = Bii 1

90

=∑

Calculemos cuanto valen la esperanza y la varianza de B.

E(B) = E( Bii 1

90

=∑ ) = E(Bi

i 1

90

)=∑ = 2 700 000

Var(B) = Var( Bii 1

90

=∑ ) = Var (Bi

i 1

90

)=∑ = 90 x 5 760 000 ⇒ σB = 22 768.39915

Como B1, …, B90 son v. a. i. i. d. con esperanza y varianza finita, a la variable B le podemos aplicarel Teorema Central del Límite.

p( B > 4 555 000) = pB 2 700 0022 768.39915

4 555 000 2 700 00

22 768.39915−

>−

≈ 1 - Φ(81.4725) = 0

Por el Teorema Central del Límite

b) Sean X1, …, Xn v. a. i. i. d., y sea S Xii 1

n

==∑

entonces

E(S) = 500n Var(B) = 900n

Hay que calcular un valor de n tal que p X 100 000 0.975 ii 1

n

=∑ ≥

=

Page 11: ESTADÍSTICA EMPRESARIAL II PROBLEMAS DE EXAMEN …media0.webgarden.es/files/media0:4bd76ebb98146.pdf.upl... · 2010. 4. 27. · Estadística Empresarial II 23 La correlación negativa

Estadística Empresarial II 31

Como no conocemos la distribución de S y es suma de variables aleatorias independientes eidénticamente distribuidas con esperanza y varianza finita, podemos aplicar el Teorema Central delLímite para obtener el valor de n.

p(S ≥ 100 000) = pS 500n

30 n

100 000 500n30 n

−≥

≈ 1 - Φ100 000 - 500n

30 n

= 0.975

⇒ 100 000 - 500n

30 n = -1.96 ⇒ n ≈ 202.

10º) Sea X el tiempo que tarda un taller en detectar una avería en un vehículo, desde elinstante en el que entra, dado en horas. Sea Y el tiempo que transcurre desde que el vehículoentra hasta que finalmente es reparado (horas). Una vez detectado el problema, comienza lareparación.

Se sabe, en teoría, que ambas variables se relacionan probabilísticamente según la funciónde densidad conjunta siguiente:

( )f x, y12

x eX Y2 y

, = − , 0 < x < y < ∞, y nula en el resto.

a) Interpretar las restricciones de fX,Y(x,y), según el contenido del problema.b) Deducir qué distribuciones siguen ambos tiempos. ¿Son independientes?c) Si Z es el tiempo que se emplea desde que se localiza el fallo hasta que se arregla, calcular,

por término medio, cuánto debemos esperar. Calcular también la varianza de Z.d) Si el tiempo que permanece un vehículo en el taller, desde que entra hasta que sale

reparado, son 3 horas, ¿cuánto tiempo es de esperar que se tarde en averiguar su avería?

(Nota: y e dy k! k y

y 0

y−

=

=∞

∫ = )

SOLUCIÓN

a) Como ambas variables miden tiempos, toman valores positivos. Además el tiempo que tarda untaller en detectar una avería en un vehículo, desde el instante en el que entra, X es siempre menorque el tiempo que transcurre desde que el vehículo entra hasta que finalmente es reparado, Y. Elvalor infinito le toman cuando no hay posibilidad de reparación.

b) Con el fin de conocer que distribuciones siguen tanto X como Y, calcularemos sus densidadesmarginales

Función de densidad marginal de X:

( ] f (x) 12

x e dy 12

x e 12

x e X2 y

y x

y2 y

y x

y 2 x= = − =−

=

=∞−

=

=∞ −∫ 0 < x < ∞

Page 12: ESTADÍSTICA EMPRESARIAL II PROBLEMAS DE EXAMEN …media0.webgarden.es/files/media0:4bd76ebb98146.pdf.upl... · 2010. 4. 27. · Estadística Empresarial II 23 La correlación negativa

32 Problemas de examen resueltos

f (x)

12

x e x 0

0 en el restoX

2 x

=>

X → γ(1,3) pues

f (x)

a(p)

x e x 0

0 resto

12!

x e x 0

0 restoX

p 1 ax 3 1 x

=>

=

>

− − − −

Γ

Función de densidad marginal de Y:

f (y) 12

x e dy e 16

x 16

y e Y2 y

x

x=yy 3

x=0

x=y3= =

=−

=

− −∫0

y 0 < y < ∞

f (y)

16

y e y 0

0 en el restoY

3 y

=>

Y → γ(1,4) ya que

f (y)

a(p)

y e y 0

0 resto

13!

y e y 0

0 restoY

p 1 ay 4 1 y

=>

=

>

− − − −

Γ

Como en general fX,Y(x,y) ≠ fX(x) fY(y) las variables X e Y son dependientes.

c) Sea Z = Y - X

E(Z) = E(Y) - E(X) = 41

31

− = 1 (la esperanza de una γ(a,p) es pa

)

E(YX) = yx 12

x e dx dy 12

y e x4

dy 2 y

x 0

x y

y 0

yy

4

x 0

x y

y 0

y−

=

=

=

=∞−

=

=

=

=∞

∫∫ ∫

=

=

18

y e dy 5 y

y 0

y−

=

=∞

∫ =

18

y e dy 5!8

15 5 y

y 0

y

= = =−

=

=∞

usando la nota del enunciado

Page 13: ESTADÍSTICA EMPRESARIAL II PROBLEMAS DE EXAMEN …media0.webgarden.es/files/media0:4bd76ebb98146.pdf.upl... · 2010. 4. 27. · Estadística Empresarial II 23 La correlación negativa

Estadística Empresarial II 33

Cov(Y,X) = E(YX) - E(Y)E(X) = 15 - 4x3 = 3

Var(Z) = Var(Y) + Var(X) - 2Cov(Y,X) = 41

3

1 2 x 3 12 2+ − = (la varianza de una γ(a,p) es

pa2 )

d) f (x) f (x, y)

f (y)

12

x e

16

y e

3 xy

X/Y yX,Y

Y

2 y

3 y

2

3=

−= = = 0 < x < y siempre que y > 0

f (x) 3 x3

=x

X/Y 3

2

3

2

= =9

0 < x < 3

E(X / Y 3) x x9

dx x36

94

2.25 2

x 0

x 3 4

x 0

x 3

= = =

= =

=

=

=

=

11º) Sea la v. a. X = “Ventas diarias de harina” (kilogramos), con distribución N(3 000, 200).

a) Si suponemos que otra fábrica vende diariamente 1 000 kg. de harina, más el doble de loque vende la primera, ¿cuál será la esperanza y la desviación típica de las ventas de lasegunda fábrica?b) Supongamos que la primera fábrica vende su producto a 50 ptas. el kilogramo. ¿Cuál serála distribución de la v. a. I = “Ingresos mensuales de la primera fábrica”, considerando quevende 30 días al mes? (Representar gráficamente, de forma aproximada, la distribución).c) Calcular y representar la probabilidad de que los ingresos mensuales estén comprendidosentre 4 420 000 y 4 580 000 ptas.

SOLUCIÓN

Sea X = “Ventas diarias de harina” (en kg.) → N(3 000,200)

a) Definimos Y = 2X + 1 000

E(Y) = E(2X + 1 000) = 2E(X) + 1000 = 2x3 000 + 1 000 = 7 000

Var(Y) = Var(2X + 1 000) = 4Var(X) = 4x2002 = 160 000 ⇒ σY = 400

b) Para cada día: Ii = 50Xi → N(150 000,10 000)

En un mes I = “Ingresos mensuales de la 1ª fábrica” = I1 + … + I30 → N(4 500 000, 30 10 000)

Page 14: ESTADÍSTICA EMPRESARIAL II PROBLEMAS DE EXAMEN …media0.webgarden.es/files/media0:4bd76ebb98146.pdf.upl... · 2010. 4. 27. · Estadística Empresarial II 23 La correlación negativa

34 Problemas de examen resueltos

c) p(4 420 000 < I < 4 580 000) = p4 420 000 4 500 000

30 1 000 Z

4 580 000 4 500 00030 1 000

< <−

=

= p(-1.46 < Z < 1.46) = Φ(1.46) - Φ(-1.46) = 0.9279 - 0.0721 = 0.8558.

12º) Una empresa de transporte tiene tres tipos de camiones. El número medio de litros decombustible que repostan los camiones de tipo A es de 300 litros, con una desviación de 40; losde tipo B, 250 de media y desviación de 30; los de tipo C, 175 y 25.

a) Si en una semana repostan 51 camiones de tipo A, 42 de tipo B y 40 de tipo C, ¿cuál es laprobabilidad de que el número total de litros repostado esté entre 32 500 y 33 500 litros?

b) Si la empresa piensa destinar al consumo de combustible para los camiones de tipo Adurante otra semana un millón quinientas mil pesetas, ¿cuál es el número máximo decamiones de tipo A que podrán repostar, con una probabilidad del 95%, sabiendo que esasemana el combustible estará a 100 ptas./litro?

SOLUCIÓN

Definimos las variables: A = número de litros de combustible que repostan los camiones de tipo AB = número de litros de combustible que repostan los camiones de tipo BC = número de litros de combustible que repostan los camiones de tipo C

Desconocemos la distribución de estas variables. Sólo conocemos sus esperanzas y sus varianzas,por lo que aplicaremos el Teorema Central del Límite.

a) Consideremos la variable aleatoria

X = número total de litros repostados = A B C ii 1

51

ii 1

42

ii 1

40

= = =∑ ∑ ∑+ +

E(X) = 51E(A) + 42E(B) + 40E(C) = 51x300 + 42x250 + 40x175 = 32 800

Var(X) = 51Var(A) + 42Var(B) + 40Var(C) = 51x402 + 42x302 + 40x252 = 144 400

luego σX = 380

42 43 44 45 46 47 48(X 100 000)

0

2

4

6

8(X 0.000001)

Page 15: ESTADÍSTICA EMPRESARIAL II PROBLEMAS DE EXAMEN …media0.webgarden.es/files/media0:4bd76ebb98146.pdf.upl... · 2010. 4. 27. · Estadística Empresarial II 23 La correlación negativa

Estadística Empresarial II 35

p(32 500 < X < 33 500) ≈ p32 500 32 800

380 Z

33 500 32 800380

< <−

= p(-0.79 < Z < 1.84) =

Aplicando el Teorema Central del Límite y tipificando

= Φ(1.84) - Φ(-0.79) = 0.9671 - 0.2148 = 0.7523.

b) Sea Y = 100 A ii 1

n

=∑

E(Y) = 100 n E(A) = 30 000n

Var(Y) = 1002 n Var(A) = 16 000 000n ⇒ σY 4 000 n =

p(Y < 1 500 000) ≈ p Z 1 500 000 30 000 n

4 000 n <

= 0.95

Aplicando el Teorema Central del Límite y tipificando

⇒ 1 500 000 30 000 n

4 000 n−

= 1645. ⇒ 30 000n + 6 580 n -1 500 000 = 0

luego

n 6 580 6 580 4 x 3 000 x 1 500 000

60 000

6.96225157.1815848

2

=− ± +

=−

con lo que

n 48.4729459551.57516024=

la solución n = 51.57 la tenemos que desechar pues al sustituir su valor en la probabilidad de partidano da 0.95, que es el valor pedido, sino que da una probabilidad de 0.05.

Por tanto el valor pedido, redondeando, es n = 48.

13º) En una pequeña empresa, consideramos las siguientes características:

X = “Número de empleados”Y = “Número de artículos fabricados diariamente”

Page 16: ESTADÍSTICA EMPRESARIAL II PROBLEMAS DE EXAMEN …media0.webgarden.es/files/media0:4bd76ebb98146.pdf.upl... · 2010. 4. 27. · Estadística Empresarial II 23 La correlación negativa

36 Problemas de examen resueltos

como variables aleatorias. La ley de probabilidad conjunta es la siguiente:

X \ Y 8 10 123 2/9 0 04 0 3/9 1/95 0 1/9 2/9

a) ¿Son independientes X e Y?b) Calcular ( ) p X 4

Y 10 .≥≤

c) Obtener el valor de p(3X + 2 ≤ 14).d) Calcular al covarianza de X e Y.e) Indicar, sin hacer ningún cálculo, basándose únicamente en los valores de la tabla anterior,cuál es el valor aproximado del coeficiente de correlación entre X e Y.

SOLUCIÓN

a) Para que dos variables sean independientes se tiene que verificar que:

fXY(x,y) = fX(x)fY(y) ∀ x,y ∈ R

X \ Y 8 10 12 p(X = x)3 2/9 0 0 2/94 0 3/9 1/9 4/95 0 1/9 2/9 3/9

p(Y = y) 2/9 4/9 3/9 1

Como fXY(3,8) = 2/9 ≠ 2/9 x 2/9 = fX(3)fY(8) ⇒ X e Y son dependientes

b) ( ) ( )( ) p X 4

Y 10 p X 4, Y 10

p Y 10 ≥

≤ =≥ ≤

≤=

( ) ( ) ( ) ( )( ) ( )

p X 4, Y 8 p X 4, Y 10 p X 5, Y 8 p X 5, Y p Y 8 p Y 10

== = + = = + = = + = =

= + ==

10

= 0 3 / 9 0 1 / 9

2 / 9 4 / 9

23

+ + +

+=

c) p(3X + 2 ≤ 14) = p(3X ≤ 12) = p(X ≤ 4) = 29

49

23

+ =

Page 17: ESTADÍSTICA EMPRESARIAL II PROBLEMAS DE EXAMEN …media0.webgarden.es/files/media0:4bd76ebb98146.pdf.upl... · 2010. 4. 27. · Estadística Empresarial II 23 La correlación negativa

Estadística Empresarial II 37

d) E(X) = 3 29

4 49

5 39

379

+ + =

E(Y) =8 29

10 49

12 39

929

+ + =

E(XY) = 3x8 29

3x10x0 3x12x0 +4x8x0 4x10 39

4x12 19

+ 5x8x0 5x10 19

5x12 29

+ + + + + + =

= 3869

Cov(X,Y) = E(XY) - E(X)E(Y) = 3869

379

929

7081

0.86419753. − = =

e) El coeficiente de correlación varía entre -1 y 1. En este caso como la covarianza calculada en elapartado anterior es positiva eso no quiere decir que el coeficiente de correlación es positivo,además, por el primer apartado sabemos que es mayor que cero pues las variables son dependientes.De la observación de la tabla de probabilidades conjuntas, podemos deducir que tendrá un valorrelativamente alto, puesto que a valores bajos de X, la Y toma también valores bajos y para valoresintermedios o altos de X, predominan los valores intermedios o altos de Y.

14º) Sea (X,Y) una v. a. bidimensional, con función de densidad conjunta:

f (x, y) 2x e 0 x 1; y 0

0 en el restoX Y

y

, =< < >

a) Las variables X e Y, ¿son independientes?b) ¿Qué distribución sigue cada una de ellas?c) Calcular el coeficiente de correlación de estas variables y comentar su significado.d) Hallar el valor de Var(5X + 2Y).

SOLUCIÓN

a) Dos variables son independientes si se verifica que:

fX,Y(x,y) = fX(x)fY(y) ∀ x,y ∈ R

Calculemos las marginales de las variables X e Y.

( ] f (x) 2x e dy 2x e 2x Xy

y 0

yy

y 0

y= = − =−

=

=∞−

=

=∞

∫ 0 < x < 1

( ] f (y) 2x e dx e x eYy

x 0

x 1-y 2

x 0

x 1 -y= = =−

=

=

=

=

∫ y > 0

Page 18: ESTADÍSTICA EMPRESARIAL II PROBLEMAS DE EXAMEN …media0.webgarden.es/files/media0:4bd76ebb98146.pdf.upl... · 2010. 4. 27. · Estadística Empresarial II 23 La correlación negativa

38 Problemas de examen resueltos

fX,Y(x,y) = 2y e x 0, 0 y 1

0 en el resto

x− > < <

= fX(x) fY(y)

luego las variables X e Y son independientes.

b) X → β(2,1)

f (x)

1(p,q)

x (1 x) 0 x 1

0 resto

2 x 0 x 10 restoX

p 1 q 1

=− < <

=

< <

− −

β

pues

1

(2,1)

(2 1)(2) (1)

2!

1! 0! 2

β=

+= =

ΓΓ Γ

Y → ε(1)

f (y) a e y 0

0 resto

e y 00 restoY

ay y

=>

=

>

− −

c) Como X e Y son independientes, son incorreladas, luego ρX,Y = 0.

d) Como sabemos que distribución sigue tanto X como Y, calculamos su varianza por la fórmula

Var(X) = pq

(p q 1)(p q)

2 x 1(2 1 1)(2 1)

1

182 2+ + +=

+ + +=

Var(Y) = 1a

11

1 2 2= =

Si no conocemos que distribución sigue X ni Y tendríamos que empezar calculando la esperanza yla varianza de X y de Y.

E(X) = x 2x dx 23

x 23

x 0

x 13

x 0

x 1

=

=

=

=

∫ =

=

E(Y) = ( ] ( ]y e dy - y e - e dy 0 e 1 y 0

y

y 0

y

y 0

y

y 0

y−

=

=∞−

=

=∞ −

=

=∞−

=

=∞

∫ ∫= − = − =y y y y

Page 19: ESTADÍSTICA EMPRESARIAL II PROBLEMAS DE EXAMEN …media0.webgarden.es/files/media0:4bd76ebb98146.pdf.upl... · 2010. 4. 27. · Estadística Empresarial II 23 La correlación negativa

Estadística Empresarial II 39

Obtendremos ahora las esperanzas de X2 y de Y2, para luego ver cuanto valen las respectivasvarianzas.

E(X2) = x 2x dx 12

x 12

2

x 0

x 14

x 0

x 1

=

=

=

=

∫ =

=

E(Y2) = ( ]y e dy - y e - 2y e dy 0 + 2 y e dy 2E(Y) 2 2

y 0

y2

y 0

y

y 0

y−

=

=∞−

=

=∞ −

=

=∞−

=

=∞

∫ ∫ ∫= − = = =y yy

y y y0

luego

Var(X) = E(X2) - E2(X) = 12

23

1

18

2

=

Var(Y) = E(Y2) - E2(Y) = 2 - 12 = 1

con lo cual, aplicando las propiedades de la varianza tendremos:

Var(5X + 2Y) = 52Var(X) + 22Var(Y) = 25 1

18 4 x1 = 5.38.+

)

por ser X e Y independientes

15º) Sea X una v. a. dada por la demanda diaria, en kilogramos, de un determinado producto,que sigue una distribución normal con media 300 y desviación típica 20.Si se vende cada kilogramo a 50 pesetas y sabemos que los costes fijos diarios son 2 000pesetas y los variables 10 pesetas por kilogramo vendido:

a) Deducir la distribución del beneficio diario.b) Obtener la probabilidad de que en un mes (30 días) el beneficio sea mayor que 304 000pesetas.c) ¿Como cambiaría el problema si no conociésemos la distribución de la variable X, aunquesí su media y su varianza?

SOLUCIÓN

X = “Demanda diaria” (en kg.) → N(300,20)

a) B = “Beneficios” = Ingresos - Costes = 50X - (10X + 2000) = 40X - 2 000

E(B) = E(40X - 2 000) = 40E(X) - 2000 = 40x300 - 2 000 = 10 000

Var(B) = Var(40X - 2 000) = 402Var(X) = 402 x 202 = 640 000 ⇒ σB = 800

luego

B → N(10 000,800)

Page 20: ESTADÍSTICA EMPRESARIAL II PROBLEMAS DE EXAMEN …media0.webgarden.es/files/media0:4bd76ebb98146.pdf.upl... · 2010. 4. 27. · Estadística Empresarial II 23 La correlación negativa

40 Problemas de examen resueltos

b) Llamando B1, B2, …, B30, al beneficio de cada uno de los días de un mes, el beneficio mensual(T) será:

T = B1 + … + B30

E(T) = E(B1 + … + B30) = E(B1) + … + E(B30) = 30x10 000 = 300 000

Var(T) = Var(B1 + … + B30) = Var(B1) + … + Var(B30) = 30x640 000 = 19 200 000

Suponiendo que los beneficios diarios son independientes

σT = 4381.78046

p(T > 304000) = p Z 304 000 300 000

4381.78046 1 (0.91) 1 0.8186 0.1814.>

= − = − =Φ

Tipificando

c) Si no supiéramos que distribución sigue X, como si conocemos su media y su varianza,podríamos aplicar el Teorema Central del Límite, ya que la variable T = Beneficio mensual es unasuma de variable aleatorias independientes idénticamente distribuidas con media y varianza finita.La única diferencia que habría sería que como consecuencia de aplicar el Teorema Central delLímite la probabilidad calculada no sería exacta, si no que sería aproximada.

16º) Sea X la v. a. proporción de tornillos defectuosos en una máquina, con función dedensidad:

f (x) 3 (1 x) 0 x 1

0 en el restoX

2

=− < <

a) ¿Cuál es la distribución de la variable X?b) ¿Cuál es la proporción esperada de tornillos defectuosos?c) Hallar la probabilidad de que la proporción de tornillos defectuosos sea menor que 0.1.d) Trabajando con 20 máquinas similares a la anterior, ¿cuál es la probabilidad de que, almenos en dos de ellas, la proporción de tornillos defectuosos sea menor que 0.1?

SOLUCIÓN

a) X → β(1,3)

f (x)

1(p,q)

x (1 x) 0 x 1

0 resto

3 (1 - x) 0 x 10 restoX

p 1 q 12

=− < <

=

< <

− −

β

ya que

Page 21: ESTADÍSTICA EMPRESARIAL II PROBLEMAS DE EXAMEN …media0.webgarden.es/files/media0:4bd76ebb98146.pdf.upl... · 2010. 4. 27. · Estadística Empresarial II 23 La correlación negativa

Estadística Empresarial II 41

1

(1,3)

(1 3)(1) (3)

3!

0! 2! 3

β=

+= =

ΓΓ Γ

b) Como X → β(1,3)

E(X) = p

p q

11 3

0.25+

=+

=

Si no conociéramos la distribución de X la esperanza la podríamos calcular:

E(X) = x 3(1- x) dx 3x 2x

3x4

312

2

x 0

x 1 2 3 4

x 0

x 1

=

=

=

=

∫ = − +

= − +

=

223

14

0 25.

c) p(X < 0.1) = ( )( ] 3(1 x) dx 1 x 0.729 1 0.271. 2

x 0

x 0.13

x 0

x 0.1− = − − = − + =

=

=

=

=

d) Definimos el suceso E = Éxito = La proporción de tornillos defectuosos es menor que 0.1, yconsideramos la variable aleatoria

Y = “Número de máquinas en las que la proporción de tornillos defectuosos esmenor que 0.1 de las veinte dadas”

Como la variable Y cuenta el número de éxitos en 20 ensayos de Bernoulli, sigue una distribuciónbinomial de parámetros n = 20 y p = p(E) = p(X < 0.1) = 0.271.

Y → B(20,0.271)

La probabilidad que nos piden es:

p(Y ≥ 2) = 1 - p(Y < 2) = 1 - [p(X = 0) + p(X = 1)] =

= 1 200 0.271 0.729

201 0.271 0.729 0 20 1 19−

= 0.9848.

17º) En una biblioteca pública han realizado un estudio sobre X = “Nº de libros prestados auna persona el misma día” e Y = “Nº de vídeos prestados a una persona el mismo día” con lossiguientes resultados:

X \ Y 0 1 20 0 0.1 01 0.1 0.1 0.22 0.2 0.1 03 0.2 0 0

a) Calcular e interpretar el coeficiente de correlación.

Page 22: ESTADÍSTICA EMPRESARIAL II PROBLEMAS DE EXAMEN …media0.webgarden.es/files/media0:4bd76ebb98146.pdf.upl... · 2010. 4. 27. · Estadística Empresarial II 23 La correlación negativa

42 Problemas de examen resueltos

b) Obtener la distribución del número de libros prestados para aquellos usuarios de labiblioteca que no se llevan en préstamo ningún vídeo, ¿cuál es su esperanza?c) Calcular la probabilidad de que el número de objetos prestados a un usuario el mismo díasea mayor que dos.d) Si en el préstamo de cada libro se tarda 20 segundos y en el de cada vídeo se tarda 30segundos, ¿cuál es el tiempo medio que se tarda en atender a una persona? ¿Y con quévarianza?

SOLUCIÓN

a) Tenemos las variables X = “Nº de libros prestados a una persona el misma día” e Y = “Nº devídeos prestados a una persona el mismo día”. Para calcular el coeficiente de correlación nos hacenfalta los siguientes cálculos:

X \ Y 0 1 2 p(X = x)0 0 0.1 0 0.11 0.1 0.1 0.2 0.42 0.2 0.1 0 0.33 0.2 0 0 0.2

p(Y = y) 0.5 0.3 0.2 1.0

E(X) = (0x0.1) + (1x0.4) + (2x0.3) + (3x0.2) = 1.6

E(X2) = (02x0.1) + (12x0.4) + (22x0.3) + (32x0.2) = 3.4

Var(X) = E(X2) - E2(X) = 3.4 - 1.62 = 0.84

E(Y) = (0x0.5) + (1x0.3) + (2x0.2) = 0.7

E(Y2) = (02x0.5) + (12x0.3) + (22x0.2) = 1.1

Var(Y) = E(Y2) - E2(Y) = 1.1 - 0.72 = 0.61

E(XY) = (0x0x0) + (0x1x0.1) + (0x0x0) + (1x0x0.1) + (1x1x0.1) + (1x2x0.2) ++ (2x0x0.2) + (2x1x0.1) (2x2x0) + (3x0x0.2) + (3x1x0) + (3x2x0) = 0.7

Cov(X,Y) = E(XY) - E(X)E(Y) = 0.7 - (1.6x0.7) = -0.42

Cov(X,Y)

Var(X) Var(Y)

0.420.84 x 0.61

0.586738694X,Yρ = =−

= −

Como el coeficiente de correlación es negativo, quiere decir que cuanto mayor es el número delibros que se lleva una persona en préstamo, menor es el número de vídeos que se lleva, y al revés, amayor cantidad de vídeos llevados en préstamo, menor es el número de libros que se lleva.

b) Empezaremos calculando la ley de probabilidad de X/Y=0.

Page 23: ESTADÍSTICA EMPRESARIAL II PROBLEMAS DE EXAMEN …media0.webgarden.es/files/media0:4bd76ebb98146.pdf.upl... · 2010. 4. 27. · Estadística Empresarial II 23 La correlación negativa

Estadística Empresarial II 43

f (0 / Y 0) f (0,0)

f (0)

00.5

0 X/Y 0X,Y

Y= = = = =

f (1 / Y 0) f (1,0)

f (0)

0.10.5

0.2 X/Y 0X,Y

Y= = = = =

f (2 / Y 0) f (2,0)

f (0)

0.20.5

0.4 X/Y 0X,Y

Y= = = = =

f (3 / Y 0) f (3,0)

f (0)

0.20.5

0.4 X/Y 0X,Y

Y= = = = =

luego

X/Y=0 p(X/Y=0)1 0.22 0.43 0.4

con lo cual su esperanza es:

E(X/Y=0) = (1x0.2) + (2x0.4) + (3x0.4) = 2.2

c) p(X + Y > 2) = p(X = 1, Y = 2) + p(X = 2, Y = 1) + p(X = 3, Y = 0) = 0.2 + 0.1 + 0.2 = 0.5

d) Definimos la variable T = 20X + 30Y. Tenemos que calcular su esperanza y su varianza, paraello aplicaremos las propiedades de ambas.

E(T) = E(20X + 30Y) = 20E(X) + 30E(Y) = (20x1.6) + (30x0.7) = 53 segundos

Var(T) = Var(20X + 30Y) = 202Var(X) + 302Var(Y) + 2x20x30Cov(X,Y) =

= (400x0.84) + (900x0.61) + (1200x(-0.42)) = 381

18º) Sea (X,Y) una variable aleatoria bidimensional con función de densidad conjunta:

f (x, y) 4 x (1 x) 0 x 1 3 y 8

0 en el restoX,Y

3

=− < < < <

a) Obtener las densidades marginales de X y de Y. Indicar que distribución siguen.b) ¿Son X e Y independientes?c) Calcular la varianza de 3X - Y + 2.d) Comparar la p(2 < Y < 4) y la ( )p 2 Y 4

X 1 / 2 < << comentando el resultado.

Page 24: ESTADÍSTICA EMPRESARIAL II PROBLEMAS DE EXAMEN …media0.webgarden.es/files/media0:4bd76ebb98146.pdf.upl... · 2010. 4. 27. · Estadística Empresarial II 23 La correlación negativa

44 Problemas de examen resueltos

e) Hallar la función de densidad de Z = e-Y y calcular su esperanza.

SOLUCIÓN

a) Función de densidad marginal de X

( ] f (x) 4 x (1 x) dy 4 x (1 x) y 20 x (1 x) 0 x 1 X3

y 3

y 83

y 3

y 8 3= − = − = − < <=

=

=

=

X → β(4,2)

f (x)

1(p,q)

x (1 x) 0 x 1

0 resto

20x (1 - x) 0 x 10 restoX

p 1 q 13

=− < <

=

< <

− −

β

ya que

1

(4,2)

(4 2)(4) (2)

5!

3! 1! 20

β=

+= =

ΓΓ Γ

Función de densidad marginal de Y

f (y) 4 x (1 x) dx x x 15

3 y 8 Y3

x 0

x 14 5

x=0

x=1

= − = −

= < <=

=

∫45

Y → U(3,8)

f (y)

1b a

a y b

0 resto

15

3 y 8

0 restoY = −

< <=

< <

b) Dos variables son independientes si se verifica que:

fX,Y(x,y) = fX(x)fY(y) ∀ x,y ∈ R

en este caso tenemos:

fX,Y(x,y) = 4 x (1 x) 0< x , 3 y 8

0 en el resto

3 − < < <

1 = fX(x) fY(y)

luego las variables X e Y son independientes.

Page 25: ESTADÍSTICA EMPRESARIAL II PROBLEMAS DE EXAMEN …media0.webgarden.es/files/media0:4bd76ebb98146.pdf.upl... · 2010. 4. 27. · Estadística Empresarial II 23 La correlación negativa

Estadística Empresarial II 45

c) Var(3X - Y + 2) = 32Var(X) + (-1)2Var(Y) = 9 x 263

2512

2.369047619

+ =

Por X e Y independientes

Var(X) = pq

(p q 1)(p q)

4 x 2(4 2 1)(4 2)

2632 2+ + +

=+ + +

=

Var(Y) = (b a)

12

(8 3)12

2512

2 2−

=−

=

d) ( ) ( ) ( )[ ]( )

( ) ( )( )p 2 Y 4

X 1/ 2 p 2 Y 4 X 1 2

p X 1 2

p 2 Y 4 p X 1 2p X 1 2

< << =

< < ∩ ≤

≤=

< < ≤

≤=

Por X e Y independientes

= p(2 < Y < 4) = 15 dy

15 y

15y 3

y 4

y 3

y 4

=

=

=

=

∫ =

=

Ambas probabilidades coinciden por ser X e Y independientes.

e) Sea Z = e-Y

FZ(z) = p(Z ≤ z) = p(e-Y ≤ z) = p(-Y ≤ Ln z) = p(Y ≥ -Ln z) = 1 - FY(-Ln z)

fZ(z) = f ( Ln z) 1

z

15 z

Y− −−

=

3 < -Ln z < 8 ⇒ -3 > Ln z > -8 ⇒ e-8 < z < e-3

luego

f (z)

15 z

e z e

0 en el restoZ

8 3

=< <

− −

E(Z) = ( ) z 1

5 z dz

15

z 15

e e . z e

z e

z e

z e3 8

8

3

8

3

=

=

=

=− −

∫ =

= −

Page 26: ESTADÍSTICA EMPRESARIAL II PROBLEMAS DE EXAMEN …media0.webgarden.es/files/media0:4bd76ebb98146.pdf.upl... · 2010. 4. 27. · Estadística Empresarial II 23 La correlación negativa

46 Problemas de examen resueltos

19º) Sea X la v. a. duración de un componente electrónico de una máquina, medida en meses,con distribución exponencial negativa de parámetro a = 0.1.

a) Calcular p(µ - 2σ < X < µ + 2σ).b) Si tenemos otros cinco componentes de repuesto y cambiamos la pieza en el mismomomento que deja de funcionar, ¿cuál será la probabilidad de que podamos mantener lamáquina en funcionamiento 60 meses sin pedir repuestos? (Indicar los cálculos. No se pide elvalor numérico concreto).c) Si consideramos 20 componentes. ¿Cuál es la probabilidad de que al menos para dos deellos su duración sea inferior a 30 meses?

SOLUCIÓN

Consideramos la variable aleatoria X = “Duración de un componente electrónico de una máquina enmeses” → ε(0.1)

a) E(X) = 1a

1

0.1 10 = = Var(X) =

1a

1

0.1 100 2 2= =

luego

µ = E(X) = 10 y σX = Var(X) = 10

con lo que

p(µ - 2σ < X < µ + 2σ) = p(10 - 2x10 < X < 10 + 2x10) = p(-10 < X < 30) =

= ( ] 0.1 e dx e 1 e 0.950212931. 0.1 x

x 0

x 300.1x

x 0

x 30 3−

=

=−

=

= −∫ = − = − =

b) Definimos la variable T = X1 + … + X6 → γ(0.1,6) pues la suma de exponenciales negativas esuna gamma.

p(T 60) 0.1

(6) t e dt

65 0.1t

t 60

t

≥ = −

=

=∞

∫ Γ

c) Sea Y = “Número de componentes cuya duración es inferior a 30 meses de los 20 considerados”.Llamando éxito al suceso

E = “Duración de un componente electrónico de una máquina inferior a 30 meses”

p(E) = p(X < 30) = 0.950212931

la variable Y cuenta en número de éxitos en 20 ensayos de Bernoulli, luego

Y → B(20, 0.950212931)

Page 27: ESTADÍSTICA EMPRESARIAL II PROBLEMAS DE EXAMEN …media0.webgarden.es/files/media0:4bd76ebb98146.pdf.upl... · 2010. 4. 27. · Estadística Empresarial II 23 La correlación negativa

Estadística Empresarial II 47

y la probabilidad que nos piden es:

p(Y ≥ 2) = 1 - p(Y < 2) = 1 -200

0 9502 0 0498201

0 9502 0 04980 20 1 19

+

. . . . =

= 1 - 3.351211505 10-24 ≈ 1

20º) En una oficina de una entidad financiera saben que los ingresos que se efectúan son portérmino medio de 100 mil pesetas con una desviación típica de 20 mil pesetas y que losreintegros medios son de 40 mil pesetas con una desviación de 10 mil. Al inicio del día unfurgón lleva la cantidad de dinero solicitada el día anterior.

a) El día 2 de febrero antes de abrir al público, había 4 millones de pesetas en caja. Se efectúan 36ingresos y 217 reintegros. La cantidad suministrada por el furgón fue de 6 millones, ¿cuál es laprobabilidad de que al hacer el arqueo al final del día haya en caja a lo sumo 5 millones?b) Una vez realizado el arqueo se observa que en caja hay 4 millones y medio para el díasiguiente. Si la previsión de ingresos y de reintegros para el día 3 de febrero coincide con losque se efectuaron el día 2, ¿qué cantidad de dinero habrá que pedir que lleve el furgón paraque con una probabilidad del 97.5% al final del día quede en caja al menos la misma cantidadque al principio del día?

SOLUCIÓN

Definimos las siguientes variables aleatorias:

X = “Ingresos en una entidad financiera” (en miles de pesetas)Y = “Reintegros en una entidad financiera” (en miles de pesetas)

sabemos que

E(X) = 100 σX = 20 E(Y) = 40 σY = 10

a) Sea W = X Y ii 1

36

ii 1

217

= =∑ ∑−

E(W) = E X Y = E(X ) E(Y ) 36 x 100 217 x 40 -5 080ii 1

36

ii 1

217

ii 1

36

ii=1

217

= = =∑ ∑ ∑ ∑−

− = − =

Var(W) = Var X Y = Var(X ) Var(Y ) 36 x 20 217 x 10 36 100ii 1

36

ii 1

217

ii 1

36

ii=1

2172 2

= = =∑ ∑ ∑ ∑−

+ = + =

σW = 190

La probabilidad pedida es:

Page 28: ESTADÍSTICA EMPRESARIAL II PROBLEMAS DE EXAMEN …media0.webgarden.es/files/media0:4bd76ebb98146.pdf.upl... · 2010. 4. 27. · Estadística Empresarial II 23 La correlación negativa

48 Problemas de examen resueltos

p X Y 4 000 6 000 5 000 p(W 5 000) ii 1

36

ii 1

217

= =∑ ∑− + + ≤

= ≤ − =

pW 5 080

190

5 000 5 080190

80

190 (0.42) 0.6628 =

+≤

+

= =Φ Φ

Tipificando Aplicando el Teorema Central del Límite

b) Hay que encontrar un valor k tal que

p(W + 4 500 + k ≥ 4 500) = p(W ≥ -k) = 0.975

luego

0.975 = p(W - k) = pW 5 080

190

- k 5 080190

1 -- k 5 080

190 ≥

+≥

+

+

Φ ⇒

Tipificando Aplicando el Teorema Central del Límite

⇒ k 5 080

190 0.025 Φ

− +

= ⇒

k 5 080190

-1.96 − +

= ⇒

⇒ k = 5 452.4 miles de pesetas.

21º) Sea X una variable aleatoria con la siguiente ley de probabilidad:

X 0 1 2 3p(X=x) 1/8 1/4 1/2 1/8

a) ¿Cuánto vale la varianza de 2X + 5?b) Calcular la p(X < 3/X ≥ 1).c) Podemos suponer que X sigue una distribución binomial, ¿por qué?

SOLUCIÓN

a) E(X) 0 x 18

1 x 14

2 x 12

3 x 18

1.625=

+

+

+

=

E(X ) 0 x 18

1 x 14

2 x 12

3 x 18

3.3752 2 2 2 2=

+

+

+

=

Page 29: ESTADÍSTICA EMPRESARIAL II PROBLEMAS DE EXAMEN …media0.webgarden.es/files/media0:4bd76ebb98146.pdf.upl... · 2010. 4. 27. · Estadística Empresarial II 23 La correlación negativa

Estadística Empresarial II 49

Var(X) = E(X2) - E2(X) = 3.375 - 1.6252 = 0.734375

Var(2X + 5) = 22Var(X) = 4 x 0.734375 = 2.9375

b) ( ) ( ) ( )[ ]( )

( ) ( )( ) ( )p X 3

X 1 p X 3 X 1

p X 1

p X 1 p X 2p X 1 p X 2 p( X 3)

<≥ =

< ∩ ≥≥

== + =

= + = + ==

14

12

14

12

18

67

=+

+ +=

c) La función de densidad de una variable aleatoria que sigue una distribución B(n,p) es:

( )p X k nk p (1 p) k 0, 1, ..., n k n k= = − =−

donde n = número de experimentos realizados p = Probabilidad de éxito

En este caso n = 3, el valor de p le obtendremos despejando, por ejemplo, en la probabilidad de Xvalga cero.

( ) p X 0 18

30 p (1 p) (1 p)

18

0 3 3= = = − ⇒ − = ⇒

⇒ − = ⇒ = 1 p 12

p 12

Luego si X sigue una distribución binomial, esta es una B(3,1/2). Comprobemos si para el resto devalores coinciden las probabilidades.

31

12

12

38

14

p(X 1) 1 2

= ≠ = = ⇒ X no sigue una distribución binomial.

22º) Dada la variable aleatoria X con función de densidad:

fX(x) = 6x(1-x) 0 < x < 1

Definimos la variable Y = 5 - 3X. Obtener su función de densidad y deducir qué distribuciónsigue.

SOLUCIÓN

Page 30: ESTADÍSTICA EMPRESARIAL II PROBLEMAS DE EXAMEN …media0.webgarden.es/files/media0:4bd76ebb98146.pdf.upl... · 2010. 4. 27. · Estadística Empresarial II 23 La correlación negativa

50 Problemas de examen resueltos

Empezaremos calculando la relación que hay entre las funciones de distribución para luego ver querelación existe entre las funciones de densidad.

F (y) p(Y y) p(5 3X y) p( 3X y 5) p X y 5

3 1 - F

5 y3Y X= ≤ = − ≤ = − ≤ − = ≥

−−

=

luego

( ) ( ) ( ) f y f5 y

3 13

= 6 5 y

3 1

5 y3

13

29

y 2 5 y Y X=−

= − −

0 5 y

3 1 0 5 y 3 5 y 2 2 y 5<

−< ⇒ < − < ⇒ − < − < − ⇒ < <

con lo que

( )( ) ( )

f y

29

y 2 5 y 2 y 5

0 en el restoY =

− − < <

Y → β(2,2,2,5)

f (y)

1(p,q)

1(b -a)

(y -a) (b y) a y b

0 resto

29

(y -2) (5 - y) 2 y 5

0 restoY

p+q-1p 1 q 1

=− < <

=

< <

− −

β

ya que

1

(2,2)

1(5-2)

(2 2)

(2) (2)13

3!

1! 1!

127

29

2+2-1 3β=

+= =

ΓΓ Γ

23º) Sea X la v. a. número de accidentes laborales diarios en un determinado grupo deempresas.

a) Sabiendo que p(X = 0) = 0.1353352, calcular la esperanza y la varianza de dicha variable yla probabilidad de que un día determinado haya más de 3 accidentes.b) Probabilidad de que en una semana (5 días) ocurran menos de 2 accidentes.

SOLUCIÓN

Sea X = número de accidentes laborales diarios en un determinado grupo de empresas → P(λ)

a) p(X 0) 0.1353352 e 0!

2 0

= = = ⇒ =−λ λ λ

Page 31: ESTADÍSTICA EMPRESARIAL II PROBLEMAS DE EXAMEN …media0.webgarden.es/files/media0:4bd76ebb98146.pdf.upl... · 2010. 4. 27. · Estadística Empresarial II 23 La correlación negativa

Estadística Empresarial II 51

Luego

E(X) = λ = 2 y Var(X) = λ = 2.

p(X > 3) = 1 - p(X ≤ 3) = 1 - 0.8571 = 0.1429.

b) Sea Y = número de accidentes laborales semanales = X ii 1

5

=∑ .

Como la suma de variables de Poisson es otra variable de Poisson, y el parámetro es la suma de losparámetros, tenemos que

Y → P(10)

p(Y < 2) = p(Y ≤ 1) = 0.0005

24º) Sea X e Y dos variables aleatorias con la siguiente función de densidad conjunta:

f (x, y) Ky e x 0 y 0

0 restoX,Y

(2x y)

=≥ ≥

− +

a) Comprobar que la constante K = 2.b) Calcular las funciones de densidad marginal de la X y de la Y. ¿Qué distribución siguen?c) ¿Son independientes las variables X e Y?d) Calcular la esperanza y la varianza la variable Z = 2X-3Y+5.e) Plantear la P(X+Y< 1).

SOLUCIÓN

a) Para comprobar que la constante vale 2, usaremos el hecho de que la integral en todo el dominiode definición de la función de densidad es 1.

1 K y e dx dy K y e 1

2 e dy

K2

y e dy (2x y)

x 0

x

y 0

yy 2x

x 0

x

y 0

yy

y 0

y

=

=

= =− +

=

=∞

=

=∞− −

=

=∞

=

=∞−

=

=∞

∫∫ ∫ ∫

K2

ye dy K2

K 2 y

y 0

y

= = ⇒ =−

=

=∞

pues la integral en todo el dominio dedefinición de una γ(1,2) vale 1

1

b) Marginal de X

Page 32: ESTADÍSTICA EMPRESARIAL II PROBLEMAS DE EXAMEN …media0.webgarden.es/files/media0:4bd76ebb98146.pdf.upl... · 2010. 4. 27. · Estadística Empresarial II 23 La correlación negativa

52 Problemas de examen resueltos

f (x) 2y e dy 2 e y e dy 2 e x 0 X(2x y) 2x y

y 0

y2x= = = ≥− + − −

=

=∞−

=

=∞

∫∫y

y

0

X → ε(2), pues

f (x) a e x 0

0 resto

2 e x 00 restoX

-ax -2x

=>

=

>

Marginal de Y

f (y) 2y e dx y e 2 e dx y e y 0 Y(2x y) y

x 0

x

= = = ≥− + − −

=

=∞−

=

=∞

∫∫ 2

0

x y

x

x

Y → γ(1,2)

f (y)

a(p)

y e y 0

0 resto

y e y 00 restoY

pp-1 -ay

-y

=>

=

>

Γ

pues

a(p)

1(2)

11!

1 p 2

Γ Γ= = =

c) Dos variables son independientes si se verifica que:

fX,Y(x,y) = fX(x)fY(y) ∀ x,y ∈ R

en este caso tenemos:

fX,Y(x,y) =2y e x 0 y 0

0 resto

(2x y)− + ≥ ≥

= fX(x) fY(y)

luego las variables X e Y son independientes.

d) Calcularemos primero las esperanzas y las varianzas de X e Y.

E(X) 1a

12

= = = 05. E(Y) pa

21

2= = =

Page 33: ESTADÍSTICA EMPRESARIAL II PROBLEMAS DE EXAMEN …media0.webgarden.es/files/media0:4bd76ebb98146.pdf.upl... · 2010. 4. 27. · Estadística Empresarial II 23 La correlación negativa

Estadística Empresarial II 53

Var(X) 1

a

12

2 2= = = 0 25. Var(Y) p

a

21

22 2= = =

E(2x - 3Y + 5) = 2E(X) - 3E(Y) +5 = (2x0.5) - (3x2) + 5 = 0

Var(2X - 3Y + 5) = 22Var(X) + (-3)2Var(Y) = (4x0.25) + (9x2) = 19

Por ser X e Y independientes

e)

p(X Y 1) 2ye dy dx (2x y)

y 0

y 1 x

x 0

x 1

+ < =

− +

=

= −

=

=

∫∫

25º) Un producto se presenta a la venta en un paquete que contiene tres bolsas de producto ensu interior. Sabiendo que:El peso del envase exterior es una v. a. con distribución N(50, 6) medida en gramosEl peso de cada bolsa de producto es una v. a. con distribución N(100, 6) medida en gramosa) Calcular la distribución de la v. a. peso total de un paquete.b) Si para su distribución los paquetes se envían en cajas de 100, calcular la probabilidad deque una caja pese más de 35.2 Kilogramos.

SOLUCIÓN

Definimos las siguientes variables:

X = Peso del envase exterior → N(50,6)Y = Peso de una bolsa de producto → N(100,6)

Las variables X e Y son independientes.

a) Sea Z = Peso total de un paquete = X + Y1 + Y2 + Y3 → N(µ,σ), pues la suma de variablesnormales es otra variable normal. Determinemos el valor de sus parámetro.

E(Z) = µ = E(X + Y1 + Y2 + Y3) = E(X) + E(Y1) + E(Y2) + E(Y3) = 50 + 100 + 100 + 100 = 350

Var(Z) = σ2 = Var(X + Y1 + Y2 + Y3) = Var(X) + Var(Y1) + Var(Y2) + Var(Y3) =

= 62 + 62 +62 +62 = 144

X

Y

X + Y = 1

1

1

Page 34: ESTADÍSTICA EMPRESARIAL II PROBLEMAS DE EXAMEN …media0.webgarden.es/files/media0:4bd76ebb98146.pdf.upl... · 2010. 4. 27. · Estadística Empresarial II 23 La correlación negativa

54 Problemas de examen resueltos

luego Z → N(350,12)

b) Sea W = Peso de una caja = Z N(n , n ) = N(100x350, 100 xii 1

100

=∑ → µ σ 12) = N(35 000,120)

p(W > 35 200) = 1 - p(W ≤ 35 200) = 1 pW 35 000

120

35 200 35 000120

−−

≤−

=

= 1 - Φ(1.67) = 1 - 0.9525 = 0.0475.

26º) Una empresa dedicada a la venta de ordenadores se plantea contratar a una persona parael montaje de los equipos en las siguientes condiciones: cobrará una cantidad fija de 75 milpesetas al mes más una cantidad variable por cada equipo que monte, que seguirá unadistribución uniforme de parámetros mil y dos mil quinientas pesetas, en función de loscomponentes que lleve.El precio de cada ordenador se verá incrementado por el concepto de montaje en unacantidad variable en función del número de componentes y que seguirá una distribuciónuniforme de parámetros dos mil y cuatro mil cuatrocientas pesetas.

a) Calcular la probabilidad de que la cantidad variable del salario del empleado por elmontaje de un equipo oscile entre novecientas y mil trescientas pesetas.b) Si se estima que el número de equipos vendidos mensualmente es 70, ¿cuál sería el salarioesperado? ¿Y con que varianza?c) ¿Cuál será el número mínimo de equipos que se deberán vender durante un mes, para quelos servicios de este empleado no superen a los ingresos obtenidos por la empresa en conceptode montaje, con una probabilidad del 97.5%?

SOLUCIÓN

a) Consideramos la variable X = Cantidad cobrada por el empleado por el montaje de un equipo (enmiles de pesetas) → U(1,2.5) según nos dice el enunciado del problema.

f (x)

1b -a

a x b

0 resto

12.5-1

1 x

0 resto

11.5

1 x

0 restoX =

< <=

< <

=< <

2 5 2 5. .

( ] p(0.9 X 1.3) f (x) dx 0 dx 1

1.5 dx 0

11.5

x 0.2 Xx 0.9

x 1.3

x 0.9

x 1

x 1

x 1.3

x 1x 1,3< < = = + = + =

=

=

=

=

=

=

==∫ ∫ ∫

b) S = Salario mensual del empleado = 75 Xii 1

70

+=∑

Page 35: ESTADÍSTICA EMPRESARIAL II PROBLEMAS DE EXAMEN …media0.webgarden.es/files/media0:4bd76ebb98146.pdf.upl... · 2010. 4. 27. · Estadística Empresarial II 23 La correlación negativa

Estadística Empresarial II 55

E(X) a b

2

1 2.52

1.75 =+

=+

=( ) ( )

Var(X) b a

12

2.5 112

0.1875 2 2

=−

=−

=

E(S) = E 75 X 75 E(X ) 75 (70 x 1.75) 197.5 ii 1

70

ii 1

70

+

= + = + =

= =∑ ∑ miles de pesetas.

Var(S) = Var 75 X Var(X ) 70 x 0.1875 13.125 ii 1

70

ii 1

70

+

= = =

= =∑ ∑

c) Sea Y = Incremento en el precio de un equipo en concepto de montaje (en miles de pesetas), quesegún el enunciado → U(2,4.4)

Definimos B = Beneficios de la empresa por el concepto de montaje.

B = Ingresos - Gastos = Y 75 X ii 1

n

ii 1

n

= =∑ ∑− −

El problema nos pide encontrar

n / p(B > 0) = 0.975

Como la distribución de B no la conocemos y es suma de variables aleatorias independientesidénticamente distribuidas con esperanza y varianza finitas, podemos aplicar el Teorema Central delLímite, es decir,

B E(B)Var(B)

N(0,1) L− →

E(B) = ( ) ( ) E Y 75 X E Y 75 E X ii 1

n

ii 1

n

ii 1

n

ii 1

n

= = = =∑ ∑ ∑ ∑− −

= − − =

=+

− −

2 4.42

n 75 (1.75 n) = 1.45 n - 75

Var(B) = ( ) ( ) Var Y 75 X Var Y + Var X ii 1

n

ii 1

n

ii 1

n

ii 1

n

= = = =∑ ∑ ∑ ∑− −

= =

=

+

(4.4 -2)2

n (0.1875 n) 2

= 0.6675 n

Page 36: ESTADÍSTICA EMPRESARIAL II PROBLEMAS DE EXAMEN …media0.webgarden.es/files/media0:4bd76ebb98146.pdf.upl... · 2010. 4. 27. · Estadística Empresarial II 23 La correlación negativa

56 Problemas de examen resueltos

0.975 = p(B 0) = pB 1.45n + 75

0.6675n

1.45n + 750.6675n

1 - 1.45n + 750.6675n

>−

>−

0 0Φ ⇒

Tipificando Aplicando el Teorema Central del Límite

⇒ 0-1.45n 75

0.6675n 0.025 Φ

+

= ⇒ 75 1.45 n

0.6675 n -1.96

−= ⇒

⇒ 1.45 n - 0.6675 n - 75 = 0 ⇒ n 7.4791918276.915738909=

⇒ n 55.938347.8274=

la solución n = 47.8274 la tenemos que desechar pues al sustituir su valor en la probabilidad departida no da 0.975, que es el valor pedido, sino que da una probabilidad de 0.025.

Por tanto el valor pedido, redondeando, es n = 56 equipos.

27º) Sean X e Y variables aleatorias independientes con funciones de densidad:

=−

resto el en00xxe

(x)fx

X

≤≤=

restoelen0

3y1y41

(y)fY

a) Calcular la función de densidad conjunta.b) Calcular la probabilidad de que la variable Y sea mayor que 2.c) Calcular la esperanza y varianza de la variable: T = 2X - 3Yd) Comprobar que se verifica: E(XY) = E(X) E(Y), indicando el motivo.e) Calcular p(X + Y < 3). (Dejar indicada la integral correspondiente).

SOLUCIÓN

a) Como las variables X e Y son independientes, la función de densidad conjunta es el producto delas marginales, luego

≤≤≥==

resto elen 0

3y10,xey x 41

(y)f(x)fy)(x,fx

YXYX,

Page 37: ESTADÍSTICA EMPRESARIAL II PROBLEMAS DE EXAMEN …media0.webgarden.es/files/media0:4bd76ebb98146.pdf.upl... · 2010. 4. 27. · Estadística Empresarial II 23 La correlación negativa

Estadística Empresarial II 57

b) p(Y > 2) = 85 y

81 dyy

41 3

2

23y

2y

=

=

=

=

=

=∫

y

y

c) Comprobemos que X → γ(1,2)

>

=>

Γ=resto0

0 x ex

resto0

0 x e x(p)a

(x)fx-

ax-1-pp

X

pues

11!1

(2)1

(p)a

2p

==Γ

luego

2 12

ap E(X) ===

2 12

ap Var(X) 22 ===

Calculemos ahora la esperanza y la varianza de Y.

613y

121dyy

41yE(Y)

3y

1y

33y

1y

=

==

=

=

=

=∫

5y161dyy

41y)E(Y

3y

1y

43y

1y

22 =

==

=

=

=

=∫

Var(Y) = E(X2) – E2(X) = 3611

6135

2

=

Con lo que

E(T) = E(2X – 3Y) = 2E(X) – 3E(Y) = ( )

613x32x2 = -2.5

Page 38: ESTADÍSTICA EMPRESARIAL II PROBLEMAS DE EXAMEN …media0.webgarden.es/files/media0:4bd76ebb98146.pdf.upl... · 2010. 4. 27. · Estadística Empresarial II 23 La correlación negativa

58 Problemas de examen resueltos

Var(T) = Var(2X – 3Y) = 22Var(X) + (-3)2Var(Y) = ( )

+

3611x92x4 = 10.75

por ser X e Y independientes

d) E(XY) == ∫ ∫∞=

=

=

=

x

0x

3y

1y

x dxdyeyx41yx = ∫

∞=

=

=

=

x

0x

3y

1y

3x2 dxy121ex = ∫

∞=

=

−x

0x

x2 dxex6

13 =

= ∫∞=

=

−x

0x

x2 dxex6

13 = 2!x6

13 = 3

13

E(X) E(Y) = 6

13 x 2 = 3

13 = E(XY)

La igualdad se verifica por que las variables X e Y son independientes.

e) p(X + Y < 3) = ∫ ∫=

=

−=

=

2x

0x

x3y

1y

x dxdyeyx41 =

= ∫ ∫=

=

−=

=

3y

1y

y3x

0

x dydxeyx41

x

28º) En una población formada por 2 400 trabajadoras, se desea estudiar las siguientesvariables aleatorias:

X = Nº de hijos ; Y = Nº de días de absentismo laboral al mes.

Si la ley de probabilidad conjunta de ambas variables es:

X \ Y 0 1 20 2/24 2/24 2/241 4/24 0 3/242 4/24 4/24 03 1/24 1/24 1/24

a) Calcular las leyes de probabilidad marginales de X e Y.

X

Y

X + Y = 3

3

3

Y = 3

Y = 1

2

1

Page 39: ESTADÍSTICA EMPRESARIAL II PROBLEMAS DE EXAMEN …media0.webgarden.es/files/media0:4bd76ebb98146.pdf.upl... · 2010. 4. 27. · Estadística Empresarial II 23 La correlación negativa

Estadística Empresarial II 59

b) ¿Cuantas empleadas tienen menos de 2 hijos?

c) Calcular:

≥≥

2X1Yp .

d) Calcular:

= 3XYE .

SOLUCIÓN

a) Para obtener las leyes de probabilidad marginales sumaremos por filas y por columnas la ley deprobabilidad conjunta

X \ Y 0 1 2 p(X = X)0 2/24 2/24 2/24 6/241 4/24 0 3/24 7/242 4/24 4/24 0 8/243 1/24 1/24 1/24 3/24

p(Y = y) 11/24 7/24 6/24 1

luego la ley de probabilidad marginal de X es

X 0 1 2 3p(X = x) 6/24 7/24 8/24 3/24

y la ley de probabilidad marginal de Y es

Y 0 1 2p(Y = y) 11/24 7/24 6/24

b) Calcularemos primero la probabilidad de que una mujer tenga menos de dos hijos y a esaprobabilidad le multiplicaremos por el número total de mujeres para obtener el valor pedido.

p(X < 2) = p(X = 0) + p(X = 1) = 2413

247

246

=+

con lo que

N p(X < 2) = 2 400 2413x = 1 300 mujeres tienen menos de 2 hijos.

c) ( )( ) =

≥≥≥

=

≥≥

2Xp2X1,Yp

2X1Yp

( ) ( ) ( ) ( )=

=+===+==+==+==

=3)p(X2)p(X

2Y3,Xp1Y3,Xp2Y2,Xp1Y2,Xp

Page 40: ESTADÍSTICA EMPRESARIAL II PROBLEMAS DE EXAMEN …media0.webgarden.es/files/media0:4bd76ebb98146.pdf.upl... · 2010. 4. 27. · Estadística Empresarial II 23 La correlación negativa

60 Problemas de examen resueltos

116

243

248

241

2410

244

=+

+++=

d) Empezaremos obteniendo la ley de probabilidad de Y condicionada por X = 3 y luegocalcularemos la esperanza.

La ley de probabilidad de Y condicionada por X = 3 se obtiene

3)p(X3)Xy,p(Y

3XyYp

===

=

==

por ejemplo para Y = 0 sería:

( )31

243

241

3)p(X3)X0,p(Y

3X0Yp ==

===

===

luego la ley de probabilidad de Y condicionada por X = 3 es:

Y/X = 3 0 1 2p(Y = y/X = 3) 1/3 1/3 1/3

con lo que la esperanza pedida es:

( ) 131x2

31x1

31x03X

YE =

+

+

==

29º) Se sabe que la proporción de grasa en cada lata de paté es una variable aleatoria X quetiene como función de densidad:

≤≤−

=resto el en0

1x0x)(1x12(x)f

2

X

a) ¿Qué distribución sigue la variable X?b) Si se analizan 10 latas, ¿cuál es la probabilidad de que al menos dos latas tengan uncontenido en grasa menor que el 30 por ciento?c) Si ahora se van analizando las latas una a una, ¿cuál es la probabilidad de que la primeralata que contiene menos del 30 por ciento en grasa sea la quinta?

SOLUCIÓN

Sea X = Proporción de grasa en cada lata de paté

Page 41: ESTADÍSTICA EMPRESARIAL II PROBLEMAS DE EXAMEN …media0.webgarden.es/files/media0:4bd76ebb98146.pdf.upl... · 2010. 4. 27. · Estadística Empresarial II 23 La correlación negativa

Estadística Empresarial II 61

a) X → β(2,3)

<<

=<<−

β=

−−

resto01x0 x)- (1x12

resto0

1x0x)(1xq)(p,

1

(x)f2

1q1p

X

ya que

212!1!

4!(3) (2))3(2

(2,3)1

==ΓΓ+Γ

b) Consideramos el suceso E = El contenido de grasa de una lata de paté es menor que el 30 %.Calculemos la probabilidad de este suceso.

p(E) = p(X < 0.3) = =+−=− ∫∫=

=

=

=

0.3x

0x

320.3x

0x

2 dx)x12x24x(12dxx)(1x12

( ] 0.3483x3x8x6 0.3x0x

432 =+−==

=

Definimos ahora la variable Y = Número de latas de las 10 analizadas con menos de un 30 % degrasa.

Llamando éxito al suceso E anterior, la variable aleatoria Y cuenta éxitos en 10 ensayos, por lotanto, sigue una distribución binomial.

Y → B(10, 0.3483)

La probabilidad que nos piden es:

p(Y ≥ 2) = 1 – p(Y < 2) =

+

− 91100 6517.03483.0

110

6517.03483.00

101 = 0.91232546

c) Definimos ahora la variable Z = Número de latas con más del 30 % de grasa antes de la primeracon menos del 30 % de grasa.

Si se seguimos llamando éxito al mismo suceso de antes, la variable Z cuenta los fracasos antes delprimer éxito, luego sigue una distribución geométrica.

Z → G(0.3483)

y la probabilidad pedida es:

p(Z = 4) = 0.65174 x 0.3483 = 0.062826715

Page 42: ESTADÍSTICA EMPRESARIAL II PROBLEMAS DE EXAMEN …media0.webgarden.es/files/media0:4bd76ebb98146.pdf.upl... · 2010. 4. 27. · Estadística Empresarial II 23 La correlación negativa

62 Problemas de examen resueltos

30º) El trayecto de una línea de autobuses urbana se realiza por término medio en 48 minutos,con una desviación de 4 minutos. Durante un día se realiza el recorrido 64 veces.

a) Calcular la probabilidad de que la duración total de los sesenta y cuatro trayectos sea mayor que3 104 minutos.b) Calcular la probabilidad de que la duración media de esos sesenta y cuatro recorridos seamenor que 47 minutos.c) Sabiendo además, que el número de viajeros diario de esa línea es de 3 200, que el preciomedio del billete es de 70 pesetas con una desviación de 10, que el litro de combustible cuesta96 pesetas, que el consumo en litros de combustible es la vigésimo cuarta parte de la duracióndel trayecto, y que cada trayecto tiene unos costes fijos de 3 300 pesetas, calcular laprobabilidad de tener beneficio al final de un día.

SOLUCIÓN

Sea la variable aleatoria X = Duración de un trayecto de una línea urbana. Sabemos que

E(X) = 48 σX = 4 número de trayecto = 64

a) Consideramos la variable aleatoria Y = Duración total de los sesenta y cuatro trayectos = ∑=

64

1iiX

( ) ∑∑∑===

==

64

1i

64

1ii

64

1ii 48XEXE = 64 x 48 = 3 072

( ) ∑∑∑===

==

64

1i

264

1ii

64

1ii 4XVarXVar = 64 x 16 = 1 024

∑=

64

1iiX

σ = 32

por ser Xi independientes

La probabilidad pedida es:

>∑

=

1043Xp64

1ii

De la variable aleatoria duración total no conocemos su distribución, pero si sabemos que es sumade variables aleatorias independientes idénticamente distribuidas con esperanza y varianza finitas,con lo cual podemos aplicar el Teorema Central del Límite, y aproximar el valor de dichaprobabilidad.

)1(132

0723104332

0723Xp1043Xp

64

1ii64

1ii Φ−≈

>−

=

>

∑∑ =

=

= 1 – 0.8413 = 0.1587

tipificando Aplicando el Teorema Central del Límite

Page 43: ESTADÍSTICA EMPRESARIAL II PROBLEMAS DE EXAMEN …media0.webgarden.es/files/media0:4bd76ebb98146.pdf.upl... · 2010. 4. 27. · Estadística Empresarial II 23 La correlación negativa

Estadística Empresarial II 63

b) Definimos X = Duración media de los sesenta y cuatro trayectos = 64

X64

1ii∑

= .

Estamos en las mismas condiciones del apartado anterior y volveremos a aplicar el Teorema Centraldel Límite. Calcularemos primero la esperanza y la varianza de la variable X .

640723

64

XE

64

XE

64

1ii

64

1ii

=

=

∑∑== = 48

22

64

1ii

64

1ii

640241

64

XVar

64

XVar =

=

∑∑== = 0.25 Xσ = 0.5

( ) )2(0.5

48470.5

48Xp47Xp −Φ≈

−<

−=< = 0.0228

tipificando Aplicando el Teorema Central del Límite

c) Disponemos de la siguiente información:

Número de viajeros de la línea = 3 200

P = Precio del billete E(P) = 70 σP = 10

Precio del litro de combustible = 96 pesetas

C = Consumo de combustible = 24X

Costes fijos de cada trayecto = 3 300

Definimos la variable

B = Beneficios = Ingresos – Gastos = ∑∑==

+−

64

1i

i2003

1ii 3003

24X96P

Al igual que en los apartados anteriores, aplicaremos el Teorema Central del Límite.

Page 44: ESTADÍSTICA EMPRESARIAL II PROBLEMAS DE EXAMEN …media0.webgarden.es/files/media0:4bd76ebb98146.pdf.upl... · 2010. 4. 27. · Estadística Empresarial II 23 La correlación negativa

64 Problemas de examen resueltos

E(B) = ( ) ( )[ ]∑∑∑∑====

+−=

+−

64

1ii

2003

1ii

64

1i

i2003

1ii 3003XE4PE3003

24X96PE =

= (3 200 x 70) – {64 x [(4 x 48) + 3 300]} = 512

Var(B) = ( ) ( )[ ]∑∑∑∑====

+=

+−

64

1ii

22003

1ii

64

1i

i2003

1ii XVar4PVar3003

24X96PVar =

= (3 200 x 102) + (64 x 42 x 42) = 336 384

σB = 579.9862067

( ) )88.0(1579.986

5120579.986

512Bp0Bp −Φ−≈

>−

=> = 1 - 0.1894 = 0.8106

tipificando Aplicando el Teorema Central del Límite

31º) Para estudiar la relación entre el precio y la cantidad vendida de un determinadoartículo, se observa durante cierto tiempo ambas variables en varias tiendas, obteniendo lasiguiente ley de probabilidad conjunta:

X = Precio del artículo; Y = Cantidad adquirida por cliente en cada compra.

X \ Y 25 30 351 0 0.05 0.252 0.1 0.05 0.053 0.4 0.1 0

a) Calcular el coeficiente de correlación de X e Y, comentando el resultado.b) Calcular la probabilidad de que el gasto total por cliente en cada compra sea mayor que 60ptas.

SOLUCIÓN

a) Para calcular el coeficiente de correlación de X e Y, nos hace falta calcular previamente lasesperanzas y las varianzas de X e Y, así como la covarianza entre ambas variables.

Empezaremos calculando las leyes de probabilidad marginales.

Page 45: ESTADÍSTICA EMPRESARIAL II PROBLEMAS DE EXAMEN …media0.webgarden.es/files/media0:4bd76ebb98146.pdf.upl... · 2010. 4. 27. · Estadística Empresarial II 23 La correlación negativa

Estadística Empresarial II 65

X \ Y 25 30 35 p(X = x)1 0 0.05 0.25 0.32 0.1 0.05 0.05 0.23 0.4 0.1 0 0.5

p(Y = y) 0.5 0.2 0.3 1

E(X) = (1x0.3) + (2x0.2) + (3x0.5) = 2.2

E(X2) = (12x0.3) + (22x0.2) + (32x0.5) = 5.6

Var(X) = E(X2) – E2(X) = 5.6 – 2.22 = 0.76

E(Y) = (25x0.5) + (30x0.2) + (35x0.3) = 29

E(Y2) = (252x0.5) + (302x0.2) + (352x0.3) = 860

Var(Y) = E(Y2) – E2(Y) = 860 – 292 = 19

E(XY) = (1x25x0) + (1x30x0.05) + (1x35x0.25) + (2x25x0.1) + (2x30x0.05) +

+ (2x35x0.05) + (3x25x0.4) + (3x30x0.1) + (3x35x0) = 60.75

Cov(X,Y) = E(XY) – E(X)E(Y) = 60.75 – (2.2x29) = -3.05

=−

==ρ19x0.76

3.05Var(Y)Var(X)

Y)Cov(X,YX, -0.80263158

El valor del coeficiente de correlación está próximo a –1 lo que quiere decir que entre las variablesX = Precio del artículo, e Y = Cantidad adquirida por cliente en cada compra hay una relación linealalta y negativa, es decir, cuando aumenta el precio disminuye la cantidad adquirida por el cliente.

b) Sea G = Gasto total por cliente en cada compra = X Y. Nos piden calcular:

p(G > 60) = p(X Y > 60) = p[(X = 2) ∩ (Y = 35)] + p[(X = 3) ∩ (Y = 25)] +

+ p[(X = 3) ∩ (Y = 30)] + p[(X = 3) ∩ (Y = 35)] = 0.05 + 0.4 + 0.1 + 0 = 0.55.

32º) Sean X e Y dos variables aleatorias con función de densidad conjunta:

<<<=

resto el en0

4xy081

y)(x,f YX,

Page 46: ESTADÍSTICA EMPRESARIAL II PROBLEMAS DE EXAMEN …media0.webgarden.es/files/media0:4bd76ebb98146.pdf.upl... · 2010. 4. 27. · Estadística Empresarial II 23 La correlación negativa

66 Problemas de examen resueltos

a) Comprobar que es función de densidad.b) Obtener las funciones de densidades marginales de X e Y.c) Obtener la función de densidad condicionada de Y por X.d) ¿Qué distribuciones siguen X e Y/X=x?e) Calcular E(Y/X=3).f) Calcular la p(X+Y ≤ 4).

SOLUCIÓN

a) Para comprobar que es función de densidad, tendremos que ver que es siempre mayor o igual quecero, y que la integral en todo el dominio de definición vale uno.

Claramente, se puede observar que es mayor o igual que cero, pues o bien vale 1/8 o bien vale 0.

Veamos que la integral vale 1.

( ] ==

∫∫ ∫=

=

==

=

=

=

=

4x

0x

xy0y

4x

0x

xy

0y

dxy81dxdy

81

1x161dxx

81 4x

0x

24x

0x

=

==

=

=

=

=∫

luego efectivamente es función de densidad.

b) Función de densidad marginal de X

fX(x) = x81y

81dy

81 xy

0y

xy

0y

=

=

=

=

=

=∫ 0 < x < 4

luego

<<=

resto elen 0

4x0x81

(x)fX

Función de densidad marginal de Y

fY(y) = y) - (481x

81dx

81 4x

yx

4x

yx

=

=

=

=

=

=∫ 0 < y < 4

X

Y

4

Y = X

Page 47: ESTADÍSTICA EMPRESARIAL II PROBLEMAS DE EXAMEN …media0.webgarden.es/files/media0:4bd76ebb98146.pdf.upl... · 2010. 4. 27. · Estadística Empresarial II 23 La correlación negativa

Estadística Empresarial II 67

luego

<<=

resto elen 0

4y0y) - 4(81

(y)fY

c) Función de densidad condicionada de Y por X.

( )x1

x8181

(x)fy)(x,f

xy/XfX

YX,xY/X ===== 0 < y < x

siempre que 0 < x < 4

d) X → βe(2,1,0,4)

<<

=<<−

β=

−−+

resto elen 0

4x0x81

resto elen 0

bxax)(ba)-x(a)-(b1

q)(p,1

(x)f

1q1p1-qp

X

ya que

81

161

0!1!2!

161

(1)(2))1(2

)04(1

(2,1)1

112 ==ΓΓ+Γ

=−β −+

Y /X=x → U(0,x)

<<=

<<==

resto elen 0

xy0x1

resto elen 0

byaa-b

1

(y)f xY/X

e) Sustituyendo X = 3 en la función de densidad Y/X=x tenemos:

( )313y/Xf 3Y/X === 0 < y < 3

Page 48: ESTADÍSTICA EMPRESARIAL II PROBLEMAS DE EXAMEN …media0.webgarden.es/files/media0:4bd76ebb98146.pdf.upl... · 2010. 4. 27. · Estadística Empresarial II 23 La correlación negativa

68 Problemas de examen resueltos

luego

( ) 1.5y61dy

31yxY/XE

3y

0y

23y

0y

=

===

=

=

=

=∫

f)

p(X + Y ≤ 4) = ∫ ∫=

=

−=

=

2y

0y

y4x

yx

dydx81 =

( ) =−=

= ∫∫

=

=

=

=

−=

=

2y

0y

2y

0y

y4x

yx

dy2y481dyx

81

( ] 0.5y4y81 2y

0y2 =−=

=

=

33º) Sean X e Y dos variables aleatorias con distribuciones γ(2,3) y T(3,6,12) respectivamente,y con Cov(X,Y) = 1. Sean S = 2X – 3Y + 4 y T = 5X + 8Y. Obtener:

a) La esperanza y la varianza de S y de T.b) El coeficiente de correlación entre S y T.

SOLUCIÓN

a) Para calcular las esperanzas y las varianzas pedidas, aplicaremos las propiedades de ambas.Empezaremos calculando la esperanza y la varianza de X y de Y.

Como X → γ(2,3)

E(X) = ==23

ap 1.5 Var(X) = == 22 2

3ap 0.75

y como Y → T(3,6,12)

E(Y) = =++

=++

31263

3bma 7

Var(Y) = ( ) ( )( ) ( ) ( )( )=

−−−=

−−−18

366123-1218

ammba-b 22

3.5

E(S) = E(2X – 3Y + 4) = 2E(X) – 3E(Y) + 4 = (2x1.5) – (3x7) + 4 = -14

X

Y

4

X + Y = 4Y = X

2

Page 49: ESTADÍSTICA EMPRESARIAL II PROBLEMAS DE EXAMEN …media0.webgarden.es/files/media0:4bd76ebb98146.pdf.upl... · 2010. 4. 27. · Estadística Empresarial II 23 La correlación negativa

Estadística Empresarial II 69

Var(S) = Var(2X – 3Y + 4) = 22Var(X) + (-3)2Var(Y) + 2x[2x(-3)]Cov(X,Y) =

= (22x0.75) + [(-3)2x3.5] + {2x[2x(-3)]x1} = 22.5

E(T) = E(5X + 8Y) = 5E(X) + 8E(Y) =(5x1.5) + (8x7) = 63.5

Var(T) = Var(5X + 8Y) = 52Var(X) + 82Var(Y) + 2x(5x8)Cov(X,Y) =

= (52x0.75) + (82x3.5) + [2x(5x8)x1] = 322.75

b) Para calcular el coeficiente de correlación entre S y T necesitamos obtener el valor de lacovarianza entre dichas variables, el cual se obtendrá aplicando sus propiedades.

Cov(S,T) = Cov[(2X – 3Y + 4),(5X + 8Y)] =

= (2x5)Cov(X,X) + (2x8)Cov(X,Y) + (-3x5)Cov(Y,X) + (-3x8)Cov(Y,Y) =

= 10Var(X) + 16Cov(X,Y) -15Cov(X,Y) – 24Var(Y) =

= (10x0.75) + (1x1) – (24x3.5) = -75.5

=−

==75.322x5.22

75.5Var(T)Var(S)

T)Cov(S,TS,ρ -0.885977244.

34º) Una tienda vende dos tipos distintos de leche A y B. Las ganancias diarias por la venta deleche, se distribuye normalmente en cada caso. Se sabe que la ganancia media diaria por laventa de la marca A es de 600 ptas. con una desviación típica de 20, que el 2.28 % de lasganancias diarias obtenida por la venta de la marca B son superiores a 800 y que el 96.9 %están comprendidas entre 690 y 800 ptas.a) Calcular la media y la varianza de las ganancias diarias obtenidas por la venta de leche de lamarca B.b) ¿Cuál será el número mínimo de días que tiene que vender leche de la marca A para que laganancia, por este concepto, supere las 21 900 ptas. con una probabilidad del 0.9938?c) ¿Qué distribución sigue la ganancia semanal (7 días) por la venta de leche?

SOLUCIÓN

a) Sea B = Ganancias obtenidas por la venta de leche de la marca B → N(µ,σ). Para determinar elvalor de los parámetros disponemos de las siguientes igualdades:

p(B > 800) = 0.0228 p(690 < B < 800) = 0.969

Tipificando y despejando tenemos:

Page 50: ESTADÍSTICA EMPRESARIAL II PROBLEMAS DE EXAMEN …media0.webgarden.es/files/media0:4bd76ebb98146.pdf.upl... · 2010. 4. 27. · Estadística Empresarial II 23 La correlación negativa

70 Problemas de examen resueltos

p(B > 800) = 0.0228 ⇒ 0.02288001 =

Φ−σ

µ⇒

⇒ 0.9772800=

Φσ

µ⇒ 2800

=−σ

µ

p(690 < B < 800) = 0.969 ⇒

⇒ 969.0690-0.9772690-800=

σµ−

Φ=

σµ−

Φ

σµ−

Φ ⇒

⇒ 0.0082690=

σµ−

Φ ⇒ 4.2690−=

σµ−

y resolviendo el sistema:

σµ−800 = 2

σµ−690 = -2.4

se obtiene que

µ = 750 y σ = 25

luego B → N(750,25).

E(B) = 750 Var(B) = 252 = 625

b) Sea A = Ganancias obtenidas por la venta de leche de la marca A → N(600,20). Definimos lavariable:

X = Ganancias obtenidas por la venta de leche de la marca A en n días = ∑=

n

1iiA

Aplicando las propiedades de la normal sabemos que X → )n20 N(600n,)n,N(n =σµ

En este apartado piden encontrar un valor de n tal que p(∑=

n

1iiA > 21 900) = 0.9938.

Tipificando se tiene:

Page 51: ESTADÍSTICA EMPRESARIAL II PROBLEMAS DE EXAMEN …media0.webgarden.es/files/media0:4bd76ebb98146.pdf.upl... · 2010. 4. 27. · Estadística Empresarial II 23 La correlación negativa

Estadística Empresarial II 71

0.9938 = p(∑=

n

1iiA > 21 900) =

−Φ−=

>−∑

=

n20n600900211

n20n60090021

n20

n600Ap

n

1ii

−Φ

n20n60090021 = 0.0062 ⇒

n20n60090021 − = -2.5 ⇒

⇒ 600n - 50 n - 21 900 = 0 ⇒

−=

6308.6

n)

=36

37.0069n

la solución n = 36 la tenemos que desechar pues al sustituir su valor en la probabilidad de partida noda 0.9938, que es el valor pedido, sino que da una probabilidad de 0.0062.

Por tanto el valor pedido, redondeando, es n = 37 días.

c) Definimos la variable aleatoria G = Ganancia semanal por la venta de leche = ( )∑=

+7

1iii BA

Como la variable G es suma de variables aleatorias normales e independientes, G también sigue unadistribución normal, veamos de que parámetros.

E(G) = ( ) ( ) ( )( ) ( ) =+=+=

+ ∑∑∑

===

7

1i

7

1iii

7

1iii 750600BEAEBAE 7x1 350 = 9 450

Var(G) = ( ) ( ) ( )( ) ( ) =+=+=

+ ∑∑∑

===

7

1i

227

1iii

7

1iii 2520BVarAVarBAVar 7x1 025 = 7 175

luego G → ( )1757450,9N

Page 52: ESTADÍSTICA EMPRESARIAL II PROBLEMAS DE EXAMEN …media0.webgarden.es/files/media0:4bd76ebb98146.pdf.upl... · 2010. 4. 27. · Estadística Empresarial II 23 La correlación negativa

72 Problemas de examen resueltos

35º) El número medio de cheques ingresados en un banco es de 50 por día con una desviación típicade 2. Calcular la probabilidad de que en un trimestre (81 días) el número de cheques ingresados estéentre 4 000 y 4 100.

SOLUCIÓN

Sea X = Número de cheques ingresados en un banco al día. Sabemos que:

E(X) = 50 σX = 2

Sea Y = Número de cheques ingresados en un banco al trimestre = ∑=

81

1iiX .

( ) ∑∑∑===

==

81

1i

81

1ii

81

1ii 50XEXE = 81 x 50 = 4 050

( ) ∑∑∑===

==

81

1i

281

1ii

81

1ii 2XVarXVar = 81 x 4 = 324

∑=

81

1iiX

σ = 18

por ser Xi independientes

Nos piden: p(4 000 < ∑=

81

1iiX < 4 100)

De la variable aleatoria número de cheques ingresados en un banco al mes no conocemos sudistribución, pero si sabemos que es suma de variables aleatorias independientes idénticamentedistribuidas con esperanza y varianza finitas, con lo cual podemos aplicar el Teorema Central delLímite, y aproximar el valor de dicha probabilidad.

<−

<−

=

<<

∑∑ =

= 1805041004

18

0504X

1805040004p1004X0004p

81

1ii81

1ii

tipificando Aplicando el Teorema Central del Límite

)78.2((2.78) −Φ−Φ≈ = 0.9973 – 0.0027 = 0.9946

Page 53: ESTADÍSTICA EMPRESARIAL II PROBLEMAS DE EXAMEN …media0.webgarden.es/files/media0:4bd76ebb98146.pdf.upl... · 2010. 4. 27. · Estadística Empresarial II 23 La correlación negativa

Estadística Empresarial II 73

36º) En una cafetería se realiza un estudio sobre las variables X = “Nº de cafés servidos pormesa” e Y = “Nº de infusiones servidas por mesa”, con la siguiente ley de probabilidadconjunta:

X \ Y 0 1 20 0 0 0.11 0 0.2 02 0.3 0.1 03 0.1 0.2 0

a) Calcular el coeficiente de correlación de X e Y. Interpretar su resultado.b) Obtener la probabilidad de que el número total de consumiciones (cafés, infusiones)servidas por mesa sea al menos tres.c) Calcular el número medio de cafés servidos por cada mesa en la que se ha servido unainfusión.

SOLUCIÓN

a) Para calcular el coeficiente de correlación de X e Y, nos hace falta calcular previamente lasesperanzas y las varianzas de X e Y, así como la covarianza entre ambas variables.

Empezaremos calculando las leyes de probabilidad marginales.

X \ Y 0 1 2 p(X = x)0 0 0 0.1 0.11 0 0.2 0 0.22 0.3 0.1 0 0.43 0.1 0.2 0 0.3

p(Y = y) 0.4 0.5 0.1 1

E(X) = (0x0.1) + (1x0.2) + (2x0.4) + (3x0.3) = 1.9

E(X2) = (02x0.1) + (12x0.2) + (22x0.4) + (32x0.3) = 4.5

Var(X) = E(X2) – E2(X) = 4.5 – 1.92 = 0.89

E(Y) = (0x0.4) + (1x0.5) + (2x0.1) = 0.7

E(Y2) = (02x0.4) + (12x0.5) + (22x0.1) = 0.9

Var(Y) = E(Y2) – E2(Y) = 0.9 – 0.72 = 0.41

E(XY) = (0x0x0) + (0x1x0) + (0x2x0.1) + (1x0x0) + (1x1x0.2) + (1x2x0) +

+(2x0x0.3) + (2x1x0.1) + (2x2x0) + (3x0x0.1) + (3x1x0.2) + (3x2x0) = 1

Cov(X,Y) = E(XY) – E(X)E(Y) = 1 – (1.9x0.7) = -0.33

Page 54: ESTADÍSTICA EMPRESARIAL II PROBLEMAS DE EXAMEN …media0.webgarden.es/files/media0:4bd76ebb98146.pdf.upl... · 2010. 4. 27. · Estadística Empresarial II 23 La correlación negativa

74 Problemas de examen resueltos

=−

==0.41x0.89

0.33Var(Y)Var(X)

Y)Cov(X,YX,ρ -0.5462947

El valor del coeficiente de correlación esta aproximadamente a la mitad entre 0 y –1 lo quequiere decir que entre las variables X = Nº de cafés servidos por mesa, e Y = Nº de infusionesservidas por mesa hay una relación lineal media y negativa, es decir, cuando aumenta el nº decafés servidos disminuye el nº de infusiones servidas por mesa.

b) p( X + Y ≥ 3) = p( X + Y = 3) + p( X + Y = 4) + p( X + Y = 5) =

= p(X = 3, Y = 0) + p(X = 2, Y = 1) + p(X = 1, Y = 2) + p(X = 3, Y = 1) +

+ p(X = 2, Y = 2) + p(X = 3, Y = 2) = 0.1 + 0.1 + 0 + 0.2 + 0 + 0 = 0.4

c)

( ) =

==+

==+

==+

==== 1Y

3Xp3x1Y2Xpx21Y

1Xpx11Y0Xpx01Y

XE

21)Y(p

1)Y3(Xpx31)Y(p

1)Y2(Xpx21)Y(p

1)Y1(Xpx1 =

=

==+

=

==+

=

===

37º) Sea (X,Y) una variable aleatoria bidimensional con función de densidad conjunta:

≤≤≤

=resto0

2yx1ky)(x,f YX,

a) Comprobar que el valor de k es 2.b) ¿Son independientes las variables X e Y?c) ¿Qué distribución sigue la variable X? ¿y cuál la variable Y?d) Calcular la función de densidad de la variable Z = 4 - 2X. ¿Qué distribución sigue lavariable Z?e) Calcular: Var( 3X-2Y+1).

f) Calcular:

=3/2YXE .

SOLUCIÓN

a) Para comprobar que la constante vale 2:

( ] k21

2x2xkdxx)(2kdxykdxdyk1

2x

1x

22x

1x

2x

1x

2yxy

2x

1x

2y

xy

=

−=−==

=

=

=

=

=

=

=

==

=

=

=

=∫∫∫ ∫

Page 55: ESTADÍSTICA EMPRESARIAL II PROBLEMAS DE EXAMEN …media0.webgarden.es/files/media0:4bd76ebb98146.pdf.upl... · 2010. 4. 27. · Estadística Empresarial II 23 La correlación negativa

Estadística Empresarial II 75

Como 1 = 21 k ⇒ k = 2

b) Para ver si son independientes las variables X e Y, comprobaremos si se cumple la igualdad:(y)f(x)fy)(x,f YXYX, = , para ello necesitamos calcular primero las marginales de X y de Y.

Función de densidad marginal de X

( ]∫=

=

== −===

2y

xy

2yxyX )x(22y2dy2(x)f 1 < x < 2

luego

<<−

=resto elen 0

2x1)x2(2(x)fX

Función de densidad marginal de Y

( ]∫=

=

== −===

yx

1x

yx1xY )1(y2x2dx2(y)f 1 < y < 2

luego

<<

=resto elen 0

2y11)-y (2(y)fY

Como (y)f(x)fy)(x,f YXYX, ≠ , entonces X e Y no son independientes.

c) X → βe(1,2,1,2)

<<

=<<−

=

−−

resto02y1x)-(2 2

resto0

byax)(ba)-(xa)-(b1

q)(p,1

(x)f

1q1p1-q+p

ya que

2 1!0!2!

(2) (1)2)(1

1)(21

(1,2)1

1-2+1 ==ΓΓ+Γ

=−β

Y → βe(2,1,1,2)

Page 56: ESTADÍSTICA EMPRESARIAL II PROBLEMAS DE EXAMEN …media0.webgarden.es/files/media0:4bd76ebb98146.pdf.upl... · 2010. 4. 27. · Estadística Empresarial II 23 La correlación negativa

76 Problemas de examen resueltos

<<−

=<<−

=

−−

resto02y1)1(y 2

resto0

b y ay)(ba)-(ya)-(b1

q)(p,1

(y)f

1q1p1-q+p

ya que

21!0!2!

(1) (2))1(2

)1-(21

(2,1)1

1-2+1 ==ΓΓ+Γ

d) Z = 4 – 2X. Para calcular la función de densidad de la variable Z primero calculamos su funciónde distribución FZ(z).

−=

≤−=

≥=≤−=≤=2

z4F12

z4Xp12

z4Xpz)2X(4pz)(Zp(z)F XZ

2z

2z422

21

2z4f

21

21

2z4f

2z4F1(z)F(z)f XXX

'ZZ =

−=

=

−=′

−==

1 < x < 2 ⇒ 1 < 2

z4 − < 2 ⇒ 2 < 4 - z < 4 ⇒ -2 < -z < 0 ⇒ 0 < z < 2

<<=

resto0

2z02z

(z)f Z

Z → βe(2,1,0,2)

<<

=<<−

β=

−−

resto0

2z02z

resto0

b y az)(ba)-(za)-(b1

q)(p,1

(z)f

1q1p1-q+p

Z

ya que

21

41

1!0!2!

41

(1) (2))1(2

)0-(21

(2,1)1

1-2+1 ==ΓΓ+Γ

e) Como sabemos la distribución que siguen las variables X e Y aplico las formulascorrespondientes para calcular las varianzas y esperanzas

Si X → βe(p,q,a,b) ⇒ E(X) = a + (b-a)qp

p+

Var(X) = ( )2

2

q)(p1)q(pqpab+++

Page 57: ESTADÍSTICA EMPRESARIAL II PROBLEMAS DE EXAMEN …media0.webgarden.es/files/media0:4bd76ebb98146.pdf.upl... · 2010. 4. 27. · Estadística Empresarial II 23 La correlación negativa

Estadística Empresarial II 77

Como X → β(1,2,1,2) e Y → β(2,1,1,2), tenemos:

E(X) = 1 + ( 2 -1)21

1+

= 34 Var(X) =

181

2)(11)2(12x11)(2

2

2

=+++

E(Y) = 1 + ( 2 -1)12

2+

= 35 Var Y =

181

2)(11)2(11x21)(2

2

2

=+++

=

=

= ∫ ∫∫ ∫

=

=

=

=

∞=

∞=

∞=

−∞=

2y

1y

yx

1x

y

- y

x

xYX, dydxyx2dydxy)(x,fyx(XY)E

( ] 2.252y

4ydy1)(yydy xy

2y

1y

242y

1y

22y

1y

yx1x

2 =

−=−==

=

=

=

=

=

=

=

= ∫∫

Cov (X,Y) = E(XY) – E(X)E(Y) = 2.25 –

35x

34 =

361

Por lo tanto: Var(3X-2Y+1) = 32 Var(X) + 22 Var(Y) – 2 x 3 x 2 x Cov (X,Y) =

= 9 Var(X) + 4 Var(Y) – 12 Cov (X,Y) =

=

181x9 +

181x4 –

361x21 =

187

f) ( ) ∫∞=

∞==

==

x

-x3/2Y

X dx(x)fx3/2 YXE

Función de densidad condicionada de X por Y: (x)f yYX/ =

( ) yx1 1-y

11)-y ( 2

2(y)f

y)(x,fxf

Y

YX,yX/Y ≤≤====

siempre que 1 ≤ y ≤ 2

<<==

resto elen 0

yx11-y

1

(x)f yX/Y ⇒ <<

== resto elen 03/2x12

(x)f 2/3X/Y

Page 58: ESTADÍSTICA EMPRESARIAL II PROBLEMAS DE EXAMEN …media0.webgarden.es/files/media0:4bd76ebb98146.pdf.upl... · 2010. 4. 27. · Estadística Empresarial II 23 La correlación negativa

78 Problemas de examen resueltos

( ) ( ]45xdx2xdx(x)fx3/2 Y

XE 3/2x1x

23/2x

1x

x

-x3/2Y

X ======

=

=

=

∞=

∞== ∫∫

38º) Para subir a la cima en una estación de ski, se necesita coger cada uno de los remontes dedos pistas. El tiempo de espera en minutos para coger el remonte de cada pista es una variablealeatoria con la siguiente función de densidad:

>

=−

resto00xe

(x)fx

X

a) ¿Qué distribución sigue la variable X?b) ¿Cuál es la probabilidad de que el tiempo de espera para subir a la cima no sea mayor que6?c) Un grupo de 21 personas desciende por una de las pistas con intención de volver a tomar elremonte de esa pista. ¿Cual es la probabilidad de que menos de dos personas de ese grupotengan que esperar para coger el remonte entre 3 y 4 minutos?

SOLUCIÓN

a) X → γ(1,1) = ε(1), pues

>

=>

=resto0

0xeresto0

0xe a(x)f

x-x-a

X

b) Sean X = Tiempo de espera para coger el remonte de la pista 1 Y = Tiempo de espera para coger el remonte de la pista 2

Son variables independientes con la misma distribución γ(1,1) = ε(1). Definimos la variable Z =Tiempo de espera para subir a la cima = X + Y. Entonces por propiedades de la distribución gammay dado que X e Y son independientes, la variable Z → γ(1,2) con lo que su función de densidad es:

>

=>

Γ=resto0

0 zez

resto0

0 zez(p)a

(z)fz-

az-1-pp

Z

pues

a(p)

1(2)

11!

1 p 2

Γ Γ= = =

Nos piden calcular la probabilidad de que el tiempo de espera para subir a la cima no sea mayor que6, luego

Page 59: ESTADÍSTICA EMPRESARIAL II PROBLEMAS DE EXAMEN …media0.webgarden.es/files/media0:4bd76ebb98146.pdf.upl... · 2010. 4. 27. · Estadística Empresarial II 23 La correlación negativa

Estadística Empresarial II 79

( ] =+−==≤ ∫∫=

=

−=

=−

=

=

6z

0z

z6z0z

z

6z

0z

z dzeezdzez)6Z(p

( ] 9826487.0e71ee6 66z0z

z6 =−=−+−= −=

=−−

c) Sea W = Nº de personas que esperan entre 3 y 4 minutos de entre las 21 personas que desciendenla pista.

W → B(n,p) donde n = 21 y p = p(éxito) = p(3 < X < 4)

p(3 < X < 4) = ( ] 90.03147142eeedxe 434x3x

x4x

3x

x =−=−= −−=

=−

=

=

−∫

W → B(21, 0.031471429)

Nos piden la probabilidad de que menos de dos personas de ese grupo tengan que esperar paracoger el remonte entre 3 y 4 minutos:

p(W < 2) = p(W = 0) + p(W = 1) =

=−

+−

= 201210 )031471429.01(031471429.0

121

)031471429.01(031471429.0021

= 0.859575567.

39º) Una empresa necesita personal auxiliar administrativo, para su selección realiza distintaspruebas, la primera y eliminatoria consiste en escribir un texto a máquina durante un minuto,pasa la prueba quien supere 240 pulsaciones correctas por minuto.Un candidato da 3, 4, 5, 6 pulsaciones por segundo con unas probabilidades de 0.1, 0.3, 0.4 y0.2 respectivamente, pero comete 0, 1, 2 errores por segundo, con probabilidades 1/5, 3/5 y 1/5respectivamente. Calcular la probabilidad de que este candidato supere la primera prueba.(Trabajar con dos decimales).

SOLUCIÓN

Consideremos X = Nº pulsaciones por segundo, su esperanza y su varianza es:

X 3 4 5 6P( X = x) 0.1 0.3 0.4 0.2

E(X) = ∑=

=6

3xx)p(Xx = (3 x 0.1) + (4 x 0.3) + (5 x 0.4) + (6 x 0.2) = 4.7

Page 60: ESTADÍSTICA EMPRESARIAL II PROBLEMAS DE EXAMEN …media0.webgarden.es/files/media0:4bd76ebb98146.pdf.upl... · 2010. 4. 27. · Estadística Empresarial II 23 La correlación negativa

80 Problemas de examen resueltos

E( 2X ) = ∑=

=6

3x

2 x)p(Xx = (32 x 0.1) + (42 x 0.3) + (52 x 0.4) + (62 x 0.2) = 22.9

Var(X) = E( 2X ) - E2(X) = 22.9 – 4.72 = 0.81

y consideremos Y = Nº errores por segundo, su esperanza y su varianza es:

Y 0 1 2P( Y = y) 1/5 3/5 1/5

E(Y) = ∑=

=2

0xy)(Ypy =

+

+

51x2

53x1

51x0 = 1

E( 2Y ) = ∑=

=2

0x

2 y)(Ypy =

+

+

51x2

53x1

51x0 222 =

57

Var(Y) = E( 2Y ) - E2(Y) = 57 – 12 =

52 = 0.4

Sea W = Nº de pulsaciones correctas en un minuto = ∑∑==

−60

1ii

60

1ii YX

Nos piden: p(W > 240) = p( ∑∑==

−60

1ii

60

1ii YX > 240)

E(W) = E( ∑∑==

−60

1ii

60

1ii YX ) = ( )∑

=

60

1iiXE – ( )∑

=

60

1iiYE = (60 x 4.7) – (60 x 1) = 222

Var (W) = Var ( ∑∑==

−60

1ii

60

1ii YX ) = ( )∑

=

60

1iiXVar + ( )∑

=

60

1iiYVar = (60 x 0.81) + (60 x 0.4) = 72.6

X e Y independientes

De la variable aleatoria W no conocemos su distribución, pero si sabemos que es suma y resta devariables aleatorias independientes idénticamente distribuidas con esperanza y varianza finitas, conlo cual podemos aplicar el Teorema Central del Límite, y aproximar el valor de dicha probabilidad.

p(W > 240) = 1 - ≈

<−−

−=

<−

∑∑∑∑ ==

== 72.6222240

72.6

222YXp1240)YX(p

60

1ii

60

1ii60

1ii

60

1ii

tipificando Aplicando el Teorema Central del Límite

0174.09826.01(2.11)1 =−=Φ−≈

Page 61: ESTADÍSTICA EMPRESARIAL II PROBLEMAS DE EXAMEN …media0.webgarden.es/files/media0:4bd76ebb98146.pdf.upl... · 2010. 4. 27. · Estadística Empresarial II 23 La correlación negativa

Estadística Empresarial II 81

40º) En una feria, una churrería hace un estudio de las variables X = “Nº de churros quecompra el cliente” e Y = “Nº de buñuelos que compra el cliente”, con la siguiente ley deprobabilidad conjunta:

X \ Y 0 6 126 0.2 0 012 0.4 0.1 0.118 0 0.1 024 0.1 0 0

a) Calcular el coeficiente de correlación. Interpretar su resultado.b) Calcular el número medio de buñuelos comprados por un cliente que sabemos hacomprado ya 12 churros.

SOLUCIÓN

a) Sean X = Nº de churros que compra el cliente, e Y = Nº de buñuelos que compra el cliente.Calculemos sus marginales, sus esperanzas, sus varianzas y la covarianza para poder obtener elcoeficiente de correlación.

X \ Y 0 6 12 P(X = x)6 0.2 0 0 0.212 0.4 0.1 0.1 0.618 0 0.1 0 0.124 0.1 0 0 0.1

P(Y = y) 0.7 0.2 0.1 1

E(X) = (6x0.2) + (12x0.6) + (18x0.1) + (24x0.1) = 12.6

E(X2) = (62x0.2) + (122x0.6) + (182x0.1) + (242x0.1) = 183.6

Var(X) = E(X2) - E2(X) = 183.6 - 12.62 = 24.84

E(Y) = (0x0.7) + (6x0.2) + (12x0.1) = 2.4

E(Y2) = (02x0.7) + (62x0.2) + (122x0.1) = 21.6

Var(Y) = E(Y2) - E2(Y) = 21.6 - 2.42 = 15.84

E(XY) = (6x0x0.2) + (6x6x0) + (6x12x0) + (12x0x0.4) + (12x6x0.1) +(12x12x0.1) +

+ (18x0x0) + (18x6x0.1) + (18x12x0) + (24x0x0.1) +(24x6x0) + (24x12x0) = 32.4

Cov(X,Y) = E(XY) - E(X)E(Y) = 32.4 - (2.4x12.6) = 2.16

10889310.15.84x24.84

2.16Var(Y)Var(X)

Y)Cov(X,YX, ===ρ

Page 62: ESTADÍSTICA EMPRESARIAL II PROBLEMAS DE EXAMEN …media0.webgarden.es/files/media0:4bd76ebb98146.pdf.upl... · 2010. 4. 27. · Estadística Empresarial II 23 La correlación negativa

82 Problemas de examen resueltos

Como el coeficiente de correlación es positivo, quiere decir que cuanto mayor es el número dechurros que compra el cliente, mayor es el número de buñuelos que compra, y al revés. Aunque elvalor es tan pequeño, tan próximo a cero, que la correlación entre el nº de churros y el nº debuñuelos que compra el cliente es prácticamente nula.

b) Empezaremos calculando la ley de probabilidad de Y/X=12.

64

0.60.4

(12)f(12,0)f

12)X/(0fY

YX,12Y/X =====

61

0.60.1

(12)f(12,6)f

12)X/(6fY

YX,12Y/X =====

61

0.60.1

(12)f(12,12)f

12)X/(12fY

YX,12Y/X =====

luego

Y/X=12 p(Y/X=12)0 4/66 1/612 1/6

con lo cual su esperanza es:

E(Y/X=12) = (0x64 ) + (6x

61 ) + (12x

61 ) = 3.

41º) Sean X e Y dos variables aleatorias continuas con función de densidad conjunta:

≤≤≤

=resto el en0

3xy0kxy)(x,f YX,

a) Comprobar que la constante k vale 91 .

b) Obtener las funciones de densidades marginales de X e Y. ¿Qué distribución sigue lavariable X?c) Obtener la función de densidad condicionada de Y por X. ¿Qué distribución sigue lavariable Y/X=x?d) Calcular E(Y/X=1).e) Calcular la p(X+Y ≤ 2).

SOLUCIÓN

a) Para comprobar que la constante vale 1/9, usaremos el hecho de que la integral en todo eldominio de definición de la función de densidad es 1.

Page 63: ESTADÍSTICA EMPRESARIAL II PROBLEMAS DE EXAMEN …media0.webgarden.es/files/media0:4bd76ebb98146.pdf.upl... · 2010. 4. 27. · Estadística Empresarial II 23 La correlación negativa

Estadística Empresarial II 83

∫∫∫ ∫=

=

=

=

=

=

=

=

=

=

=

−=

=

=

3y

0y

223y

0y

3x

yx

23y

0y

3x

yx

dy 2y

23 k dy

2xk dy dxk x 1

91k k9

6yy

29k

3y

0y

3

=⇒=

−=

=

=

o bien:

( ]91k k9

3xkdx xkdxyx k dx dyx k1

3x

0x

33x

0x

2xy0y

3x

0x

3x

0x

xy

0y

=⇒=

===

=

=

=

=

=

==

=

=

=

=

=

=∫∫∫ ∫

b) Marginal de X

∫=

=

=

=

≤≤=

==

xy

y 0

2xy

0yX 3x0

9x

9yx dy

9x (x)f

X → βe(3,1,0,3)

<<

=<<−

=

−−+

resto elen 0

3x0x91

resto elen 0

bxax)(ba)-x(a)-(b1

q)(p,1

(x)f21q1p

1-qp

ya que

91

271

0!1!3!

271

(1)(3))1(3

)03(1

(3,1)1

113 ==ΓΓ+Γ

=− −+β

Marginal de Y

∫=

=

=

=

≤≤=

==

3 23x

yx

2

Y 3y018y-

21

18xdx

9x(y)f

x

yx

c) Función de densidad condicionada de Y por X: (y)f xXY/ =

Page 64: ESTADÍSTICA EMPRESARIAL II PROBLEMAS DE EXAMEN …media0.webgarden.es/files/media0:4bd76ebb98146.pdf.upl... · 2010. 4. 27. · Estadística Empresarial II 23 La correlación negativa

84 Problemas de examen resueltos

( ) xy0x1

9x9x

(x)fy)(x,f

yf 2X

YX,xY/X ≤≤====

siempre que 0 ≤ x ≤ 3

Y/X=x → U(0,x)

<<=

<<==

resto elen 0

xy0x1

resto elen 0

byaa-b

1

(y)f xY/X

d) Sustituyendo X = 1 en la función de densidad Y/X=x tenemos:

( ) 1yf 1Y/X == 0 < y < 1

luego

21

2ydy1y1)Y/X(E

1y

0y

21y

0y

=

===

=

=

=

=∫

f)

p(X + Y ≤ 2) = ∫ ∫=

=

−=

=

1y

0y

y2x

yx

dydx9x =

=−

=

= ∫∫

=

=

−=

=

=

=

dy922dy

18x 1y

0y

y2x

yx

1y

0y

2 y

= ( ]91y2y

91 1

02 =−

=

=

yy

42º) Del presupuesto destinado para las fiestas, se tiene que el gasto (en millones de ptas.) paralos fuegos artificiales de un día es una variable aleatoria cuya función de densidad es lasiguiente:

≤≤−=

resto el en01x0x)(13(x)f

2

X

a) ¿Qué distribución sigue dicha variable?

X

Y

1

X + Y = 2

Y = X

3

1

Page 65: ESTADÍSTICA EMPRESARIAL II PROBLEMAS DE EXAMEN …media0.webgarden.es/files/media0:4bd76ebb98146.pdf.upl... · 2010. 4. 27. · Estadística Empresarial II 23 La correlación negativa

Estadística Empresarial II 85

b) Calcular la probabilidad de que el gasto en los fuegos artificiales de un día sea inferior a100 000 ptas.c) Hay fuegos artificiales durante nueve días. Calcular la probabilidad de que al menos un díade esos el gasto sea menor que 100 000 ptas.

SOLUCIÓN

a) X → β(1,3)

<<

=<<−

=

−−

resto01x0 x)- (13

resto0

1x0x)(1xq)(p,

1

(x)f2

1q1p

ya que

32!0!

3!(3) (1))3(1

(1,3)1

==ΓΓ+Γ

b) p(X< 0.1) = ( ] 271.0x)(1dxx)(13 0.1x0x

30.1x

0x

2 =−−=−=

=

=

=∫

c) Sea W = Nº de días en los que el gasto en fuegos artificiales es menor que 0.1 millones de entrelos nueve días

W → B(n,p) donde n = 9 y p = probabilidad de éxito = p( X < 0.1) = 0.271, luego

W → B(9,0.271)

p(W ≥ 1) = 1- p(W < 0) = 1 – p(W = 0) =

= 1 - 90 0.271)(1271.009

= 1 - 0.0581497 = 0.9418503

43º) Una compañía de teatro tiene dos obras en cartelera: “Sueños” y “Pasiones”, cada díarealiza una representación de cada una. Las ganancias (en miles de pesetas) por función sedistribuye normalmente en cada caso.Se sabe que la ganancia media por representación de la obra de teatro “Sueños” es de 600 milptas. con una desviación típica de 20, que el 2,28% de las ganancias obtenidas porrepresentación de la obra “Pasiones” son superiores a 800 mil ptas. y que el 96,9% estáncomprendidas entre 690 y 800 mil ptas.

a) Calcular la media y la varianza de las ganancias obtenidas por representación de la obra“Pasiones”.

Page 66: ESTADÍSTICA EMPRESARIAL II PROBLEMAS DE EXAMEN …media0.webgarden.es/files/media0:4bd76ebb98146.pdf.upl... · 2010. 4. 27. · Estadística Empresarial II 23 La correlación negativa

86 Problemas de examen resueltos

b) ¿Qué distribución sigue la ganancia total semanal (7 días) por la representación de ambasobras?

SOLUCIÓN

a) Sean X = Ganancia (en miles ptas.) por la representación de la obra “Sueños” → N(600,20) Y = Ganancia (en miles ptas.) por la representación de la obra “Pasiones” → N(µ,σ)

Para determinar el valor de los parámetros disponemos de las siguientes igualdades:

p(B > 800) = 0.0228 p(690 < B < 800) = 0.969

Tipificando y despejando tenemos:

p(B > 800) = 0.0228 ⇒ 0.02288001 =

Φ−σ

µ⇒

⇒ 0.9772800=

Φσ

µ⇒ 2800

=−σ

µ

p(690 < B < 800) = 0.969 ⇒

⇒ 969.0690-0.9772690-800=

σµ−

Φ=

σµ−

Φ

σµ−

Φ ⇒

⇒ 0.0082690=

σµ−

Φ ⇒ 4.2690−=

σµ−

y resolviendo el sistema:

σµ−800 = 2

σµ−690 = -2.4

se obtiene que

µ = 750 y σ = 25

luego Y → N(750,25).

E(Y) = 750 Var(Y) = 252 = 625

Page 67: ESTADÍSTICA EMPRESARIAL II PROBLEMAS DE EXAMEN …media0.webgarden.es/files/media0:4bd76ebb98146.pdf.upl... · 2010. 4. 27. · Estadística Empresarial II 23 La correlación negativa

Estadística Empresarial II 87

b) Sea Z = X + Y→ N(750 + 600, 22 2520 + ) = N(1350, 1025 ).

Definimos la variable:

G = Ganancia total semanal por la representación de ambas obras =∑=

7

1iiZ

Aplicando las propiedades de la normal sabemos que

G → 71759450,(N)1025x7 1350,xN(7)n,N(n ==σµ )

44º) Una empresa de alquiler de vehículos es rentable si el número total de coches alquiladosen los 80 primeros días del año es superior al número total de motos alquiladas en todo el año(360 días).

Al día, el número de coches que esa empresa alquila es de 10, 11, 12, 13 y 14 con unasprobabilidades de 0.2, 0.1, 0.4, 0.1 y 0.2 respectivamente, y el número de motos alquiladas,también al día, es de 1, 2, 3 y 4 con probabilidades 0.2, 0.3, 0.3 y 0.2 respectivamente. Calcularla probabilidad de que el negocio sea rentable.

SOLUCIÓN

Definimos las variables: X = Nº coches alquilados al día e Y = Nº motos alquiladas al día.Calculemos sus medias y sus varianzas.

X 10 11 12 13 14P(X = x) 0.2 0.1 0.4 0.1 0.2

E(X) = ∑=

=14

10xx)p(Xx = (10 x 0.2) + (11 x 0.1) + (12 x 0.4) + (13 x 0.1) + (14 x 0.2) = 12

E( 2X ) = ∑=

=14

10x

2 x)p(Xx = (102 x 0.2) + (112 x 0.1) + (122 x 0.4) + (132 x 0.1) + (142 x 0.2) = 145.8

Var(X) = E( 2X ) - E2(X) = 145.8 – 122 = 1.8

Y 1 2 3 4P(Y = y) 0.2 0.3 0.3 0.2

E(Y) = ∑=

=4

1xy)p(Yy = (1 x 0.2) + (2 x 0.3) + (3 x 0.3) + (4 x 0.2) = 2.5

E( 2Y ) = ∑=

=4

1x

2 y)p(Yy = (12 x 0.2) + (22 x 0.3) + (32 x 0.3) + (42 x 0.2) = 7.3

Var(Y) = E( 2Y ) - E2(Y) = 7.3 – 2.52 = 1.05

Page 68: ESTADÍSTICA EMPRESARIAL II PROBLEMAS DE EXAMEN …media0.webgarden.es/files/media0:4bd76ebb98146.pdf.upl... · 2010. 4. 27. · Estadística Empresarial II 23 La correlación negativa

88 Problemas de examen resueltos

El negocio es rentable si ∑∑==

>360

1ii

80

1ii YX

Sea W = ∑∑==

−360

1ii

80

1ii YX

Nos piden: p(W > 0) = p( ∑∑==

−360

1ii

80

1ii YX > 0)

E(W) = E( ∑∑==

−360

1ii

80

1ii YX ) = ( )∑

=

80

1iiXE – ( )∑

=

360

1iiYE = (80 x 12) – (360 x 2.5) = 60

Var (W) = Var ( ∑∑==

−360

1ii

80

1ii YX ) = ( )∑

=

80

1iiXVar + ( )∑

=

360

1iiYVar = (80 x 1.8) + (360 x 1.05) = 522

X e Y independientes

De la variable aleatoria W no conocemos su distribución, pero si sabemos que es suma y resta devariables aleatorias independientes idénticamente distribuidas con esperanza y varianza finitas, conlo cual podemos aplicar el Teorema Central del Límite, y aproximar el valor de dicha probabilidad.

p(W > 0) = 1 - ≈

≤−−

−=

≤−

∑∑∑∑ ==

== 522600

522

60YXp10)YX(p

360

1ii

80

1ii360

1ii

80

1ii

tipificando Aplicando el Teorema Central del Límite

(-2.63)1 Φ−≈ = 1 – 0.0043 = 0.9957.

45º) El tiempo medido en horas, que un profesor emplea en resolver una duda a un alumnopor medio de tutoría electrónica es una variable aleatoria que sigue una distribución gammaγ(2, ¼).a) Si en un día resuelve ocho consultas, ¿cuál es la probabilidad de que tarde menos de unahora y cuarto?. ¿Cuál es el tiempo medio que tarda en resolver ocho consultas?b) Si durante el curso recibe 240 consultas, ¿cuál es la probabilidad de que emplee entre 25 y35 horas en responder?c) Durante todo el curso, este profesor ha realizado 40 horas de atención al alumno medianteel procedimiento de tutoría electrónica. ¿Cuántas consultas calculas que habrá contestadocomo máximo, con una probabilidad de 0.9772?

Page 69: ESTADÍSTICA EMPRESARIAL II PROBLEMAS DE EXAMEN …media0.webgarden.es/files/media0:4bd76ebb98146.pdf.upl... · 2010. 4. 27. · Estadística Empresarial II 23 La correlación negativa

Estadística Empresarial II 89

SOLUCIÓN

a) X = “Tiempo empleado en contestar a una consulta” → γ (2, ¼)

X1, …X8 i.i.d. γ (2, ¼) ⇒ ( )2,2X8

1ii γ→∑

=

por lo tanto:

( ) =

+−===

< ∫∫∫∑ −−−−

=

1.25

0

2x1.250

2x1.25

0

2x1.25

0

2x8

1ii dxexe2dx2xe2dx4xe1.25Xp

0.71270251ee2.52

e2e2.5 2.5-2.5-1.25

0

2x2,5 =+−−=

−+−=

1apXE

8

1ii ==

∑=

El tiempo medio en resolver 8 consultas es una hora.

b) Sea Y = “Tiempo empleado en responder 240 consultas” = .X240

1ii∑

=

X1, …X240 i.i.d. γ (2, ¼) con µ = E(Xi) = 81 < ∞ y σ2 = Var(Xi) =

161 < ∞, aplicando el

T. C. L.

)1,0N(15

30Y L→−

( )T.C.L.elporotipificand 15

303515

30Y15

3025p35Y25p↓↓=

−<

−<

−=<<

( ) ( ) ( ) 803.0129.1229.129.1T.C.L.elpor

=−=−−≅↓

φφφ

c) Y = “Tiempo empleado en responder n consultas” = ( )4,2Xn

1ii

nγ→∑=

.

Si el tiempo dedicado a tutoría electrónica en todo el año es 40 horas, cabe suponer que el númerode consultas sea suficientemente grande (n>30), y podemos aplicar el T. C. L.

X1, …Xn i.i.d. γ (2, ¼) con µ = E(Xi) = 81 < ∞ y σ2 = Var(Xi) =

161 < ∞ , por el T. C. L.

1)N(0,16n

8nXL

n

1ii

→−∑

=

( ) 0.97724n

8n404n

8nYp40YpT.C.L. elpor otipificand↓↓=

−<

−=<

0320-n4n24n

8n40=+⇒=

=⇒−

=válida256

válidano400n

1620

n

Page 70: ESTADÍSTICA EMPRESARIAL II PROBLEMAS DE EXAMEN …media0.webgarden.es/files/media0:4bd76ebb98146.pdf.upl... · 2010. 4. 27. · Estadística Empresarial II 23 La correlación negativa

90 Problemas de examen resueltos

46º) Sea la variable aleatoria bidimensional (X, Y), donde X es el número de kg. de aceitunas eY es el número de kg. de pepinillos vendidos diariamente en una tienda de encurtidos.Conocemos las siguientes funciones de densidad:

( ) 0x050

x10xfX 1<<−

=

( ) 10yxx10

1y/xfY/X <<−

=

a) Calcular la función de densidad conjunta de (X, Y) y la marginal de Y.b) ¿Las marginales siguen alguna distribución conocida? ¿Son independientes X e Y?c) Calcular la esperanza y la varianza del número total de kg. vendidos diariamente entreaceitunas y pepinillos.d) Calcular la probabilidad de que se vendan como máximo 10 kg. entre aceitunas ypepinillos.e) Calcular el número de kg. de pepinillos que se espera vender al cabo de un día, sabiendoque se venden 4 kg. de aceitunas.f) Si los pepinillos se venden a 1 000 ptas. el kg., ¿qué distribución siguen los ingresos diariospor las ventas de este producto?

SOLUCIÓN

a) ( ) 10yx0501

x101

50x10yx,f xy <<<=

−−

=

( ) 10y050y

50xdx

501yf

y

0

y

0Y <<=

== ∫

b) X → βe(1, 2, 0, 10) e Y → βe(2, 1, 0, 10)

La constante en ambos casos va a ser la misma:

( )( ) ( ) ( )( )

501

1002

103

211

abqp,1

1211qp ==

ΓΓΓ

=− −+

−+β

X e Y son dependientes pues fX(x) fY(y) ≠ fXY(x, y)

c) ( ) ( )3

10qp

pabaXE =+

−+= ( ) ( )320

qppabaYE =+

−+=

( ) ( )( )( ) 9

50qp1qp

pqabXVar 2

2

=+++

−= ( ) ( )

( )( ) 950

qp1qppqabYVar 2

2

=+++

−=

( ) ( ) =−=

== ∫ ∫ ∫∫

10

0

10

x

10

0

210

x

10

0

2

dxx100x100

1dx2y

50xdydx

50xyXYE

Page 71: ESTADÍSTICA EMPRESARIAL II PROBLEMAS DE EXAMEN …media0.webgarden.es/files/media0:4bd76ebb98146.pdf.upl... · 2010. 4. 27. · Estadística Empresarial II 23 La correlación negativa

Estadística Empresarial II 91

254

x50x100

110

0

42 =

−=

Por tanto, ( )925

320

31025YX,Cov =−=

Esperanza del número total de kg. de aceitunas y pepinillos vendidos en un día:

( ) ( ) ( ) 10320

310YEXEYXE =+=+=+ kg.

Varianza del número total de kg. de aceitunas y pepinillos vendidos en un día:

( ) ( ) ( ) ( )3

509252

950

950YX,2CovYVarXVarYXVar =++=++=+

d)

5

10

105

x = y

y = 10 -x

( ) =−

=

==<+ ∫∫∫ ∫

−−

dx50

2x10dx50ydydx

50110YXp

5

0

x10

x

5

0

5

0

x10

x

[ ] [ ]212550

501x10x

501 5

02 =−=−=

e) ( ) 10y4614y/xfY/X <<==

( ) 72

1044Y/XE =+

== , por ser uniforme la función de densidad condicionada.

f) Sea Z = Ingresos por las ventas diarias de pepinillos = 1000 Y

( ) ( ) ( )

=

≤=≤=≤=

1000zF

1000zYpzY1000pzZpzF YZ

( ) 10.000z050.000.000

z1000

11000

zfzf YZ <<=

=

Por tanto, Z → βe(2, 1, 0, 10.000).

Page 72: ESTADÍSTICA EMPRESARIAL II PROBLEMAS DE EXAMEN …media0.webgarden.es/files/media0:4bd76ebb98146.pdf.upl... · 2010. 4. 27. · Estadística Empresarial II 23 La correlación negativa

92 Problemas de examen resueltos

47º) El consumo mensual de carne de vacuno por persona, en cuatro países de la ComunidadEuropea antes de la crisis de las vacas locas, se distribuía normalmente. Los datos de lasmedias y desviaciones típicas figuran en la siguiente tabla:

Nº Medio de kg. D. TípicaGran Bretaña 2.6 0.2Francia 2.8 0.3Alemania 3.6 0.6España 2.5 0.5

Se puede considerar que estas cantidades son independientes, por dependerfundamentalmente de los hábitos de consumo y costumbres gastronómicas de los distintospaíses. A raíz del problema de las vacas locas, se ha observado que Gran Bretaña hadisminuido su consumo en un 30%, Francia en un 10%, Alemania en un 15% y España en un8%.

a) ¿Cuál es la probabilidad de que actualmente un individuo en Gran Bretaña consumamenos de 2.1 kg. de carne de vacuno al mes?b) ¿Cuál es la probabilidad de que un alemán actualmente consuma menos carne de vaca queun español antes de la crisis?c) A un curso de un mes de duración programado para el verano de 2001, piensan acudir unestudiante de cada uno de estos cuatro países. ¿Cuál es la probabilidad de que el consumo delos cuatro esté entre 9 y 10.4 kg. de carne de vaca?

SOLUCIÓN

Sean las variables:

X = “Consumo mensual en kg. por persona, de carne de vaca en Gran Bretaña antes de la crisis”.Y = “Consumo mensual en kg. por persona, de carne de vaca en Francia antes de la crisis”.Z = “Consumo mensual en kg. por persona, de carne de vaca en Alemania antes de la crisis”.T = “Consumo mensual en kg. por persona, de carne de vaca en España antes de la crisis”.

X → N(2.6, 0.2) Y → N(2.8, 0.3) Z → N(3.6, 0.6) T → N(2.5, 0.5)

a) ( ) ( ) ( ) 9772.020.2

2.630.2

2.6Xp3Xp2.1X0.7potipificand

==

<−

=<=<↓

φ

b) Debemos considerar la variable 0.85Z – T, que por ser una combinación lineal de variablesnormales independientes, sigue una distribución normal de parámetros:

0.85Z – T → N(0.85 x 3.6 – 2.5, 222 5.06.085.0 + ) ≡ N(0.56, 5101.0 )

( ) ( ) 2177.078.00.5101

0.560.5101

0.56TZ0.85p0TZ0.85p =−=

−<

−−=<− φ

c) Sea la variable S = 0.70 X + 0.9 Y + 0.85 Z + 0.92 T. Por ser combinación lineal de variablesaleatorias independientes y normales, sigue una distribución normal.

( ) 9.72.5x0.923.6x0.852.8x0.92.6x0.7SE =+++=

Page 73: ESTADÍSTICA EMPRESARIAL II PROBLEMAS DE EXAMEN …media0.webgarden.es/files/media0:4bd76ebb98146.pdf.upl... · 2010. 4. 27. · Estadística Empresarial II 23 La correlación negativa

Estadística Empresarial II 93

( ) 0.56420.25x0.920.36x0.850.09x0.90.04x0.7SVar 2222 =+++=

Entonces, ( )0.56429.7,NS →

( ) =

−<

−<

−=<<↓ 0.5642

9.710.40.5642

9.7S0.5642

9.79p5.10S9potipificand

( ) 0.647610.82382x193.02 =−=−φ=

48º) El diámetro de las piezas que fabrica una máquina es una variable aleatoria X, condistribución uniforme. Se sabe que el diámetro posible más pequeño es 50 y que el diámetromedio es 53 mm.a) Calcular la distribución de la v.a. X.b) Una pieza es apta para su venta si su diámetro está comprendido entre 51 y 55 mm.Calcular la probabilidad de que una pieza sea defectuosa.c) Si en un día se fabrican 10 piezas, calcular la probabilidad de que al menos 8 seancorrectas.

SOLUCIÓN

a) X → U(a, b), con a = 50. ( ) 532

baXE =+

= ; luego b = 56.

b) p(Defectuosa) = 1 - ( ) =−=<< ∫55

51dx

61155X51p

31

321

651551

6x1

55

51

=−=−

−=

−= .

c) Y = “Nº de piezas correctas en un día”. Y → B(10, 2/3)

( ) ( ) ( ) ( )10Yp9Yp8Yp8Yp =+=+==≥ =

=0101928

31

32

1010

31

32

910

31

32

810

+

+

= 2991413.032

3210

3245 10

10

10

9

10

8

=++ .

49º) En un determinado sector de empresas, consideramos las variables aleatorias, X = “Nº deempleados fijos” e Y = “Nº de empleados temporales”, con la siguiente ley de probabilidadconjunta:

X\Y 0 1 21 2/9 1/9 02 2/9 2/9 03 0 1/9 1/9

Page 74: ESTADÍSTICA EMPRESARIAL II PROBLEMAS DE EXAMEN …media0.webgarden.es/files/media0:4bd76ebb98146.pdf.upl... · 2010. 4. 27. · Estadística Empresarial II 23 La correlación negativa

94 Problemas de examen resueltos

a) ¿Son X e Y independientes?b) Calcular .1Y

3YXp

≤≤+

c) Calcular el coeficiente de correlación de X e Y.

SOLUCIÓN

X\Y 0 1 2 p(X=x)1 2/9 1/9 0 3/92 2/9 2/9 0 4/93 0 1/9 1/9 2/9

p(Y=y) 4/9 4/9 1/9

a) No son independientes puesto que p(X=x, Y=y) ≠ p(X=x). p(Y=y)

b)

≤≤+

13

YYXp =

( )=

≤≤≤+

)1(1;3

YpYYXp

==+=

==+==+==+==+===

)1()0()0,3()1,2()0,2()1,1()0,1(

YpYpYXpYXpYXpYXpYXp 7/8

c) 9

17923

942

931)( =++=XE

937

923

942

931)( 2222 =++=XE

8144

917

937)(

2

=

−=XVar

32

912

941

940)( =++=YE

98

912

941

940)( 2222 =++=YE

94

32

98)(

2

=

−=YVar

914

91.2.3

91.1.3

92.1.2

91.1.1)( =+++=XYE

278

32

917

914),( =−=YXCov ⇒ 603022.0

94

8144278

),( ==ρ YX .

Entre X e Y hay una relación directa.

50º) Un agente decide invertir en unos fondos con interés variable, sin riesgo. Elcomportamiento de las ganancias obtenidas (X) sigue una distribución con función dedensidad.

( ) ( )

<<−=

restoelen0

2x0x2x43

xfX

Page 75: ESTADÍSTICA EMPRESARIAL II PROBLEMAS DE EXAMEN …media0.webgarden.es/files/media0:4bd76ebb98146.pdf.upl... · 2010. 4. 27. · Estadística Empresarial II 23 La correlación negativa

Estadística Empresarial II 95

Esta ganancia es reinvertida en fondos con interés variable, en esta ocasión con riesgo depérdida. Dependiendo del valor de las ganancias dadas por X, las nuevas ganancias/pérdidas(Y) siguen una distribución con función de densidad:

( )yfxX

Y=

=

<<−

restoelen0

2xyx3x1

a) Indicar cuál sería la distribución de las ganancias X.b) Describir el comportamiento de la reinversión, en función de los posibles valores que tomay de su distribución (variable xX

Y= ).

c) Por término medio, si las ganancias han sido de 1 u.m., ¿cuánto cabe esperar que ganemostras reinvertirlas?d) Obtener la covarianza entre las dos variables, explicando el resultado. Para ello, tener encuenta que:

( )

( )

( )

<<−

<<−+

=

restoelen0

4y04y321

0y22y81

yf 2

2

Y

SOLUCIÓN

a) X → βe (2,2,0,2)

b) xXY

= → U(-x, 2x). Esto significa que podemos perder una cantidad x ó ganar una cantidad 2x,

teniendo en cuenta que la distribución es uniforme, es decir, para intervalos con la misma longitud,la probabilidad es la misma. En nuestro caso, la probabilidad de perder x ó de ganar x sería lamisma (estamos considerando los intervalos (-x,0) y (0,x).

c) xXY

= → U(-x, 2x). Por tanto, una de las formas de obtener esta esperanza es:

[ ]22

22

xxxbaxX

YE =+−

=+

== . En el caso concreto en que X = 1, el valor medio es 0.5.

d) [ ]XE = ( ) aabqp

p+−

+ = ( ) 002

222

+−+

= 1

[ ]YE = ( ) dyyy∫− +0

2

2281 + ( ) dyyy∫ −

4

0

24321 = dyyyy∫−

++0

2

23

844 + dyyyy∫

+−4

0

23

32168 =

= 0

2

234

23

448

1

++ yyy +

4

0

234

83

8432

1

+− yyy =

61

− + 32 =

21 .

Page 76: ESTADÍSTICA EMPRESARIAL II PROBLEMAS DE EXAMEN …media0.webgarden.es/files/media0:4bd76ebb98146.pdf.upl... · 2010. 4. 27. · Estadística Empresarial II 23 La correlación negativa

96 Problemas de examen resueltos

( )yxf XY , = ( ) ( )xfyf XxX

Y=

= ( )xxx

−243

31 =

<<−<<−

restoelen

xyxxx

0

2,204

2

[ ]XYE = dxdyxxyx

x∫ ∫−

2

0

2

42 = dxdyxxy

x

x∫ ∫−

−2

0

2 2

42 = dxdyyxx

x

x∫

−2

0

222

242 =

= dxdyxxx∫

−2

0

22

23

42 = dxdyxx∫

−2

0

43

836 =

2

0

54

85

34

6

− xx=

53 = 0.6.

Entonces, Cov(X,Y) = [ ]XYE - [ ]XE [ ]YE = 53 -1 x

21 =

101 . Si las ganancias en el primer caso son

elevadas, parece ser que las ganancias/pérdidas de la reinversión tienden a ser elevadas (signopositivo de la covarianza), aunque tendríamos que conocer el coeficiente de correlación para poderdar un grado de asociación entre las dos variables.

51º) Una empresa se dedica a la venta de un producto que ofrece al público en un envase demetal. Se sabe que el peso del contenido es una variable aleatoria, X, con distribución normalde parámetros 1000 y 10 gramos. Por otra parte, el peso del envase es otra v.a., Y, tambiénnormal e independiente de X, con parámetros 20 y 2 gramos.a) Calcular la distribución de la v.a. “peso total del producto”.b) Para que el producto cumpla con las especificaciones legales, su peso no debe ser menor de1005 gramos. Calcular (con dos decimales) la probabilidad de que el producto cumpla lasespecificaciones.c) Si tenemos 20 productos. Calcular la probabilidad de que, como mucho, 2 de ellos seandefectuosos.

SOLUCIÓN

a) X→ N(1000, 10); Y→ N(20, 2); X e Y independientes. Peso total = T = X + Y.T→ N(1020, 104 )

E(T) = E(X) + E(Y) = 1000 + 20 = 1020Var(T) = Var(X) + Var (Y) = 100 + 4 = 104; 104=Tσ

b) p (el producto cumpla las especificaciones) = p(T ≥ 1005) = 1- p(T < 1005) =

= 1 -

−<

−104

102010051041020Tp = 1- φ(-1.47) = 1- 0.07 = 0.93 (con dos decimales)

c) Sea Z = nº de productos defectuosos en los 20. Z → B(20, 0.07)

p (defectuoso) = 0.07

p(Z≤ 2) = p(Z = 0) + p(Z = 1) + p(Z = 2) = =

+

+

182191200 93.007.0220

93.007.0120

93.007.0020

= 0.8389971.

Page 77: ESTADÍSTICA EMPRESARIAL II PROBLEMAS DE EXAMEN …media0.webgarden.es/files/media0:4bd76ebb98146.pdf.upl... · 2010. 4. 27. · Estadística Empresarial II 23 La correlación negativa

Estadística Empresarial II 97

52º) Una compañía de alquiler de vehículos ha determinado que la probabilidad de que uncoche necesite una revisión en un mes es 0.2. La compañía tiene 1000 automóviles. Sisuponemos que la distribución del número de revisiones es una binomial:

a) Calcular la probabilidad aproximada de que el número de vehículos revisados estécomprendido entre 179 y 221. Comentar todos los pasos a seguir a la hora de buscar estaaproximación y el resultado compararlo con la probabilidad exacta que toma el valor 0.90303.

b) Teniendo en cuenta el número medio de vehículos revisados y la probabilidad obtenida enel apartado anterior, ¿cómo cabría esperar que fuese la probabilidad de que el número devehículos revisados fuese superior a 250? Razonar la respuesta, puesto que no hace faltarealizar ninguna operación adicional.

SOLUCIÓN

a) Para cada vehículo, definimos una variable de Bernoulli

=contariocaso0

revisadoesésimoivehículoel1Xi

Son variables B(1,p), donde p es la probabilidad de ser revisado, es decir, 0.2. La suma de estasvariables,∑ iX sigue una distribución binomial B(n,p). En este caso n = 1000 (n ≥ 30) y p = 0.2.Para calcular la probabilidad pedida, podemos aplicar el Teorema Central del Límite. Tan sólonecesitamos conocer la esperanza del total de vehículos revisados, es decir ∑ iX , y su varianza.

[ ]∑ iXE = np = 1000 x 0.2 = 200

[ ]∑ iXVar = npq = 1000 x 0.2 x 0.8 = 160

[ ]∑ << 221179 iXp Tipif=

−<

−<

− ∑160

200221160

200

160200179 iX

p... LCT

= [ ]66.166.1 <<− Zp =

= [ ]66.1<Zp - [ ]66.1−<Zp = 0.9515 – 0.0485 = 0.903. La aproximación es casi perfecta.

b) La probabilidad dentro del intervalo [179, 221] es elevada: [ ]∑ << 221179 iXp ≈ 0.9. Esto

indica que, para un número superior a 221, como es 250, la probabilidad [ ]∑ > 250iXp será muybaja.

53º) Una librería ha observado que la densidad conjunta de sus ventas mensuales en miles deeuros, correspondientes al Diccionario de la Real Academia en la edición de sencilla (X) y enla edición de lujo (Y), es de la forma:

( ) 6xy036yyx,fXY <<<=

Page 78: ESTADÍSTICA EMPRESARIAL II PROBLEMAS DE EXAMEN …media0.webgarden.es/files/media0:4bd76ebb98146.pdf.upl... · 2010. 4. 27. · Estadística Empresarial II 23 La correlación negativa

98 Problemas de examen resueltos

a) Calcular las marginales. ¿Alguna de ellas sigue una distribución de tipo conocido? ¿Sonindependientes las variables X e Y?b) Calcular las esperanzas y varianzas marginales y probar que Cov(X,Y) = 0.9.c) ¿Cuáles son las ventas esperadas en la edición de lujo, sabiendo que las ventascorrespondientes a la edición sencilla han sido 3000 €?d) Si el 40% de las ventas de la edición sencilla y el 20% de las ventas de la edición de lujo serealizan por internet, ¿cuál es la esperanza y varianza de las ventas realizadas por este medio?

SOLUCIÓN

a)

( ) 60727236

2

0

2

0<<=

== ∫ xxydyyxf

xx

X

( ) [ ] ( )60

366

363666

<<−

=== ∫ yyy

xydxyyf yyY

Las variables son dependientes porque ( ) ( ) ( )yfxfyx,f YXXY ≠

X → βe (3, 1, 0, 6) Y → βe (2, 2, 0, 6)

b) ( ) 5.4XE = ( ) 3YE =

( ) 1.352027XVar == ( ) 1.8

59YVar ==

( ) [ ] [ ]14.4

540x

dx108xdxy

108xdxdy

36xyXYE

60

56

0

4x0

36

0

6

0

x

0

2

===== ∫∫∫ ∫

Cov(X,Y) = E(XY) – E(X)E(Y) = 14.4 – 4.5x3 = 0.9

c) ( ) ( )( ) xy0

x2y

72x36y

xfyx,f

y/xf 22X

XYY/X <<===

Concretamente, si x = 3, la función de densidad condicionada es:

( ) 3y092y

x2yy/xf 2Y/X <<==

6

6

y=x

Page 79: ESTADÍSTICA EMPRESARIAL II PROBLEMAS DE EXAMEN …media0.webgarden.es/files/media0:4bd76ebb98146.pdf.upl... · 2010. 4. 27. · Estadística Empresarial II 23 La correlación negativa

Estadística Empresarial II 99

( ) [ ] 2y272dy

92y3Y/XE 3

033

0

2

==== ∫

por lo que se espera vender 2000 € por término medio de la edición de lujo, cuando se han vendido3000 € de la edición sencilla.

d) Z = “Ventas por internet” = 0.40 X + 0.20 Y

E(Z) = 0.40 E(X) + 0.20 E(Y) = 0.40 x 4.5 + 0.20 x 3 = 2.4

Var(Z) = 0.402 Var(X) + 0.202 Var(Y) + 2 x 0.20 x 0.40 Cov(X,Y) = 0.432

54º) Las distribuciones de las variables aleatorias, X e Y, nº de enciclopedias vendidas al díapor dos vendedores, en zonas independientes de una ciudad, son:

X 0 1 2 3 Y 0 1 2P(X = xi) 0.2 0.4 0.3 0.1 P(Y = yj) 0.5 0.3 0.2

a) Calcular la ley de probabilidad conjunta de X e Y.b) Calcular la probabilidad de que, entre los dos vendedores, en un día vendan más de 3enciclopedias.c) Si el primer vendedor vende sus enciclopedias a 300 € la unidad y el otro a 400 €, calcular lamedia y desviación típica de los ingresos obtenidos al día en total.

SOLUCIÓN

a) Por ser independientes: p (X = xi; Y = yj) = p(X = xi) . p/(Y = yj) ∀i, j

X \ Y 0 1 2 P(X = xi)0 0.10 0.06 0.04 0.21 0.20 0.12 0.08 0.42 0.15 0.09 0.06 0.33 0.05 0.03 0.02 0.1

P(Y = yj) 0.5 0.3 0.2 1

b) P(X + Y > 3) = p(X = 2; Y = 2) + p(X = 3; Y = 1) + p(X = 3; Y = 2) = 0.06 + 0.03 + 0.02 = 0.11

c) I = “Ingresos diarios” = 300 X + 400 Y

E ( X ) = 0. 0.2 + 1. 0.4 + 2. 0.3 + 3. 0.1 = 1.3

E ( X2 ) = 02. 0.2 + 12. 0.4 + 22. 0.3 + 32. 0.1 = 2.5

Var( X ) = E( X2) – (E( X ))2 = 2.5 – 1.32 = 0.81

E ( Y ) = 0. 0.5 + 1. 0.3 + 2. 0.2 = 0.7

E ( Y2 ) = 02. 0.5 + 12. 0.3 + 22. 0.2 = 1.1

Page 80: ESTADÍSTICA EMPRESARIAL II PROBLEMAS DE EXAMEN …media0.webgarden.es/files/media0:4bd76ebb98146.pdf.upl... · 2010. 4. 27. · Estadística Empresarial II 23 La correlación negativa

100 Problemas de examen resueltos

Var( Y ) = E( Y2) – (E( Y ))2 = 1.1 – 0.72 = 0.61

E ( I ) = 300.E ( X ) + 400.E ( Y ) = 300. 1.3 + 400. 0.7 = 670 €

Por ser X e e independientes, Cov (X, Y ) = 0, por tanto:

Var ( I ) = 3002.Var ( X ) + 4002.Var ( Y ) = 3002 .0.81 + 4002 . 0.61 = 170500

170500=Iσ = 412.91646

55º) Una fábrica de alfombras confecciona determinados modelos a medida. La superficie delas alfombras que los clientes demandan es una variable aleatoria que sigue una distribuciónuniforme entre 3 y 6 m2. El mes pasado fabricaron 81 alfombras.a) Si el precio de venta es de 12 € el m2. De alfombra, calcular la probabilidad de que el valortotal de las ventas del último mes supere los 4500 €.b) Se desea estudiar el tipo de alfombra que suele solicitar el cliente. Para ello se calcula lamedia aritmética de la superficie de las ochenta y una alfombras. ¿Con qué probabilidadestará comprendida entre 4.2 y 4.8 m2.?

SOLUCIÓN

a) T = “Cantidad de m2. de alfombra vendidos el mes pasado” = ∑=

81

1iiX .

Las variables aleatorias X1, X2, …, X81 son independientes idénticamente distribuidas según unaU(3, 6) y por lo tanto tienen esperanza y varianza finitas,

( ) ∞<=+

= 4.52

XE iba

( ) ( )∞<=

−= 0.75

12XVar

2

iab

por el Teorema Central del Límite, la suma de estas variables, tipificada, converge en ley a la N(0,1).

( ) ( ) 5.6434.5x81XEXETE81

1ii

81

1ii ===

= ∑∑

==

( ) ( ) 75.060.75x81XVarXVarTVar81

1ii.

81

1ii ===

= ∑∑

==indep

La probabilidad pedida es:

( ) ( ) =

−<

−−=>=>

60.755.643753

60.755.643T

p1375Tp450012Tpotipificand

( ) 0.08850.911511.35φ1.porelT.C.L

=−=−=

Page 81: ESTADÍSTICA EMPRESARIAL II PROBLEMAS DE EXAMEN …media0.webgarden.es/files/media0:4bd76ebb98146.pdf.upl... · 2010. 4. 27. · Estadística Empresarial II 23 La correlación negativa

Estadística Empresarial II 101

b) X = “Superficie de las alfombras” → U(3, 6)

Y = Superficie media = 81

XX

81

1ii∑

==

Las variables aleatorias X1, X2, …, X81 son independientes idénticamente distribuidas conesperanza y varianza finitas, luego por el T.C.L, la media aritmética de estas variables, tipificada,converge en ley a la N(0, 1).

( ) ( ) 4.5XEXE i == ( ) ( )108

1nXVar

XVar i ==

( ) ( )10,N1084.5X L→−

( ) ( ) ( ) ( )( ) =−<−<−=<< 1084.54.81084.5X1084.54.2p4.8X4.2potipificand

( ) ( ) ( ) 0.998210.9991x213.12φ23.12φ3.12φ.porelT.C.L

=−=−=−−= .

56º) En una fábrica donde se hacen cajas de cartón, tenemos en cuenta las variables:X = “cantidad de cartón fabricada diariamente (en kg.)” e Y = “cantidad de cartón que, alfinal del día, se ha encontrado en mal estado (en kg.)” (deteriorado por la fabricación, eltransporte, la humedad, etc.). Sabemos que siguen una distribución normal, con medias 1500,20, respectivamente y varianzas, 24 y 10, respectivamente.a) Supongamos que el control de calidad exige que, cada día, la cantidad de cartón en malestado no supere los 22 kg. Consideremos un periodo de un mes (30 días). Calcular laprobabilidad de que el número de días en los que la cantidad de cartón defectuoso no superelos 22 kg. sea inferior a 2.b) Supongamos que Cov(X,Y) = 11. Si T es la variable “cantidad de cartón que al final de lajornada se encuentran en perfecto estado (en kg.)”, con distribución normal, obtener suesperanza y su varianza.

SOLUCIÓN

a) Sabemos que X → N(1500, 24 ) y que Y → N(20, 10 ).

Entonces: p[Y < 22] = p[Z < 10

2022 − = 0.63] = 0.7357, siguiendo Z una distribución normal

tipificada. Ésta sería la probabilidad de nuestro “éxito”. En 30 días, el número de días en los que severifica el éxito sigue una distribución binomial:

N = “nº de días en un mes en los que la cantidad de cartón defectuosa no supera los22 kg.” → B(30, 0.7357).

La probabilidad que nos piden es: p[N < 2] = p[N ≤ 1] = p[N = 0] + p[N = 1] =

Page 82: ESTADÍSTICA EMPRESARIAL II PROBLEMAS DE EXAMEN …media0.webgarden.es/files/media0:4bd76ebb98146.pdf.upl... · 2010. 4. 27. · Estadística Empresarial II 23 La correlación negativa

102 Problemas de examen resueltos

= 73570x26430130

73570x264300

30 29030 ....

+

≈ 0.

a) Cov(X,Y) = 11.

T = X – Y ⇒ E[T] = E[X – Y] = E[X] – E[Y] = 1500 – 20 = 1480.

Var[T] = Var[X – Y] = Var[X] + Var[Y] – 2 Cov(X,Y) = 24 + 10 – 2x11 = 12.

57º) Sea (X, Y) una v.a. bidimensional con función de densidad conjunta;

<<<<

=resto el en0

2y01;x0yxky)(x,f2

YX,

a) Obtener el valor de k, y comprobar si estas variables son o no independientes. ¿Quédistribución sigue cada una de las variables?b) Obtener la función de densidad de Z = 3Y + 2. ¿Sigue algún tipo de distribución conocida?c) Calcular Var(3X – 2Y).

SOLUCIÓN

a) Para que XYf sea función de densidad debe cumplir que:

1dydxy)(x,f YX, =∫ ∫∞

∞−

∞−

por tanto:

1k32

3x2kdx2xkdx

2yxkdxdyykx

1

0

31

0

21

0

2

0

22

1

0

2

0

2 ==

==

=

∫∫∫ ∫

de donde 23k =

Calculamos las marginales de X e Y:

1x03x2

yx23dyyx

23y)dy(x,f(x)f 2

2

0

22

2

0

2YX,X <<=

=== ∫∫

∞−

2y02y

3xy

23dyyx

23y)dx(x,f(y)f

1

0

31

0

2YX,Y <<=

=== ∫∫

∞−

La distribución de X puede comprobarse que es β(3,1)

La distribución de Y es una βe(2,1,0,2)

Las variables son independientes, puesto que ( ) ( ) ( )yfxfyx, YXXY =f , para todo valor de x e y.

Page 83: ESTADÍSTICA EMPRESARIAL II PROBLEMAS DE EXAMEN …media0.webgarden.es/files/media0:4bd76ebb98146.pdf.upl... · 2010. 4. 27. · Estadística Empresarial II 23 La correlación negativa

Estadística Empresarial II 103

b) Sea Z = 3Y + 2

c)

=

≤=≤+=≤=3

2zF3

2zYpz)2p(3Yz)p(Z(z)F YZ

<

−<

−=

==resto elen 0

23

2z018

2z

31

32zf

dz(z)dF(z)f Y

ZZ , esto es:

<<

−=

resto elen 0

8z218

2z(z)fZ , es decir, sea una βe(2,1,2,8)

d) Var(3X – 2Y) por ser X e Y independientes es: Var(3X – 2Y) = 32 Var(X) + 22 Var(Y)

Calculamos las varianzas de X e Y.

Puesto que conocemos su distribución: 803

q)1)(pq(ppqVar(X) 2 =+++

= ;

92

q)1)(pq(ppqa)(bVar(Y) 2

2

=+++

−= , de donde

7208832Y)Var(3X =− .

58º) Se consideran las variables X = “Tiempo de espera de conexión a la red”, expresada enminutos, e Y = “Tiempo que se permanece conectado”, también en minutos. Tras observar sucomportamiento durante cierto periodo de tiempo, se llega a la conclusión de que sudistribución conjunta es:

X \ Y 60 120 180 1 0.04 0.08 0.30 5 0.02 0.20 0.19 30 0.10 0.02 0.05

a) Calcular la probabilidad de que un usuario esté frente al ordenador dos horas y media ómás, desde que se intenta conectar hasta que apaga el ordenador.b) ¿Son independientes las variables? Caso contrario, calcular la covarianza e comentar susignificado.c) Suponiendo que el tiempo de conexión ha sido de 30 minutos, ni más ni menos, indicar, portérmino medio, cuál sería el tiempo que se ha estado conectado.

SOLUCIÓN

a) Definimos la variable X + Y = “Tiempo que un usuario está frente al ordenador, desde que seintenta conectar hasta que desconecta”. Esta variable viene expresada en minutos. Entonces:

[ ]150YXp ≥+ = [ ]130Y30,Xp == + [ ]180Y1,Xp == + [ ]180Y5,Xp == ++ [ ]180Y30,Xp == = 0.02 + 0.30 + 0.19 + 0.05 = 0.56.

Page 84: ESTADÍSTICA EMPRESARIAL II PROBLEMAS DE EXAMEN …media0.webgarden.es/files/media0:4bd76ebb98146.pdf.upl... · 2010. 4. 27. · Estadística Empresarial II 23 La correlación negativa

104 Problemas de examen resueltos

b) [ ]601 == Y,Xp = 0.04 ≠ [ ]1=Xp [ ]60=Yp = 0.42 x 0.16 = 0.0672.

X \ Y 60 120 180 [ ]xXp = 1 0.04 0.08 0.30 0.42 5 0.02 0.20 0.19 0.41 30 0.10 0.02 0.05 0.17[ ]yYp = 0.16 0.30 0.54 1

E[X] = [ ]∑ = xXpx = 7.57 E[Y] = [ ]∑ = yYpy = 142.8

E[XY] = [ ]∑∑ ==i j

jiji y,YxXpyx = 885

⇒ Cov(X,Y) = E[XY] – E[X] E[Y] = 885 – 7.57 x 142.8 = -195.996. La relación entre las variableses inversa. Cuanto más tardemos en conectarnos, más pronto lo dejaremos.

c) La distribución condicionada 30=XY :

30=XY

==

30XyYp

60 10/17120 2/17180 5/17

1

[ ]30=XYE =

175180

172120

171060 ++ =

171740 = 102.3529

59º) Un sistema electrónico compuesto por 30 componentes independientes actúa de la formasiguiente: al principio sólo funciona el primer componente y cuando éste falla, empieza afuncionar automáticamente el segundo y así sucesivamente hasta el último, de forma que elsistema sólo deja de funcionar cuando han fallado los 30 componentes de forma sucesiva.Sabiendo que la duración (en horas) de cada componente es una v. a. con distribuciónexponencial y tal que p( X ≤ 1) = 0.6321205:

a) Comprobar que el valor del parámetro a es 1.b) Calcular la duración media del sistema.c) Calcular de forma exacta (plantear) y aproximada la probabilidad de que la duración delsistema sea mayor que 31 horas.

SOLUCIÓN

a) Sea X = “Duración (en horas) de un componente”. Sabemos que X sigue una distribución ε(a),por tanto:

<≥

=−

000

xxae

fax

)x(X

Page 85: ESTADÍSTICA EMPRESARIAL II PROBLEMAS DE EXAMEN …media0.webgarden.es/files/media0:4bd76ebb98146.pdf.upl... · 2010. 4. 27. · Estadística Empresarial II 23 La correlación negativa

Estadística Empresarial II 105

Luego: [ ] ..eedxae)X(p aaxax 6321205011 10

1

0

=−=−==≤ −−−∫

Despejando a = -ln(0.3678795) = 1.

b) D = “Duración total del sistema” = X1 + X2 +…+ X30. Puesto que 11==

a)X(E

E(D) = E(X1) + E(X2) +…+ E(X30) = 30 x 1 = 30

Hallamos la distribución de D. Puesto que X sigue una distribución ε(a), es decir una γ(a,1), por lapropiedades de la distribución gamma:

),()...,(XDi

i 301 unadecir es111130

1γγ +++↔=∑

=

por tanto, de forma exacta:

∫∞

−−

Γ=>

31

13030

30131 dxex

)()D(p x

Como hay que calcular esta integral, podemos hacerlo de forma aproximada utilizando el TeoremaCentral del Límite, puesto que:

))n,n(NXDi

i 30 N(30, caso esteen 30

1σµ≈= ∑

=

de donde:

4286057140118011825030

303130

3031 ..).().Z(pDp)D(p =−=Φ−=>≈

−>

−=>

60º) La cantidad diaria de leche que se sirve en una cafetería se comporta con independencia alo largo de los distintos días del año. De lunes a viernes sigue una distribución normal demedia 50 litros y desviación típica 10 litros. Los sábados y domingos, dicha cantidad seincrementa en un 40%.a) ¿Qué distribución sigue la cantidad de leche consumida en un día de fin de semana?b) ¿Qué distribución sigue la cantidad de leche que se sirve semanalmente?. Calcular laprobabilidad de que en una semana se utilicen más de 450 litros de leche.c) Teniendo en cuenta que un año tiene 52 semanas, calcular la probabilidad de que al menosen una semana al año, la cantidad de leche consumida en esa cafetería sea superior a 450litros.

SOLUCIÓN

a) X = “Cantidad de leche consumida los días de diario” → N(50, 10) Y = “Cantidad de leche consumida los días de fin de semana” = 1.4 X → N(70, 14)pues teniendo en cuenta las propiedades de la distribución normal,

Page 86: ESTADÍSTICA EMPRESARIAL II PROBLEMAS DE EXAMEN …media0.webgarden.es/files/media0:4bd76ebb98146.pdf.upl... · 2010. 4. 27. · Estadística Empresarial II 23 La correlación negativa

106 Problemas de examen resueltos

si X → N(µ, σ) ⇒ Y = bX→ N(bµ, bσ)

b) Z = “Cantidad de leche consumida semanalmente” = ∑∑==

+2

1

5

1 ii

ii YX . Por ser suma de normales

independientes, Z también sigue una distribución normal, con los siguientes parámetros:

( ) ( ) ( ) 3907025052

1

5

1=+=+= ∑∑

==

xxYEXEZEi

ii

i

( ) ( ) ( ) 892196210052

1

5

1=+=+= ∑∑

==

xxYVarXVarZVari

ii

i

Z → N(390, 892 )

( ) ( ) 0228097720121892

3904501450 ..Zp =−=Φ−=

−Φ−=>

c) T = “Nº de semanas al año con consumo de leche superior a 450 litro” →B(52, 0.0228)

( ) ( ) =

−==−=≥ 520 9772002280

052

1011 ..TpTp 1 – 0.3013976 = 0.6986024

61º) Contestar las siguientes cuestiones de forma razonada (incluso si la decisión ha sidoaleatoria):a) Para explicar el comportamiento estadístico del tiempo que se tarda en realizar unaoperación bancaria (evaluada en días) se puede optar entre un modelo gamma o unadistribución beta extendida. Indicar cuál de los dos modelos sería el más acertado en el casoen el que el tiempo máximo de la operación fuera de 12 días.b) Sea X una v.a. con distribución γ(5,1) e Y una v.a. con distribución T(1,2,4). Si la varianzade X+Y es 450103 , ¿podemos afirmar que las variables son independientes?c) Siguiendo un plan de calidad, los auditores de una empresa analizan durante cinco días laexistencia de incidencias y si es posible una solución de manera efectiva. Suponiendo que elnúmero de incidencias diario es una v.a. de Poisson de parámetro 0,45, calcular laprobabilidad de que el número total de incidencias durante los cinco días sea superior a 2 einferior a 5.

SOLUCIÓN

a) En el caso de tener extremos finitos, sería más adecuado utilizar una beta extendida.

b) Var(X) = 2ap =

251 ; Var(Y) =

18a)m)(m(ba)(b 2 −−−− =

187 .

Var(X+Y) = Var(Y) + Var(Y) + 2 Cov(X,Y) = 251 +

187 + 2 Cov(X,Y) =

450103 ⇒

Page 87: ESTADÍSTICA EMPRESARIAL II PROBLEMAS DE EXAMEN …media0.webgarden.es/files/media0:4bd76ebb98146.pdf.upl... · 2010. 4. 27. · Estadística Empresarial II 23 La correlación negativa

Estadística Empresarial II 107

Cov(X,Y) = -0.1 ≠ 0 ⇒ existe cierto grado de dependencia (lineal, al menos).

c) Xi → ( )45.0Ρ ⇒ Y = ∑=

5

1iiX → ( )25.2Ρ

p(2 < Y < 5) = p(Y = 3) + p(Y = 4) = 2.254

2.253

e4!

2.25e3!

2.25 −− + = 0.312646625

62º) Sea X una v.a. con distribución β(4,1) e Y otra v.a. con función de densidad igual a:

<<−=

restoelen0

1y0)3y(134

(y)Yf

Calcular:a) La esperanza y la varianza de X e Y.b) Sabiendo que la Cov(X,Y) = 1/75, la esperanza y la varianza de 4X – 8Y.

SOLUCIÓN

a) E(X) = qp

p+

= 14

4+

= 54 .

Var(X) = 2q)1)(pq(ppq

+++= 21)1)(41(4

4x1+++

= 752 ;

E(Y) = ∫=

=

−1y

0y

3 )dyy(134y =

1y

0y

52 y154y

32 =

=

− =

154

32− =

52 .

E(Y2) = ∫=

=

−1y

0y

32 )dyy(134y =

1y

0y

63 y92y

94 =

=

− =

92

94− =

92 .

Var(Y) = E(Y2) - E(Y) = 92 -

2

52

=

22514 ;

b) E(4X – 8Y) = 4E(X) – 8E(Y) = 544x -

528x = 0.

Var(4X – 8Y) = 42Var(X) + 82Var(Y) – 2x4x8Cov(X,Y) = 75216x +

2251464 -

75164 =

932

Page 88: ESTADÍSTICA EMPRESARIAL II PROBLEMAS DE EXAMEN …media0.webgarden.es/files/media0:4bd76ebb98146.pdf.upl... · 2010. 4. 27. · Estadística Empresarial II 23 La correlación negativa

108 Problemas de examen resueltos

63º) La normativa de contratos de obra en un determinado sector exige que se concedanmediante una subasta. El presupuesto (en millones de €) que presenta cada empresa se puedeconsiderar, desde un punto de vista probabilístico, que es una variable aleatoria condistribución N(1,23; 0,1).

a) Obtener, en porcentaje, el número de empresas que probablemente presenten unpresupuesto inferior a 1 millón de €.b) Supongamos que antes de iniciada la obra, de forma inesperada y, por tanto,independientemente del presupuesto inicial, se detecta que se tendrá que hacer frente a unoscostes adicionales, que también se distribuyen normalmente. La previsión es que, por términomedio, este coste adicional sea de 100.000 €, con una desviación típica de 0,24 millones de €.Obtener la distribución del coste final de la obra y calcular la probabilidad de que dicho costesea inferior a 2 millones de €.

SOLUCIÓN

a) Sea X =” Presupuesto que presenta una empresa (millones de €)”→ N(1.23,0.1).

( )1Xp < =

<−

0.11.231

0.11.23Xp = ( )2.3Zp −< = )3.2(−Φ = 0.0107 ≈ 1% de empresas.

b) Sean Y = “Costes adicionales” → N(0.1, 24.0 ) y C = “Coste final de la obra” = X + Y. ComoX e Y están normalmente distribuidas su suma también es una normal, veamos de que parámetros.

E(C) = E(X+Y) = E(X) + E(Y) = 1.23 + 0.1 = 1.33.

Var (C) = Var(X+Y) = Var(X) + Var(Y) = 0.01 + 0.24 = 0.25 ⇒ 0.5σ C = .

X e Y independientes

⇒ C → N(1.33,0.5) ⇒ ( )2Cp < =

<0.51.332Zp = )34.1(Φ = 0.9099.

tipificando

64º) En la elaboración de un determinado tipo de galleta, una fábrica utiliza, entre otros, doscomponentes (A y B). Si definimos las variables X = “Nº de gramos del producto A por cadagalleta” (no excede de 1 gramo) e Y = “Nº de gramos del producto B por cada galleta” (noexcede de 10 gramos), con función de densidad conjunta:

( ) yx251yx,f = , cuando 0 < x < 1; 0 < y < 10.

a) Obtener las distribuciones marginales de cada una de las variables. Indicar si pertenece aalguno de los modelos conocidos.b) Calcular el número medio de gramos de cada ingrediente en cada galleta.

Page 89: ESTADÍSTICA EMPRESARIAL II PROBLEMAS DE EXAMEN …media0.webgarden.es/files/media0:4bd76ebb98146.pdf.upl... · 2010. 4. 27. · Estadística Empresarial II 23 La correlación negativa

Estadística Empresarial II 109

c) ¿Son independientes? Obtener el valor de E(XY).d) Si un paquete de galletas se compone de 20 unidades, indicar con qué probabilidad elnúmero de galletas que contienen menos de 2 gramos del producto B en su composición es 1 alo sumo.

SOLUCIÓN

a) ( )xfX = ∫=

=

10y

0y

dyxy251 =

10y

0y

2

2yx

251

=

=

= 2x 0 < x < 1 ⇒ X → β(2,1).

( )yfY = ∫=

=

1x

0x

dxyx251 =

1x

0x

2

2xy

251

=

=

= 10y0y,

501

<< ⇒ Y → βe(2,1,0,10).

b) ( )qp

pXE+

= = 32

( ) ( ) aabqp

pYE +−+

= = 320

c) Claramente, las variables son independientes pues:

( ) yx251yx,f YX, = = 2x y

501 = ( )xfX ( )yfY .

Como son independientes, la covarianza es cero ⇒ E(XY) = E(X) E(Y) = 32

320 =

940 .

d) S = “número de galletas con menos de 2 gramos en su composición en un paquete de 20”.

S seguirá una distribución B(20, [ ]2Yp < ).

[ ]2Yp < = ∫=

=

2y

0y

ydy501 =

2y

0y

2

100y

=

=

=

1004 = 0.04 ⇒ S → B(20,0.04).

[ ]1Sp ≤ = [ ]0Sp = + [ ]1Sp = = 200 96.004.0020

+ 191 96.004.0

120

= 0.810337795.

Page 90: ESTADÍSTICA EMPRESARIAL II PROBLEMAS DE EXAMEN …media0.webgarden.es/files/media0:4bd76ebb98146.pdf.upl... · 2010. 4. 27. · Estadística Empresarial II 23 La correlación negativa

110 Problemas de examen resueltos

65º) El tiempo, en horas, que tarda un empleado en completar un expediente, sigue unadistribución gamma de media 2 horas y varianza igual a 1. Si dicho empleado realiza 50expedientes al mes:

a) Calcular la probabilidad aproximada de que el tiempo total que emplea sea mayor de 95horas. Indicar asimismo como se calcularía esta probabilidad de forma exacta.b) Si por cada expediente el empleado recibe 12 € fijos, más un plus de 10 € por horatrabajada, calcular la media y la desviación típica de la v.a. ingresos mensuales.

SOLUCIÓN

a) Sea la v.a. T = tiempo (en horas) en completar un expediente.

T → γ(a,p), tal que 2apE(T) == y 1

apVar(T) 2 == de donde a = 2 y p =4

Sea X = tiempo total en completar 50 expedientes = T1 + T2 +…+ T50.

Su distribución exacta por ser suma de variables aleatorias independientes con distribución gamma,con el mismo parámetro a, será γ(2, 200) y por tanto:

∫∞=

=

−=>x

95x

2x199200

dxex(200)Γ295)p(X que manualmente es casi imposible de resolver

La distribución aproximada de T por ser suma de v.a independientes, utilizando el Teorema Centraldel Límite es N(50x2, 1x 50 ) es decir )50N(100, , por tanto

0.76110.238910.71)(10.707)p(Z5010095

50100Xp95)p(X Z =−=−Φ−=−>≈

−>

−=>

tipificando Aplicando el Teorema Central del Límite

b) Sea G= ingresos mensuales (por los 50 expedientes)

G = 12x50 + 10 (T1 + T2 +…+ T50) = 600 + 10 X

E(G) = 600 + 10E(X) = 600 + 10x100 = 1 600

Var(G) = Var(10X) = 102Var(X) = 100x50 = 5 000 ⇒ 5000σ G =

Page 91: ESTADÍSTICA EMPRESARIAL II PROBLEMAS DE EXAMEN …media0.webgarden.es/files/media0:4bd76ebb98146.pdf.upl... · 2010. 4. 27. · Estadística Empresarial II 23 La correlación negativa

Estadística Empresarial II 111

66º) Una empresa comercializa un producto en lotes de 10 unidades que se embalan en unacaja de madera. Se sabe que el peso de cada unidad es una variable aleatoria con distribuciónnormal de media 90 gramos y varianza 4. Además el peso de la caja de madera, en gramos, esotra variable aleatoria con distribución N(300, 20).

a) Si cada unidad, según su etiqueta, debe pesar al menos 92 gramos, calcular la probabilidad de queuna unidad no cumpla lo estipulado.b) En un lote, ¿cuál es la probabilidad de que todas las unidades del mismo no cumplan loindicado en su etiqueta?c) Calcular la distribución de la v.a. peso total del lote embalado y la probabilidad de quedicho peso sea mayor que 1.250 gramos.

SOLUCIÓN

a) Sea X la v.a. peso en gramos de una unidad, puesto que Var(X) = 4, X → N(90,2)

0.8413(1)1)p(Z2

90922

90Xp92)p(X Z =Φ=<=

<−

=<

siendo la distribución de Z una N(0,1).

b) Sea T = número de unidades que no cumplen lo estipulado en las 10 unidades de un lote.

Llamando Éxito = una unidad no cumple lo estipulado, la v.a. T sigue una distribución binomial deparámetros n = 10 y p = p(X < 92) = 0.8413. Nos piden:

0.17760.84130.15870.841310

1010)p(T 10010 ==

==

c) Sea Y = peso de la caja de madera (en gramos) → N(300, 20) y sea P = peso total de una loteembalado. Podemos escribir esta variable como:

YX...XXP 1021 ++++=

y puesto que:

200 130010x90E(Y))E(X...)E(X)E(XE(P) 1021 =+=++++=

44040010x4Var(Y))Var(X...)Var(X)Var(XVar(P) 1021 =+=++++=

La distribución de P → )440200, N(1

Por tanto:

0.00870.9913-1(2.38)12.38)p(Z440

200 1250 1440

200 1Pp250) 1p(P Z ==Φ−=>=

−>

−=> .

Page 92: ESTADÍSTICA EMPRESARIAL II PROBLEMAS DE EXAMEN …media0.webgarden.es/files/media0:4bd76ebb98146.pdf.upl... · 2010. 4. 27. · Estadística Empresarial II 23 La correlación negativa

112 Problemas de examen resueltos

67º) Sea la variable aleatoria bidimensional (X,Y), de la que poseemos los siguientes datos desu ley de probabilidad conjunta:

X\Y 1 2 3 p(X=xi)0 0 a b 3/241 4/24 c d 9/242 e f 0 9/243 g 0 0 3/24

p(Y=yj) 8/24 12/24 4/24

a) Sabiendo que 412)Yp(X =≤+ comprobar que p(X=0, Y=2) = 1/12.

b) Completar la tabla.c) Calcular la esperanza y la varianza de 2X - 3Y.d) Calcular ( )2Y

XE = .

SOLUCIÓN

a) 1/44/24a01)Y1,p(X2)Y0,p(X1)Y0,p(X2)Yp(X =++===+==+===≤+

de donde a = 2/24.

b) La tabla de la ley de probabilidades conjunta es:

X\Y 1 2 3 p(X = xi)0 0 2/24 b 3/241 4/24 c d 9/242 e f 0 9/243 g 0 0 3/24

p(Y = yj) 8/24 12/24 4/24

sabiendo como se calculan las distribuciones marginales a partir de la conjunta es sencillocompletar la tabla. Por ejemplo de la primera fila:

0 + 2/24 + b = 3/24 de donde b = 1/24

La tabla completa es la siguiente:

X\Y 1 2 3 p(X = xi)0 0 2/24 1/24 3/241 4/24 2/24 3/24 9/242 1/24 8/24 0 9/243 3/24 0 0 3/24

p(Y = yj) 8/24 12/24 4/24

Page 93: ESTADÍSTICA EMPRESARIAL II PROBLEMAS DE EXAMEN …media0.webgarden.es/files/media0:4bd76ebb98146.pdf.upl... · 2010. 4. 27. · Estadística Empresarial II 23 La correlación negativa

Estadística Empresarial II 113

c) Calculamos las esperanzas y varianzas de X e Y y la covarianza entre ambas

23

243 3

249 2

249 1

243 0E(X) =+++=

3243 3

249 2

249 1

243 0)E(X 22222 =+++=

43

233E(X))E(XVar(X)

222 =

−=−=

611

244 3

2412 2

248 1E(Y) =++=

623

244 3

2412 2

248 1)E(Y 2222 =++=

3617

611

623E(Y))E(YVar(Y)

222 =

−=−=

25

243 3

248 4

241 2

243 3

242 2

244 1E(XY) =+++++=

41

611

23

25E(X)E(Y)E(XY)Y)Cov(X, −=−=−=

25

611 3

23 23E(Y)2E(X)3Y)E(2X −=−=−=−

441

41 12

3617 9

43 4Y)X,2x2x3xCov(Var(Y)3Var(X)23Y)Var(2X 22 =++=−+=−

d) Calculamos la ley de probabilidad de la variable condicionada X/Y=2.

2)p(Y2)Y,xp(X

2YxXp ii

===

=

==

Page 94: ESTADÍSTICA EMPRESARIAL II PROBLEMAS DE EXAMEN …media0.webgarden.es/files/media0:4bd76ebb98146.pdf.upl... · 2010. 4. 27. · Estadística Empresarial II 23 La correlación negativa

114 Problemas de examen resueltos

de donde se obtiene:

2YX

= 0 1 2

==

2YxXp i 1/6 1/6 2/3

(La probabilidad para X = 3 es cero)

( )23

32 2

61 1

61 02Y

xXpx2YXE

ix

ii =++=

==== ∑

68º) Sea (X, Y) una variable aleatoria bidimensional con función de densidad conjunta:

f(x,y) = 10x2y 0 < y < x < 1

Calcular:a) Las funciones de densidad marginal de X e Y, ¿son independientes?b) La covarianza entre X e Y.c) La función de densidad de Y/X=1/2. ¿Qué distribución sigue?d) La esperanza de Y/X=1/2.

SOLUCIÓN

Sea la función de densidad conjunta f(x,y) = 10x2y 0 < y < x < 1

a) Función de densidad marginal de X

fX(x) = ( ] 4xy0y

22xy

0y

2 x5y5xdyy10x ===

=

=

=∫ 0 < x < 1

luego

<<

=resto elen 0

1x0x5(x)f

4

X

Función de densidad marginal de Y

fY(y) = )y - y(13

10yx3

10dxy10x 31x

yx

31x

yx

2 =

=

=

=

=

=∫ 0 < y < 1

X

Y

1

Y = X1

Page 95: ESTADÍSTICA EMPRESARIAL II PROBLEMAS DE EXAMEN …media0.webgarden.es/files/media0:4bd76ebb98146.pdf.upl... · 2010. 4. 27. · Estadística Empresarial II 23 La correlación negativa

Estadística Empresarial II 115

luego

<<=

resto elen 0

1y0)y - y(13

10

(y)f3

Y

Dos variables son independientes ⇔ fXY(x,y) = fX(x) fY(y) ∀ (x,y). Veamos si se verifica laigualdad en este caso.

Como fX(x) fY(y) = )y - y(13

105x 34 ≠ y10x2 = fXY(x,y) ⇒

⇒ X e Y son estadísticamente dependientes.

b) X → β(5,1) pues

<<

=<<−

β=

−−

resto0

1x0x5

resto0

1x0x)(1xq)(p,

1

(x)f4

1q1p

X

ya que

50!4!

5!(1) (5))1(5

(5,1)1

==ΓΓ+Γ

luego

E(X) = qp

p+

= 15

5+

= 65 .

Var(X) = 2q)1)(pq(ppq

+++= 21)1)(51(5

5x1+++

= 2525 ;

E(Y) = ∫=

=

−1y

0y

3 )dyyy(13

10y = 1y

0y

63

6y

3y

310

=

=

− =

95 .

E(Y2) = ∫=

=

−1y

0y

32 )dyyy(13

10y = 1y

0y

74

7y

4y

310

=

=

− =

145 .

Page 96: ESTADÍSTICA EMPRESARIAL II PROBLEMAS DE EXAMEN …media0.webgarden.es/files/media0:4bd76ebb98146.pdf.upl... · 2010. 4. 27. · Estadística Empresarial II 23 La correlación negativa

116 Problemas de examen resueltos

Var(Y) = E(Y2) - E(Y) = 145 -

2

95

=

134 155 ;

E(XY) = =

=

∫ ∫∫ ∫=

=

=

=

∞=

∞=

∞=

−∞=

1x

0x

y

0y

23x

- x

y

yYX, dxdyyx10dxdyy)(x,fyx

x

2110x

2110dxx

310dx

3y10x

1x

0x

71x

0x

61x

0x

y

0y

33 =

==

=

=

=

=

=

=

=

=

=∫∫

x

Cov (X,Y) = E(XY) – E(X)E(Y) = 2110 –

95x

65 =

3785

c) Función de densidad condicionada de Y por X.

( ) 24

2

X

YX,xY/X x

y 2 x5

yx10(x)f

y)(x,fxy/Xf ===== 0 < y < x

siempre que 0 < x < 1.

En particular, para X = 1/2, tenemos:

( ) ( ) y82

1y 21/2y/Xf 21/2Y/X ==== 0 < y < 1/2

Veamos que Y/X=1/2 sigue una distribución βe(2,1,0,1/2)

<<

=<<−

==

−−+

=resto elen 0

1/2y0y8

resto elen 0

byay)(ba)-(ya)-(b1

q)β(p,1

1/2)(y/Xf

1q1p1-qp

1/2Y/X

ya que

840!1!

2!4(1)(2))1(2

)02/1(1

(2,1)1

112 ==ΓΓ+Γ

=−β −+

d) E(Y/X = 1/2) = ∫=

=

1/2y

0y

dyy 8y = ( ] 1/2y0y

3y38 =

= = 31 .

Page 97: ESTADÍSTICA EMPRESARIAL II PROBLEMAS DE EXAMEN …media0.webgarden.es/files/media0:4bd76ebb98146.pdf.upl... · 2010. 4. 27. · Estadística Empresarial II 23 La correlación negativa

Estadística Empresarial II 117

69º) El número de servicios que realiza un chófer de una empresa al día (X) tiene la siguienteley de probabilidad

X 7 8 9 10P(X = x) 0,2 0,3 0,4 0,1

a) Calcular la probabilidad de que en un trimestre (81 días) el número total de serviciosrealizados (T) esté comprendido entre 650 y 700.b) Sabiendo que por cada servicio cobra 9,25 €, que el número de kilómetros recorridos porservicio es de 11,5, que cada kilómetro recorrido tiene un coste en combustible de 0,07 € y queal mes tiene unos gastos fijos de 700 €, calcular la probabilidad de que la ganancia del chóferen un trimestre sea mayor que 3.750 €.

SOLUCIÓN

a) Sean X = número de servicios que realiza el chofer de una empresa al día, y

T = número de servicios que realiza el chofer de una empresa al trimestre = ∑=

81

1iiX

Hay que calcular p(650 < T < 700), pero como no conocemos la distribución de T y es una suma devariables aleatorias independientes idénticamente distribuidas con esperanza y varianza finitas, laprobabilidad pedida la podemos aproximar aplicando el Teorema Central del Límite.

E(X) = 7x0.2 + 8x0.3 + 9x0.4 + 10x0.1 = 8.4

E(X2) = 72x0.2 + 82x0.3 + 92x0.4 + 102x0.1 = 71.4

Var(X) = E(X2) – E(X) = 71.4 – 8.42 = 0.84

E(T) = ( ) ∑∑∑===

==

81

1i

81

1ii

81

1ii 8.4XEXE = 81 x 8.4 = 680.4

Var(T) = ( ) ∑∑∑===

==

81

1i

81

1ii

81

1ii 0.84XVarXVar = 81 x 0.84 = 68.04 ⇒

por ser Xi independientes

⇒∑

σ=

81

1iiX = 8.248636251

<−

<−

=

<<

∑∑ =

= 248636251.84.068700

248636251.8

4.068X

248636251.84.068650p700X650p

81

1ii81

1ii

tipificando Aplicando el Teorema Central del Límite

Page 98: ESTADÍSTICA EMPRESARIAL II PROBLEMAS DE EXAMEN …media0.webgarden.es/files/media0:4bd76ebb98146.pdf.upl... · 2010. 4. 27. · Estadística Empresarial II 23 La correlación negativa

118 Problemas de examen resueltos

)69.3((2.38) −Φ−Φ≈ = 0.9913 – 0.0001 = 0.9912

b) G = Ganancia = 100 2 X0.07 x 11.5X25.981

1ii

81

1ii −− ∑∑

==

= 8.445T – 2 100

E(G) = E(8.445T – 2 100) = 8.445E(T) – 2 100 = 8.445 x 680.4 – 2 100 = 3 645.978

Var(G) = Var(8.445T – 2 100) = 8.4452Var(T) = 8.4452 x 68.04 = 4 852.478421 ⇒

⇒ σT = 69.65973314

( ) ≈

>−

=>469.6597331

645.978 3750 3469.6597331

645.978 3Gp750 3Gp

tipificando Aplicando el Teorema Central del Límite

)49.1(1 Φ−≈ = 1 – 0.9319 = 0.0681.

70º) Las variables X1 = “Nº de delitos cometidos anualmente en la ciudad A” y X2 = “Nº dedelitos cometidos anualmente en la ciudad B” se consideran independientes. Se sabe que lasdistribuciones de estas variables son normales con media 1.520 y desviación típica 50, en elprimer caso, y con media 480 y desviación típica 20, en el segundo caso. El número deefectivos policiales que anualmente se destina en cada ciudad es una v.a. que dependerá delnúmero de delitos cometidos. La relación en ambos casos es la siguiente:

2420XY 1

1 += 1210XY 2

2 +=

¿Con qué probabilidad la diferencia del número de efectivos entre las ciudades A y B seráinferior a 36?

SOLUCIÓN

( )10,2540,N1210X

20XYY 21

21 →+−=−

p(Y1 – Y2 < 36) = )25,125,104036p(Z −=−

< = 0,1056.

Page 99: ESTADÍSTICA EMPRESARIAL II PROBLEMAS DE EXAMEN …media0.webgarden.es/files/media0:4bd76ebb98146.pdf.upl... · 2010. 4. 27. · Estadística Empresarial II 23 La correlación negativa

Estadística Empresarial II 119

71º) Sea X =”Nº de días que una mercancía se encuentra depositada en un almacén” e Y =“Coste diario de almacenamiento” (cientos de euros), que depende del peso de la mercancía.La distribución conjunta de estas variables es la siguiente:

X \ Y 1 21 0,2 0,12 0,1 0,23 0,3 0,04 0,0 0,1

a) Calcular el coste medio diario sabiendo que el número de días que estará almacenada lamercancía será menor que 3. Compara este resultado con el coste medio diario dealmacenamiento e indica si este resultado induce a pensar en la dependencia entre lasvariables.b) Si la variable producto, XY, representa el coste total (número de días que estuvodepositada la mercancía x coste diario), calcular la probabilidad de que el coste total seainferior a 500 €.

SOLUCIÓN

a) E(Y/X ≤ 2) = 1,5 ⇒ 150 €. E(Y) = 1,4 ⇒ 140 €.

Si las variables fuesen independientes, las medias deberían ser iguales.

b) p(XY < 5) = 1 – p(XY ≥ 5) = 1 – p(X=3,Y=2) – p(X=4,Y=2) = 1 – 0 – 0,1 = 0,9.

72º) Sea T = aX + bY + 2 donde a, b∈ R y X e Y son dos variables aleatorias tales que:

E(X) = 3 E(Y) = 6 E(X2) = 34 E(Y2) = 136 E(XY) = 58.

Indicar qué relación deben cumplir a y b para que los coeficientes de correlaciónYT,XT, ρρ =− sean iguales

SOLUCIÓN

Var(–X) = (–1)2 Var(X) = 25 Var(Y) = 100

( ) ( ) ( ) ( ) 40b25aYX,bCovXaVarXbY,aXCovXT,Cov −−=−−=−+=−

( ) ( ) ( ) ( ) 0b10a40YbVarYX,aCovYbY,aXCovYT,Cov +=+=+=

( ) ( )YT,

TYTXXT, ρ

σσYT,Cov

σσX-T,Covρ ===

−−

YT,XT, ρ10

100b40a5

40b25a-ρ =+

=−

=−

⇒ a = –2b

Page 100: ESTADÍSTICA EMPRESARIAL II PROBLEMAS DE EXAMEN …media0.webgarden.es/files/media0:4bd76ebb98146.pdf.upl... · 2010. 4. 27. · Estadística Empresarial II 23 La correlación negativa

120 Problemas de examen resueltos

73º) Sea (X,Y) v.a. bidimensional con función de densidad conjunta:

<<<<+=

resto el en0

2y04;x0xy)(1241

y)(x,f YX,

Calcular ( )2XYE = .

SOLUCIÓN

( ) ∫∞=

−∞= ===

y

ydy(y)fy2X

YE2X

Y

Puesto que (x)f

y)(x,f(y)f

X

YX,

xXY =

= calculamos la marginal de X.

( ) 4x012

x12x2241

2yxy

241dyxy)(1

241dyy)(x,f(x)f

2

0

22

0YX,X<<

+=+=

+=+==

=

=

=

=

∞=

−∞= ∫∫y

y

y

y

y

y

2y04;x0x)2(1

xy1

12x1

xy)(1241

(x)fy)(x,f

(y)fX

YX,

xXY <<<<

++

=+

+==

=

Sustituyendo x por 2: 2y062y1(y)f

2XY <<

+=

=

( ) ∫ ∫∞=

−∞=

=

=

=

====

+=

+===

y

y 911

1822

3y2

2y

61dy

62y1ydy(y)fy2X

YE2y

0y

322y

0y2XY

74º) La duración en horas de un determinado servicio tiene distribución gamma deparámetros a = 3 y p = 1. Una persona alquila tres veces seguidas dicho servicio. Sabiendo quela duración de un servicio es independiente de los anteriores y que el coste es de 11 € la hora,calcular la distribución de la variable C, coste total y la esperanza de la v.a. C2 + 3C –5.

SOLUCIÓN

a) Sea X = Duración de un servicio en horas → γ(3,1)

X1, X2, X3, i.i.d γ(3,1) ⇒ Duración total = 321 XXX ++ → γ(3, 1 +1 + 1) ≡ γ(3,3)

Page 101: ESTADÍSTICA EMPRESARIAL II PROBLEMAS DE EXAMEN …media0.webgarden.es/files/media0:4bd76ebb98146.pdf.upl... · 2010. 4. 27. · Estadística Empresarial II 23 La correlación negativa

Estadística Empresarial II 121

Coste total = ( )321 XXX11C ++= →

,3113γ

b) 53E(C))E(C5)3CE(C 22 −+=−+

11

1133E(C) == ;

3121

1133(C)E)E(CVar(C) 2

22 =

=−=

de donde 3

484113

121(C)EVar(C))E(C 222 =+=+=

356853x11

348453E(C))E(C5)3CE(C 22 =−+=−+=−+

75º) El número medio de entradas diarias que se venden en una sala de cine es 100 con unadesviación típica de 50. El dueño quiere cerrar la sala para reformarla y los gastos de dichareforma ascienden a 17.000 €.a) Si el precio de la entrada es de 4 € y los costes fijos diarios son 100 €, ¿qué probabilidadestiene el propietario de recuperar el dinero invertido en un plazo de dos meses sin subir elprecio de la entrada?b) ¿Cuántos días necesitaría para recuperarse de la inversión con una probabilidad del 95%?

SOLUCIÓN

a) Sea X = Nº de entradas vendidas diariamente E(X) = 100 Var(X) = 2500

y B = Beneficio diario = 4X – 100 E(B) = 300 Var(B) = 40000

B1, …, B60, sucesión de v.a.i.id. con esperanza y varianza finitas, por el T.C.L.

( ) ( )6020018000,N60,60NB L60

1ii

≡σµ→∑=

=

−Φ−=

≤−=

> ∑∑

== 602001800017000117000Bp117000Bp

TCLelpor

60

1ii

60

1ii

( ) 0,74220,257810,651 =−=−Φ−=

Page 102: ESTADÍSTICA EMPRESARIAL II PROBLEMAS DE EXAMEN …media0.webgarden.es/files/media0:4bd76ebb98146.pdf.upl... · 2010. 4. 27. · Estadística Empresarial II 23 La correlación negativa

122 Problemas de examen resueltos

b) ( ) ( )n200300n,Nn,nµNB Ln

1ii

≡σ→∑=

0,95n200300n17000117000Bp117000Bp

TCLelpor

n

1ii

n

1ii

=

−Φ−=

≤−=

> ∑∑

==

1,645n200300n170000,05

n200300n17000

−=−

⇒=

Φ

017000n329300n =−−

Absurdo6,99n

65,55n8,096n

⇒−=

=⇒=

La solución válida es n ≈ 66

76º) Un propietario afirma que la ganancia en un determinado negocio, expresada en miles deeuros, viene representada por una v.a. X que sigue una distribución N(50,2). Tiene dosopciones: si elige la primera, correrá con unos gastos adicionales, cantidad que se puedeinterpretar como una v.a. Y1 con distribución N(14,2). En la segunda opción, los gastos sepresentan como una v.a. Y2 que sigue una distribución N(10,8). Gasto y ganancia se suponeque son independientes.a) Indicar qué opción es mejor desde el punto de vista de obtener un mayor beneficio portérmino medio.b) Si su meta se fija en obtener unos beneficios de 30.000 euros como mínimo, ¿cuál será lamejor opción desde el punto de vista probabilístico? Comentar el resultado, comparándolocon el del apartado anterior.

SOLUCIÓN

X → N(50,2) Y1 → N(14,2) Y2 → N(10,8)

B1 = X - Y1 → N(36, 8 ) B2 = X – Y2 → N(40, 68 )

a) Se obtiene mayor beneficio por término medio con la segunda opción: E(B1) = 36; E(B2) = 40.

b) p(B1 ≥ 30) =

−=−

≥ 2,1283630Zp = 1 – 0,0170 = 0,9830.

Page 103: ESTADÍSTICA EMPRESARIAL II PROBLEMAS DE EXAMEN …media0.webgarden.es/files/media0:4bd76ebb98146.pdf.upl... · 2010. 4. 27. · Estadística Empresarial II 23 La correlación negativa

Estadística Empresarial II 123

P(B2 ≥ 30) =

−=−

≥ 1,2168

4030Zp = 1 – 0,1131 = 0,8869.

Con la segunda opción, los beneficios medios son superiores a los de la primera; sin embargo, laprobabilidad pedida es mayor en el primer caso.

77º) De la distribución conjunta entre dos variables, conocemos la siguiente información:

X\Y 1 2 3 p(X=x)1 - - - 0,202 0,06 0,04 0,20 0,303 0,20 0,10 0,20 0,50

p(Y=y) - - - 1,00

Además, se sabe que las variables X y Y/X=1 (condicionada) están igualmente distribuidas.Completar la tabla y hallar el coeficiente de correlación. Comentar el resultado.

SOLUCIÓN

Si X y X/Y=1 están igualmente distribuidas, toman los mismos valores con las mismasprobabilidades.

X p(X=x) Y/X = 1 P(Y = y/X = 1)1 0,20 1 0,202 0,30 2 0,303 0,50 3 0,50

⇒ p(X = 1, Y = y) = p(Y = y/X = 1) p(X = 1) = p(Y = y/X = 1) x 0,20

Por ejemplo:

p(X = 1,Y = 1) = p(Y = 1/X = 1) p(X = 1) = p(Y = 1/X = 1) x 0,20 = 0,20 x 0,20 = 0,04.

X\Y 1 2 3 P(X=x)1 0,04 0,06 0,10 0,202 0,06 0,04 0,20 0,303 0,20 0,10 0,20 0,50

P(Y=y) 0,30 0,20 0,50 1,00

E(X) = 2,3 E(X2) = 5,9 Var(X) = 0,61 E(Y) = 2,2 E(Y2) = 5,6 Var(Y) = 0,76

E(XY) = 4,94 Cov(X,Y) = –0,120,76x0,61

0,12ρXY

−= = –0,1762.

78º) Sean X e Y dos variables aleatorias independientes con media 0 y desviación típica 1, ysean S y T:

Page 104: ESTADÍSTICA EMPRESARIAL II PROBLEMAS DE EXAMEN …media0.webgarden.es/files/media0:4bd76ebb98146.pdf.upl... · 2010. 4. 27. · Estadística Empresarial II 23 La correlación negativa

124 Problemas de examen resueltos

3bY2XS ++

=a

2YXT −= con a, b ∈ R

Calcular el valor de a y b para que E(S) = 1 y Var(T) = 5.Calcular Cov(S,T) para cualquier par de valores a y b. ¿Son S y T variables independientes?

SOLUCIÓN

( ) ( ) ( ) 13b

3bYE

31XE

32SE ==++= ⇒ b = 3

( ) ( )( ) ( ) 52a

5412a

14Var(Y)XVar2a

1TVar ==+=+=

luego a = 1 ó a = –1.

=

−++

=a

2YX,3

bY2XCovT)Cov(S,

( ) ( ) ( ) ( )( ) 0Y2VarYX,CovYX,4CovX2Var3a1

=−+−=

por lo tanto, S y T están incorreladas, pero no son necesariamente independientes.

79º) Sea (X,Y) v.a. bidimensional con función de densidad conjunta:

><<

=−

resto el en00y1;x0e6x

y)(x,f2y2

YX,

¿Qué distribución siguen X e Y? ¿Son independientes? Hallar las esperanzas de X e Y.Calcular ( )1Y0,5;Xp ≥≤

SOLUCIÓN

a) =−== ∫∫∞=

=

∞=

∞=

y

y 0

y

-y YX,Xdy2ye26xdyy)(x,f(x)f

1x023x2

2ye26xdy2ye26x

y

0y

y

0y<<=

−−=−=

∞=

=

∞=

=∫

0y2ye232x2yedx26x2yedx2ye26xdxy)(x,f(y)f1x

0x

1x

0x

1x

0 x

x

-x YX,Y>−=

−=−=−==

=

=

=

=

=

=

∞=

∞= ∫∫∫

Page 105: ESTADÍSTICA EMPRESARIAL II PROBLEMAS DE EXAMEN …media0.webgarden.es/files/media0:4bd76ebb98146.pdf.upl... · 2010. 4. 27. · Estadística Empresarial II 23 La correlación negativa

Estadística Empresarial II 125

luego X e Y son independientes pues yx,(y)f (x)fy)(x,fYXYX,

∀=

además X → β (3, 1) e Y → ε(2).

Por tanto, E(X) = 3/4 y E(Y) = 1/2.

b)

( ) =

−=

==≥≤ ∫∫ ∫∫ ∫

=

=

∞=

=

−=

=

∞=

=

−=

−∞=

∞=

=

0,5x

0x

y

1y

2y20,5x

0x

y

1y

2y20,5x

x

y

1y YX,dx

2e6xdxdye6xdydxy)(x,f1Y0,5;Xp

( ] 0,01698

exedx3xe2

0,50

320,5

0

22 ====−

=

=−=

=

− ∫xx

x

x (= p(X ≤ 0,5) p(Y ≥ 1)).

80º) El tiempo de fabricación de una determinada pieza (en horas) es una variable aleatoriacon distribución triangular de parámetros 2, 4 y 7.Calcular la probabilidad de tardar más de 3 horas en fabricar una pieza.Si a lo largo de una semana se fabrican 10 piezas. ¿Cuál es la probabilidad de que al menos endos de las piezas se hayan invertido como mucho tres horas en su fabricación?

SOLUCIÓN

Sea X = Tiempo de fabricación de una pieza (en horas).

a) Sustituyendo el valor de los parámetros, la función de densidad de X será:

≤≤−

≤≤−

=

resto elen 0

7x4x)(7152

4x22)(x51

(x)fX

0,92x7x

1522x

2x

51dxx)(7

152dx2)(x

51dx(x)f3)p(X

7

4

24

3

27

4

4

33 X=

−+

−=−+−==> ∫∫∫

b) Sea la v.a. Y = “Número de piezas en las que se han invertido menos de tres horas en sufabricación de las 10 producidas”.

Y → B(10, p) siendo la probabilidad de éxito: 0,10,913)p(X13)p(Xp =−=>−=≤= .

[ ]==+=−=<−=≥ 1)p(Y0)p(Y12)p(Y12)p(Y 0,2639.0,90,11

100,90,1

010

1 91100 =

+

+

Page 106: ESTADÍSTICA EMPRESARIAL II PROBLEMAS DE EXAMEN …media0.webgarden.es/files/media0:4bd76ebb98146.pdf.upl... · 2010. 4. 27. · Estadística Empresarial II 23 La correlación negativa

126 Problemas de examen resueltos

81º) Unos pintores estiman que en pintar una casa de hasta 120 m.2, tardan por términomedio 3 días con una desviación típica de 0,6 y en pintar una casa de mayor tamaño, tardanpor término medio 5 días con una desviación típica de 1,4. Se supone que el equipo de pintoresno realiza de manera simultánea trabajos en casas de tamaños diferentes. Este año se hancomprometido con 40 casas “pequeñas” y 30 casas “grandes”.a) Calcular la probabilidad de que pasen más días pintando casas “pequeñas” que “grandes”.b) Calcular la probabilidad de que les queden al menos 120 días libres al año entre fines desemana y vacaciones.

SOLUCIÓN

X = Tiempo que tardan en pintar una casa “pequeña” E(X) = 3 Var(X) = 0,36

Y = Tiempo que tardan en pintar una casa “grande” E(Y) = 5 Var (Y) = 1,96

X1, …, X40 es una sucesión de v.a.i.i.d. con esperanza y varianza finitas ⇒ (T.C.L.)

( )14,4120,NX L40

1ii →∑

=

Y1, …, Y30 es una sucesión de v.a.i.i.d. con esperanza y varianza finitas ⇒ (T.C.L.)

( )8,58150,NY L30

1ii →∑

=

a) ( )73,2,30NYX L30

1ii

40

1ii −→− ∑∑

==

( ) 0,00020,999813,51173,23010YXp

30

1ii

40

1ii

=−=Φ−=

Φ−=

>− ∑∑

==

b) ( )73,2270,NYX L30

1ii

40

1ii →+ ∑∑

==

( ) 0,0018.2,9273,2

270245524YXp30

1ii

40

1ii

=−Φ=

−Φ=

≤+ ∑∑

==

82º) Sea (X,Y) una variable aleatoria bidimensional con función de densidad conjunta:

fX,Y(x,y)

<<<<−=

resto0

1y02x0x)(2yx23 2

a) Calcular las funciones de densidad marginal de las variables aleatorias X e Y.

Page 107: ESTADÍSTICA EMPRESARIAL II PROBLEMAS DE EXAMEN …media0.webgarden.es/files/media0:4bd76ebb98146.pdf.upl... · 2010. 4. 27. · Estadística Empresarial II 23 La correlación negativa

Estadística Empresarial II 127

b) ¿Qué distribución sigue cada una de las variables?c) ¿Son X e Y v. a. independientes?

d) Calcular P(X > 1, Y < 21 ).

e) Calcular E(X) y

= yYXE . Comenta los resultados.

f) Calcular Cov(3X - 8, 2Y + 5).

SOLUCIÓN

a) fX(x) = x)(2x43

2yx)(2x

23dyyx)(2x

23dyx)(2yx

23 2

1y

0y

221y

0y

21y

0y

2 −=

−=−=−

=

=

=

=

=

= ∫∫

Luego fX(x) =

<<−

resto0

2x0x)(2x43 2

fY(y) = 2y8

3xxydx x)(2x23 ydxx)(2yx

23

2x

0x

432x

0x

22x

0x

2 =

−=−=−

=

=

=

=

=

= ∫∫ ≡ <<

resto01y02y

b) X sigue una distribución βe (3, 2, 0, 2) e Y sigue una distribución β (2, 1).

c) Sí son independientes X e Y porque fX,Y(x,y) = fX(x) fY(y), para todo x, y

d) p(X >1, Y<21 ) = p(X >1) p(Y<

21 ), por ser independientes X e Y

p(X >1) = 1611dxx)(2x

432x

1x

2 =−∫=

=; p( Y<

21 ) =

41dyy2

1/2

0=∫

=

=

y

y

Entonces, p(X >1, Y<21 )=

41x

1611 =

6411

e) Como X e Y son independientes,

= yYXE = E(X)

Como X sigue una βe (3, 2, 0, 2) E(X) = a + (b – a)qp

p+

= 56 .

f) Cov(3X – 8, 2Y + 5) = 3x2xCov(X,Y) = 0 puesto que las variables son incorreladas.

83º) En unos grandes almacenes, el número medio de devoluciones por hora en undeterminado departamento es de 4. Calcular:

a) La probabilidad de que no haya ninguna devolución en una hora.b) La probabilidad de que en una hora haya al menos una devolución.

Page 108: ESTADÍSTICA EMPRESARIAL II PROBLEMAS DE EXAMEN …media0.webgarden.es/files/media0:4bd76ebb98146.pdf.upl... · 2010. 4. 27. · Estadística Empresarial II 23 La correlación negativa

128 Problemas de examen resueltos

c) La probabilidad de que en 4 horas haya 10 devoluciones.d) De las 12 horas que permanece abierto ese gran almacén, ¿cuál es la probabilidad de quehaya como máximo 2 horas en las que no se produce ninguna devolución?e)Se sabe que en lo que va de hora ha habido ya al menos una devolución. ¿Cuál es laprobabilidad de que no se produzcan más de dos devoluciones durante esa hora?f) Si T = X2 – 5X + 3 es el tiempo en minutos que tarda el dependiente en realizar ladevolución, calcular el tiempo medio que tarda el dependiente en realizar una devolución.

SOLUCIÓN

a) X = nº de devoluciones por hora. X sigue una P(4). E(X) = 4. p(X = 0) = 40

e0!4 − = 0,0183.

b) p(X ≥ 1) = 1 – p(X < 1) = 1 – p(X = 0) = 1 – 0,0183 = 0.9817.

c) Xi = nº devoluciones en la hora i-ésima. Xi son independientes.

Y = X1 + X2 + X3 + X4 = nº de devoluciones en cuatro horas, que sigue una P(4x4)≡P(16).

p(Y = 10) = 1610

e10!16 − = 0,03401

d) Z = nº de horas en las que no hay devoluciones de las 12h. que permanece abierto.

Z sigue una B(n, p) ≡ B(12, 0,0183), puesto que p = p(éxito) = p(X = 0) = 0,0183.

p(Z ≤ 2) = p(Z = 0) + p(Z = 1)+ p(Z = 2) =

= 102111120 9817,00183,02

129817,00183,0

112

9817,00183,00

12

+

+

= 0,9988.

e) ( )1X2Xp ≥≤ = ==+=

=≥≤≤

0,98172)p(X1)p(X

1)p(X2)Xp(1

0,9817

e2!4e

1!4 4

24 −− +

2239,09817,02198,0

==

f) E(X) = Var(X) = 4 Var(X) = E(X2) – E(X)2 = E(X2) – 42. Entonces E(X2) = 20.

E(T)= E(X2 –5X + 3) = E(X2) – 5E(X) + 3 = 20 – 5x4 + 3 = 3 minutos es el tiempo medio que tardaen hacer la devolución.

84º) Sean X e Y variables aleatorias de las que sabemos los siguientes datos:

<<−

=resto el en0

1x0x)(1x6(x)fX

E(Y) = 2; Var(4 Y + 5) = 48; Cov(3X, 2Y) = 6.Calcular:

Page 109: ESTADÍSTICA EMPRESARIAL II PROBLEMAS DE EXAMEN …media0.webgarden.es/files/media0:4bd76ebb98146.pdf.upl... · 2010. 4. 27. · Estadística Empresarial II 23 La correlación negativa

Estadística Empresarial II 129

a) Var(X + Y) y Var (X – Y).b) Cov(2X + 3, X + Y).c) E(XY – 3Y2 + 5).

SOLUCIÓN

X sigue una distribución beta de parámetros 2, 2. Por tanto:

201

2)1)(22(22x2Var(X);

21

222E(X) 2 =

+++==

+=

3Var(Y)48Var(Y)45)Var(4Y 2 =⇒==+ 1Y)Cov(X,6Y)2x3xCov(X,3Y)Cov(2X, =⇒==

a) 20

101Y)Cov(X,2Var(Y)Var(X)Y)Var(X =++=+ ;

2021Y)Cov(X,2Var(Y)Var(X)Y)Var(X =−+=−

b) =+=+++=++ Y)Cov(X,2X)Cov(X,2Y)3,XCov(2X)3,XCov(2Y)X3,XCov(2

1021Y)Cov(X,2Var(X)2 =+=

c) E(Y) E(X) -Y) E(X Y)Cov(X, = ⇒ 2E(Y) E(X)Y)Cov(X,Y) E(X =+=

(Y)E -)E(YVar(Y) 22= ⇒ 7(Y)EVar(Y))E(Y 22 =+=

Luego, 145)E(Y 3E(XY)5)3YE(XY 22 −=+−=+−

85º) Las facturas cobradas por los servicios técnicos de reparación de una empresa, constande dos conceptos: piezas y mano de obra. La media del primer concepto es 3,60 € y la delsegundo 12 €. Las desviaciones típicas respectivas son 0,90 € y 1,20 €. La relación entre lascantidades facturadas por ambos conceptos viene expresada por un coeficiente de correlaciónigual a 0,56.

a) Calcula la esperanza y varianza de la facturación total de un servicio.b) ¿Cuál es la probabilidad de que con 100 servicios se alcance al menos una facturación totalde 1.530 €?c) En el supuesto de que solo se realizasen nueve servicios y las cantidades facturadas enambos conceptos fueran independientes y normalmente distribuidas, calcular la probabilidadde que las cantidades ingresadas en concepto de mano de obra superen a las obtenidas porrecambio de piezas en 79,2 € como máximo. ¿Qué ocurriría si las distribuciones no fuerannormales?

SOLUCIÓN

Page 110: ESTADÍSTICA EMPRESARIAL II PROBLEMAS DE EXAMEN …media0.webgarden.es/files/media0:4bd76ebb98146.pdf.upl... · 2010. 4. 27. · Estadística Empresarial II 23 La correlación negativa

130 Problemas de examen resueltos

a) Sean: Xi = facturación por piezas en el servicio i-ésimoYi = facturación por mano de obra en el servicio i-ésimoTi = facturación total en el servicio i-ésimo = Xi + Yi

E(Xi) = 3,6 Var (Xi) = 0,81 E(Yi) = 12 Var (Yi) = 1,44 ρXY = 0,56

Y)Var(X)Var(Y)Cov(X,ρ

XY= ⇒ Cov(X,Y) = Y)Var(X)Var(ρ

XY = 0,56 x 0,9 x 1,2 = 0,6048

Luego: E(Ti) = E(Xi) + E(Yi) = 15,6

Var(Ti) = Var (Xi) + Var (Yi) + 2 Cov(Xi, Yi) = 3,4596 ⇒iT

σ = 1,86

b) Sea Z = facturación de 100 servicios = ∑=

100

1iiT

Las Ti son variables aleatorias i.i.d. con esperanza y varianza finitas, luego teniendo en cuenta que:

1.56015,6x100TE100

1ii ==

∑=

96,4534596,3x100TVar100

1ii ==

∑=

por el Teorema Central del Límite:

1)N(0,18,6

1.560TL

100

1ii

→−∑

=

( ) ( ) 0,94630,053711,61Φ118,6

1.5601.530Φ11.530ZpT.C.L.

=−=−−=

−−≅≥

c) Si las v.a. son normales e independientes, por las propiedades de la normal:

E(Di) = E(Yi) – E(Xi) = 8,4 Var(Di) = Var(Xi) + Var (Yi) = 2,25

Di = Yi – Xi → N(8,4, 1,5)

∑=

9

1iiD → N(75,6, 4,5) ⇒ ( ) 0,78810,879,2Dp

9

1ii =Φ=

≤∑

=

Si las variables no son normales no se puede calcular la probabilidad.

86º) Un agente inmobiliario está interesado en averiguar cuál es la relación entre el número delíneas de un anuncio en prensa sobre un apartamento y el volumen de demanda deinformación por parte de posibles inquilinos. Llamemos X a la variable aleatoria número delíneas e Y a la variable aleatoria volumen de demanda, que toma el valor 0, si despierta poco

Page 111: ESTADÍSTICA EMPRESARIAL II PROBLEMAS DE EXAMEN …media0.webgarden.es/files/media0:4bd76ebb98146.pdf.upl... · 2010. 4. 27. · Estadística Empresarial II 23 La correlación negativa

Estadística Empresarial II 131

interés, 1 para un interés moderado y 2, si despierta un fuerte interés. El agente estima que laley de probabilidad conjunta de ambas variables es:

Volumen de demanda (Y)Número de líneas (X) 0 1 2

3 0,03 0,06 0,084 0,04 0,20 0,165 0,10 0,18 0,15

a) Calcular las leyes de probabilidad marginales de X e Y. ¿Son independientes?b) Calcular la ley de probabilidad de Y, sabiendo que el número de líneas es 4 y calcular suesperanza.c) Calcular ( )4X

1Yp >≥ e interpretar el resultado.

d) Calcular la covarianza de X e Y e interpretar el resultado.

SOLUCIÓN

a)X\Y 0 1 2 p(X=x)

3 0,03 0,06 0,08 0,174 0,04 0,20 0,16 0,405 0,10 0,18 0,15 0,43

p(Y=y) 0,17 0,44 0,39 1,00

No son independientes; por ejemplo: 0,17 x 0,17 ≠ 0,03.

b)

Y/X=4

==

4XyYp j

0 0,04/0,40 = 0,101 0,20/0,40 = 0,502 0,16/0,40 = 0,40

E(Y/X=4) = 0x0,10 + 1x0,5 + 2x0,4 = 1,3

c) ( ) 0,76744333

0,430,150,18

5)p(X1)Y5;p(X

4X1Yp ==

+=

=≥=

=>≥ .

Cuando el número de líneas del anuncio es muy alto (5), existe una alta probabilidad (76,74%) deque la demanda sea moderada o alta.

d) E(XY) = 5,14 E(X) = 4,26 E(Y) = 1,22 0,0572E(Y)E(X)Y)E(XY)Cov(X, −=−= .

Esto implica que hay una relación inversa entre el número de líneas y el volumen de demanda.

87º) Sea X1 la recaudación, en euros, por un artículo en un kiosko de lunes a viernes. Sabemosque se distribuye normalmente, con media 150 y desviación típica 3. Sea X2 la recaudaciónpor el mismo concepto durante el fin de semana. Su distribución es N(175, 6).

Page 112: ESTADÍSTICA EMPRESARIAL II PROBLEMAS DE EXAMEN …media0.webgarden.es/files/media0:4bd76ebb98146.pdf.upl... · 2010. 4. 27. · Estadística Empresarial II 23 La correlación negativa

132 Problemas de examen resueltos

a) Indicar en qué caso está más asegurada la recaudación, en los cinco primeros días de lasemana o durante el fin de semana, teniendo en cuenta la dispersión en cada caso.b) Calcular la probabilidad de que la recaudación total semanal sea superior a 315 €.c) Calcular la probabilidad de que se recaude más de lunes a viernes que durante el fin desemana.

d) Supongamos que las variables son dependientes y su coeficiente de correlación es 32 .

Sabiendo que la recaudación total bajo el caso de dependencia sigue una distribución normal,calcular la esperanza y la varianza, así como la probabilidad de que tome un valor superior a315 €. Compararla con el caso de independencia.

SOLUCIÓN

a) Para el caso del fin de semana (X2), la varianza es superior que la de la primera variable. Estoinduce a pensar que podemos obtener recaudaciones diferentes a la media, más que con el primercaso.

b) X1 + X2 → )45N(325,

p(X1 + X2 > 315) =

−=−

> 1,4945

325315Zp = 1 – ( )1,49Φ − = 1 – 0,0681 = 0,9319.

c) p(X1 > X2) = p(X1 – X2 > 0). Como X1 – X2 → )4525,N(− .

P(X1 > X2) = p(X1 – X2 > 0) =

=−−

> ,73345

25)(0Zp = 1 – ( ),733Φ = 1 – 0,9999 = 0,0001.

d) Como 32

Y)Var(X)Var(Y)Cov(X,ρ

XY== ⇒ Cov(X,Y) = Y)Var(X)Var(ρ

XY = 6x3x

32 = 12

E(X1 + X2) = 325; Var(X1 + X2) = Var(X1) + Var(X2) + 2 Cov(X,Y) = 9 + 36 + 2x12 = 69

Entonces, X1 + X2 → )69N(325, .

P(X1 + X2 >315) =

−=−

> 1,2069

325315Zp = 1 – ( )1,20Φ − = 1 – 0,1151 = 0,8849.

En este caso, la probabilidad pedida es algo inferior al caso de independencia.

88º) El tiempo que tarda un trabajador de un servicio técnico en realizar cada reparación,

expresada en días, es una v.a. que sigue una distribución

214,γ .

a) Si suponemos que las tareas que realiza son independientes, calcular la probabilidad de quetarde menos de medio día en cubrir dos servicios.

Page 113: ESTADÍSTICA EMPRESARIAL II PROBLEMAS DE EXAMEN …media0.webgarden.es/files/media0:4bd76ebb98146.pdf.upl... · 2010. 4. 27. · Estadística Empresarial II 23 La correlación negativa

Estadística Empresarial II 133

b) Si se tuvieran que realizar 50 reparaciones, plantear la probabilidad exacta de que eltiempo empleado en total no fuese superior a los 5 días y buscar mediante algunaaproximación su valor, comentando las condiciones que se deben cumplir para ello.

SOLUCIÓN

Por cada trabajo defino una v.a.: Ti = “Tiempo que se tarda en una reparación” →

214,γ . Para dos

tareas independientes, el tiempo total T → ( )4,1γ ≡ ( )4ε .

a)

<+

21TTp

21 = dte4

0,5

0

t4∫ − = 2e1 −− = 1 – 0,1353 = 0,8647.

b) T = ∑=

50

1iiT → ( )4,25γ .

La probabilidad exacta: 5)p(T < = ( ) dtet25Γ

4 t4245

0

25−∫ . Se resolvería integrando por partes.

Dado que disponemos de 50 v.a.i.i.d., con esperanza y varianza finitas, podemos aplicar el T.C.L.para resolver esta cuestión.

E(T) = 425 ; Var(T) =

1625 ⇒ 1)p(Z5)p(T

T.C.L.−<≈< = 0,1587.

89º) Sea X = “Tiempo que se dedica a realizar una actividad”, expresada en horas, que sigueuna distribución con función de densidad:

( )

<<+=

restoelen0

2x01)x41

xfX(

a) Calcular E(X) y la Var(X).b) Calcular la probabilidad de que el tiempo empleado sea superior a hora y media.c) Si se realizasen 10 actividades del mismo tipo, bajo las mismas condiciones, calcular laprobabilidad de que, exactamente en 3 de ellas el tiempo que se emplea en realizar cada unasea superior a hora y media.

SOLUCIÓN

a)E(X) = dx1)(x41x

2

0+∫ =

67 E(X2) = dx1)(x

41x

2

0

2 +∫ = 35

Var(X) = E(X2) – E2(X) = 35 –

2

67

=

3611

b) p(X > 1,5) = dx1)(x412

5,1+∫ =

3211 = 0,34375.

Page 114: ESTADÍSTICA EMPRESARIAL II PROBLEMAS DE EXAMEN …media0.webgarden.es/files/media0:4bd76ebb98146.pdf.upl... · 2010. 4. 27. · Estadística Empresarial II 23 La correlación negativa

134 Problemas de examen resueltos

c) Y = nº actividades entre las 10 en las que el tiempo que se emplea en realizar es mayor que 1,5 h.

Y → B(10, 3211 ) ⇒ P(Y = 3) =

73

3221

3211

310

= 0,2555.

90º) Sean X e Y dos variables aleatorias continuas con función de densidad conjunta:

<<<=

restoelen0

4xy081

y)(x,fYX,

a) Calcular las funciones de densidad marginales de las variables X e Y.b) ¿Qué distribución conocida siguen cada una de las variables X e Y?c) ¿Son X e Y variables independientes?d) Calcular las funciones de densidad condicionadas (x)f

yX/Y= y )(yf

xY/X=.

e) Calcular ( )3Y/XE = .f) Calcular el coeficiente de correlación lineal e interpretar su resultado.g) Calcular la esperanza y la varianza de la variable Z = 3X – 2Y + 1.h) Calcular p(X – 2Y > 0).

i) Calcular la función de densidad de la variable 8X -2T= .

SOLUCIÓN

a)

<<==∫

=

= resto elen 0

4x08x

dy 81(x)f

xy

0y X

<<

−==∫

=

= resto elen 0

4y08

y4 dx

81(y)f

4 x

y xY

b) X → βe(2, 1, 0 ,4) Y → βe(1, 2, 0 ,4)

c) X e Y no son independientes porque (y)f(x)fy)(x,f YXYX, ≠

d)

4y0 que siempreresto elen 0

4xyy4

1

(y)fy)(x,f

(x)fY

YX,yX/Y

<<

<<

−===

4x0 que siempreresto elen 0

xy0x1

(x)fy)(x,f

(y)fX

YX,xY/X

<<

<<===

e)

<<==

resto elen 0

3y031

(y)f 3Y/X ( )23 dy

31ydy(y)fy3Y/XE

3y

0y

3y

0y 3Y/X ==== ∫∫=

=

=

= =

f) 38

qpp a)(baE(X) =+

−+=34

qpp a)(baE(Y) =+

−+=

Page 115: ESTADÍSTICA EMPRESARIAL II PROBLEMAS DE EXAMEN …media0.webgarden.es/files/media0:4bd76ebb98146.pdf.upl... · 2010. 4. 27. · Estadística Empresarial II 23 La correlación negativa

Estadística Empresarial II 135

(Y)Var98

1)q(pq)(ppqa)(b(X)Var 2

2==

+++−

=

4dx dy81yx dx dy y)(x,fy x E(XY)

4 x

0 x

xy

0y

4 x

0 x

xy

0y YX,=

=

= ∫ ∫∫ ∫

=

=

=

=

=

=

=

=

Cov(X, Y) = E(XY) – E(X) E(Y) = 4 – 8/3 4/3 = 4/9

21

9/89/89/4

Var(Y)Var(X)Y)X,(Covρ YX, ===

g) E(Z) = E(3X – 2Y + 1) = 3E(X) – 2E(Y) + 1 = 19/3

Var(Z) = Var(3X – 2Y + 1) = 9Var (X) + 4Var (Y) – 2x3x2Cov (X, Y) = 56/9

h) 21dx dy

81dx dyy)(x,f 0) 2Y X p(

4 x

0 x

x/2y

0y

4 x

0 x

x/2y

0y YX,=

=

=>− ∫ ∫∫ ∫

=

=

=

=

=

=

=

=

j) t))(8(2F1)t)8(2Xp(t)T(p(t)FXT

−−=−>=<=

<<−

=−−−=resto elen 0

2t 3/2t)(288)(t))(8(2f(t)f

XT

91º) En una gasolinera se hace un estudio sobre dos aspectos de su funcionamiento: el gastoque realizan sus clientes y el tiempo que se tarda en realizar el pago. El gasto de los clientespor el llenado del depósito sigue una distribución normal de media 50 € y desviación típica 4€, pero los pagos con tarjeta de crédito tienen un coste para la gasolinera de un 3% encomisión bancaria. Por otra parte, el tiempo en minutos que tarda un cliente en realizar elpago sigue una distribución U(1, 3) si lo efectúa al contado, y una distribución γ(3, 9) si utilizatarjeta.

a) ¿Cuál es la distribución de los ingresos de un día en el que se realizan 30 pagos al contado y100 con tarjeta? ¿Cuál es la probabilidad de dichos ingresos sean como mucho 6.400 €?b) En ese mismo día, ¿cuál es la probabilidad de que la persona encargada de la caja hayaestado ocupada cobrando, al menos durante cinco horas y media?c) Cierto día, solo se realizaron pagos con tarjeta. Con una probabilidad de 0,9772, ¿cuántos

pagos es previsible que se hicieran, si la persona que atiende la caja no estuvo ocupada másde cinco horas y veinte minutos?

SOLUCIÓN

Consideremos las siguientes variables:

E = Ingresos por pagos en efectivo → N(50, 4)T = Ingresos por pagos con tarjeta = 0,97E → N(48,5, 3,88)X = tiempo de un pago en efectivo → U(1, 3) E(X) = 2 Var(X) = 1/3

Page 116: ESTADÍSTICA EMPRESARIAL II PROBLEMAS DE EXAMEN …media0.webgarden.es/files/media0:4bd76ebb98146.pdf.upl... · 2010. 4. 27. · Estadística Empresarial II 23 La correlación negativa

136 Problemas de examen resueltos

Y = tiempo de un pago con tarjeta → γ(3, 9) E(Y) = 3 Var(Y) = 1

a) ( )1.985,446.350,NTEI100

1ii

30

1ii →+= ∑∑

==

( ) 0,86861,121.985,44

6.3506.4006.400)p(I =Φ=

−Φ=<

b) ( )1060,NX L30

1ii →∑

=

( )10300,NY L100

1ii →∑

=

( )110360,NYXT L100

1ii

30

1ii →+= ∑∑

==

( ) ( ) 9979,086,2 1110

360330110

360Tp1033TpT.C.L.otipificand

=−Φ−=

−<

−−=≥

c) ( )nn,3NY Ln

1ii →∑

=

0,9772n

3n320n

3n320n

3nYp320Yp

T.C.L.

n

1ii

otipificand

n

1ii

=

−Φ=

≤−

=

∑∑ =

=

2n

n3032=

− 3n + 2 n – 320 = 0

n = 10 ⇒ n = 100 n = –10,6667 (esta solución no vale)

92º) Un comercial de una empresa ha estado estudiando los valores de X = Número de visitasque realiza al día, e Y = Número de ventas efectuadas al día; obteniendo los siguientesresultados de la distribución conjunta:

X \ Y 2 3 44 0,08 0,02 0,005 0,07 0,07 0,066 0,04 0,10 0,167 0,01 0,11 0,28

Calcular:

Page 117: ESTADÍSTICA EMPRESARIAL II PROBLEMAS DE EXAMEN …media0.webgarden.es/files/media0:4bd76ebb98146.pdf.upl... · 2010. 4. 27. · Estadística Empresarial II 23 La correlación negativa

Estadística Empresarial II 137

a) el número medio y la varianza de las visitas y de las ventas efectuadas.b) ¿son independientes las variables?c) el coeficiente de correlación entre ambas variables. Interpretarlo.d) el número medio de ventas al día cuando realiza 5 ó menos visitas al día.e) la probabilidad de que haya efectuado como mínimo 5 visitas si ha efectuado 3 ventas.f) la probabilidad de que en 10 días de trabajo en al menos 6 haya efectuado 4 ventas.

SOLUCIÓN

X \ Y 2 3 4 P(X = x)4 0,08 0,02 0,00 0,105 0,07 0,07 0,06 0,206 0,04 0,10 0,16 0,307 0,01 0,11 0,28 0,40

P(Y = y) 0,20 0,30 0,50 1,00

a) E(X) = 6 E(Y) = 3,3 E(X2) = 37 E(Y2) = 11,5Var(X) = 1 Var(Y) = 0,61

b) fX(4) fY(2) = 0,10 x 0,20 = 0,02 ≠ 0,08 = fX,Y(4, 2) ⇒ X e Y son dependientes

c) E(XY) = 20,24 Cov(X, Y) = 0,44 ρ(X, Y) = 0,56336227

d)Y/X ≤ 5 2 3 4

P(Y/X ≤ 5) 15/30 9/30 6/30

E(Y/X ≤ 5) = 2x0,5 + 3x0,3 + 4x0,2 = 2,7

e) p(X ≥ 5 / Y = 3) = p(X ≥ 5, Y = 3) / p(Y = 3) = 28/30

Sea Z = Número de días en los que efectúa 4 ventas de los 10 días → B(10, 0,5)

p(Z ≥ 6) = 1 – p(Z < 6) = 1 – p(Z ≤ 5) = 1 – 0,6230 = 0,3770.

93º) Sea la variable aleatoria X = Porcentaje (en tanto por uno) de las visitas realizadasmensualmente de las previamente asignadas a un comercial. Se sabe que esta variable sigueuna distribución β(2, 1).

a) Obtener el porcentaje medio de visitas y su varianza.b) Calcular p(X < 0,1) y p(X > 0,9). Comparando ambas probabilidades, ¿se podría deducir laidea de que es más probable que el comercial realice un número elevado de visitas?c) Sabiendo que el salario del comercial se compone de una parte fija de 900 € y otra variableque es proporcional al porcentaje de visitas realizadas con un máximo de 300 €, construir lavariable salario mensual del comercial. Demostrar que sigue una distribución βe(2, 1, 900,1.200). Obtener el salario medio y su varianza.d) La empresa tiene a su disposición 150 comerciales en activo, todos ellos con el mismosistema salarial. Calcular la probabilidad aproximada de que la cantidad total dedicada asalarios por la empresa mensualmente sea superior a 168.000 €.

Page 118: ESTADÍSTICA EMPRESARIAL II PROBLEMAS DE EXAMEN …media0.webgarden.es/files/media0:4bd76ebb98146.pdf.upl... · 2010. 4. 27. · Estadística Empresarial II 23 La correlación negativa

138 Problemas de examen resueltos

SOLUCIÓN

a) X → β(2, 1) ⇒ E(X) = qp

p+

= 32 ; Var(X) = 2q)(p1)q(p

pq+++

= 181

b) ( ) <<

=restoelen0

1x0x2xfX

p(X < 0,1) = ∫0,1

0dxx2 = ( ] 0,1

02x = 0,01; p(X > 0,9) = ∫

1

0,9dxx2 = ( ] 1

0,92x = 0,19.

Es más probable que realice un porcentaje alto visitas.

c) Si X → β(p, q), aplicando el cambio de variable: Y = (b – a)X + a → βe(p, q, a, b).

En nuestro caso, p = 2, q = 1, a = 900 y b = 1.200. Entonces, Y → βe(2, 1, 900, 1.200).

E(Y) = (b – a) E(X) + a = 300 32 + 900 = 1100 Var(Y) = (b – a)2 Var(X) = 3002

181 = 5000

d) Defino Y1, …, Y150, v.a.i.i.d., siendo Yi = “salario mensual del i-ésimo comercial”.

E(Yi) =1100; Var(Yi) = 5000. Entonces, ∑=

150

1iiY = “Salario total mensual”.

Su esperanza:

∑=

150

1iiYE = 150 x E(Yi) = 150 x 1100 = 165000

Su varianza:

∑=

150

1iiYVar = 150 x Var(Yi) = 150 x 5000 = 750000

El problema nos pregunta sobre

>∑

=

150

1ii 168000Yp . Como el número de variables implicadas es

superior a 30, podemos aplicar el teorema central del límite para resolver esta cuestión. Entonces:

>∑

=

150

1ii 168000Yp

... LCT≈

−>

750000165000168000Zp = ( )46,3Zp > = 1 - ( )46,3Zp ≤ =

= 1 – 0,9997 = 0,0003.

La probabilidad de que ocurra es muy baja.

Page 119: ESTADÍSTICA EMPRESARIAL II PROBLEMAS DE EXAMEN …media0.webgarden.es/files/media0:4bd76ebb98146.pdf.upl... · 2010. 4. 27. · Estadística Empresarial II 23 La correlación negativa

Estadística Empresarial II 139

94º) Sea (X, Y) una variable aleatoria bidimensional con función de densidad conjunta:

<<<<−

=resto el en0

2y01;x0xy1y)(x,f

YX,

a) Obtener las funciones de densidad marginales de X e Y, e indicar cuál es su distribución ysi coincide con alguna de las conocidas. ¿Son X e Y independientes?b) Calcular Cov(X, Y), sabiendo que Var(3X + 2Y) = 2.

c) Calcular

<>

0,5X1Yp .

SOLUCIÓN

a) 1x0x)2(12x22

xyyxy)dy(1(x)f2y

0y

22y

0yX<<−=−=

−=−=

=

=

=

=∫ X → β(1, 2)

2y0y)(221

2y1

2yxxxy)dx(1(y)f

1x

0x

21x

0xY<<−=−=

−=−=

=

=

=

=∫ Y → βe(1, 2, 0, 2)

Las variables no son independientes puesto que:

(y)(x)ff)y(221x)2(1xy1 y)(x,f

YXYX,=−−≠−=

b) Puesto que sabemos las distribuciones de X e Y:

181

q)1)(pq(ppqVar(X) 2 =+++

= ;92

q)1)(pq(ppqa)(bVar(Y) 2

2

=+++

−=

2Y)Cov(X,232Var(Y)2Var(X)32Y)Var(3X 22 =×××++=+ ⇒

⇒21611

12922

18132

Y)Cov(X,22

=−−

=

c)

<>

0,5X1Yp =

0,5)p(X0,5)X1,p(Y

<<>

( ) ( ) =

−=−=<> ∫ ∫∫ ∫

=

=

=

=

=

=

=

=

0,5x

0x

2y

1y

0,5x

0x

2y

1ydxdyxy1dydxxy10,5)X1,p(Y

165

2x

23xdx

23x1dx

2xyy

0,5x

0x

25,0

0x

5,0

0

2y

1y

2

=

−=

−=

−=

=

=

=

=

=

=

=

=∫∫

xx

x

Page 120: ESTADÍSTICA EMPRESARIAL II PROBLEMAS DE EXAMEN …media0.webgarden.es/files/media0:4bd76ebb98146.pdf.upl... · 2010. 4. 27. · Estadística Empresarial II 23 La correlación negativa

140 Problemas de examen resueltos

43

2xx2x)dx2(10,5)p(X

0,5x

0x

20,5x

0x=

−=−=<

=

=

=

=∫

<>

0,5X1Yp =

0,5)p(X0,5)X1,p(Y

<<> = 0,4167

125

43

165

==

95º) En una sala de cine con capacidad para 100 personas, se sabe que el gasto en otrosproductos (palomitas, chuches, refrescos, etc.) por cada persona que entra al cine es unavariable aleatoria que sigue una distribución normal de media 6 euros y que el 98,3% de laspersonas que entran en el cine se gastan en otros productos más de 1,76 euros cada una.

a) Calcular la probabilidad de que el gasto total en otros productos de todas las personas queentran en el cine un día que se llena la sala, esté comprendido entre 550 y 620 euros.b) Las 10 últimas sesiones se ha llenado la sala. ¿Cuál es la probabilidad de que en más deocho sesiones el gasto en otros productos estuviera comprendido entre 550 y 620 euros?

SOLUCIÓN

Sea X = gasto en otros productos por cada persona que entra al cine → N(6, σ)

0,983)σ4,24 p(Z1 )

σ61,76

σ6X ( p1,76)(X p =

−≤−=

−>

−=>

0,017000,9831)σ4,24 p(Z =−=

−≤ ⇒ 2,12

σ4,24

−=−

⇒ σ = 2 ⇒ X→N(6, 2)

a) Sea ∑=

=100

1iiXY = gasto total en otros productos de todas las personas que entran en el cine

cuando se llena la sala → N(600, 20)

0,8351 0,0062 0,84132,5)Φ(Φ(1)20

600620 20

600Y20

600550 p 620)Y p(550 =−=−−=

≤−

≤−

=≤≤

b) Sea W = número de sesiones en las que el gasto en otros productos estuvo entre 550 y 620 euros

W → B(10; 0,8351)

P(W > 8) = p(W = 9) +p(W = 10) = 01019 0,1649 0,8351 1010

0,1649 0,8351 9

10

+

= 0,490699288.

Page 121: ESTADÍSTICA EMPRESARIAL II PROBLEMAS DE EXAMEN …media0.webgarden.es/files/media0:4bd76ebb98146.pdf.upl... · 2010. 4. 27. · Estadística Empresarial II 23 La correlación negativa

Estadística Empresarial II 141

96º) En una subasta donde el lote se compone de un único artículo, sea X = “la cantidadofrecida por el artículo” (en cientos de euros) una variable aleatoria con función de densidad:

fX(x) =

<<

resto0

20x8121

a) Comenta el comportamiento del modelo probabilístico de la cantidad ofertada.b) Calcular la probabilidad de que la oferta por un artículo no supere los 1.600€.c) Una empresa desea adquirir 60 objetos para decorar una de sus salas de exposición. ¿Cuálserá la probabilidad de que no gaste más de 85.000€?d) Si el presupuesto fuese de 100.000€, ¿cuántos objetos podría adquirir como máximo, conunas garantías de, al menos, un 99,65%?

SOLUCIÓN

a) X → U(8, 20)

b) p(X < 16) = dx12116x

8x∫=

= = 0,6667.

c) X1, …, X100 v.a.i.i.d., E(Xi) = 14 y Var(Xi) = 12

720 1260XVar;8401460XE60

1ii

60

1ii =×=

=×=

∑∑==

p(∑=

60

1iiX ≤ 850) = p(

720840850

720

840X60

1ii −

≤−∑

= )TCL≈ p( 37,0Z ≤ ) = 0,6443

d) X1, …, Xn v.a.i.i.d., 12nXVar;14nXEn

1ii

n

1ii ×=

×=

∑∑==

p(∑=

n

1iiX ≤ 1.000) ≥ 0,9965 ⇒ p(

12n14n1.000

12n

14nXn

1ii −

≤−∑

= ) ≥ 0,9965

Aplicando el Teorema Central del Límite, 12n

14n1.000 − = 2,70 ⇒ 14n + 9,3531 n – 1000 = 0.

Las dos soluciones para n son 8,1241 y –8,7922. La segunda no tiene sentido, por tanto, n = 66.

Page 122: ESTADÍSTICA EMPRESARIAL II PROBLEMAS DE EXAMEN …media0.webgarden.es/files/media0:4bd76ebb98146.pdf.upl... · 2010. 4. 27. · Estadística Empresarial II 23 La correlación negativa

142 Problemas de examen resueltos

97º) Sea (X, Y) una v.a. bidimensional con función de densidad conjunta:

><<

=resto el en0

0y1;x0ex2y)(x,f

-y

YX,

a) Obtener las funciones de densidad marginales de X e Y. ¿Su distribución es conocida? ¿SonX e Y independientes?b) Calcular la esperanza y desviación típica de la variable Z = 3X – 5Y + 3.

c) Calcular

<>

0,5X1Yp .

SOLUCIÓN

a) ( ] 1x02x2xedy2xe(x)f y0y

yy

0y

yX

<<=−==∞=

=−∞=

=

−∫ X → β(2,1)

( ] 0yexedx2xe(y)f y1x0x

2y1x

0x

yY

>=== −=

=−=

=

−∫ Y → ε(1)

Las variables son independientes puesto que:

(y)(x)ff2xe2xe y)(x,f YXyy

YX, === −−

b) Puesto que conocemos la distribución:

E(X) = 32

qpp

=+

; E(Y)= 111

a1

== 181

q)1)(pq(ppqVar(X) 2 =

+++= ; Var(Y) = 1

11

a1

12 ==

y puesto que son independientes la covarianza es cero

=+−=+− 35E(Y)3E(X)3)5YE(3X 035x1323x =+−

( )25125x1

1819xVar(Y) 5Var(X)35Y)Var(3X3)5YVar(3X 22 =+=−+=−=+−

de donde la desviación típica será: 5,049753)5YVar(3X =+−

c)

<>

0,5X1Yp =

0,5)p(X0,5)X1,p(Y

<<>

=

==<> ∫ ∫∫ ∫

=

=

∞=

=

−=

=

∞=

=

− 0,5x

0x

y

1y

y0,5x

0x

y

1y

y dxdy2xedydx2xe0,5)X1,p(Y

( ] ( ] 10,5x0x

210,5x

0x

10,5x

0x

y1y

y e41xedx2xedx2xe −=

=−=

=

−=

=

∞=

=− ===−= ∫∫

Page 123: ESTADÍSTICA EMPRESARIAL II PROBLEMAS DE EXAMEN …media0.webgarden.es/files/media0:4bd76ebb98146.pdf.upl... · 2010. 4. 27. · Estadística Empresarial II 23 La correlación negativa

Estadística Empresarial II 143

( ]41x2xdx0,5)p(X 0,5x

0x20,5x

0x===<

=

=

=

=∫

<>

0,5X1Yp =

0,5)p(X0,5)X1,p(Y

<<> = 1

1

e

41

e41

= = 0,367879 = p(Y>1) por ser independientes.

98º) Las ventas diarias de una administración de lotería (en miles de euros) es una variablealeatoria que sigue una distribución normal con media 5 euros y varianza 4.

a) Si se consideran los meses de 30 días ¿cuál es la probabilidad de que en siete meses y medio,las ventas en esa administración de lotería estén comprendidas entre 1.122.000 euros y1.125.300 euros?b) ¿Cuál sería el número máximo de días para que las ventas de esa administración nosuperen los 56.760 euros con una probabilidad del 97,5%?

SOLUCIÓN

a) Sea X = Ventas diarias de la administración en miles de euros → N(5, 2)

Siete meses y medio son 225 días (7 x 30 + 15 = 225)

Sea ∑=

=225

1iiXY = Ventas en siete meses y medio de la administración en miles de euros

Y → N(225x5, 225 x2) ≡ N(1.125, 30)

=

≤−

≤−

=≤≤30

1.1251.125,3 301.125Y

301.1251.122 p 1.125,3)Y p(1.122 0,0438 0,1)Φ(Φ(0,01) =−−

b) Sea V = ∑=

n

1iiX = Ventas en n días de la administración en miles de euros → N(5n, 2 n )

0,975 )n2

5n56,76 p(Zn2

5n56,76n25nVp 56,76) V p( =

−≤=

−≤

−=≤

1,96n2

5n56,76=

−⇒ 5n + 3,92 n – 56,76 = 0 ⇒ n = 3 y n = -3,784

La solución negativa no tiene sentido, por tanto, n = 9

Page 124: ESTADÍSTICA EMPRESARIAL II PROBLEMAS DE EXAMEN …media0.webgarden.es/files/media0:4bd76ebb98146.pdf.upl... · 2010. 4. 27. · Estadística Empresarial II 23 La correlación negativa

144 Problemas de examen resueltos

99º) Una empresa que reparte a domicilio pedidos de comida rápida estima que el tiempo (enminutos) que dedica a cubrir cada servicio sigue una distribución gamma de parámetros a =0,25 y p = 4. Por otra parte, el ingreso medio por pedido es 24€, con una desviación típica de 2.

a) Indicar cuál es la distribución de la variable tiempo total dedicado a realizar 100 servicios,plantear la probabilidad de que en esos 100 servicios el tiempo total supere las 30 horas ycalcularla de forma aproximada.b) Si la compañía ha de servir 100 pedidos, obtener la probabilidad aproximada de que elingreso total que generan sea superior a 2.390€.

SOLUCIÓN

a) Sea X = Tiempo en minutos que se dedica a cubrir un servicio → γ(0,25; 4)

T = ∑=

100

1iiX = Tiempo total dedicado a realizar 100 servicios → γ(0,25; 400)

P(∑=

100

1iiX > 1.800) = ∫

∞=

=

−x

1.800x

0,25x399400

dxex399!

0,25

6.400 64100XVar1.600;16100XE100

1ii

100

1ii =×=

=×=

∑∑==

P(∑=

100

1iiX > 1.800) = p(

801.6001.800

80

1.600X100

1ii −

>−∑

= ) TCL≈ p( 2,5Z > ) = 1 – Φ(2,5) = 0,0062.

b) Sea Y = Ingreso por pedido Y1, …, Y100 v.a.i.i.d., E(Yi) = 24 y Var(Yi) = 4.

400 4100YVar2.400;24100YE100

1ii

100

1ii =×=

=×=

∑∑==

p(∑=

100

1iiY > 2.390) = p(

202.4002.390

20

2.400Y100

1ii −

>−∑

= ) TCL≈ p(Z > –0,5) = 1 – Φ(–0,5) = 0,6915.